Вы находитесь на странице: 1из 293

ENGINEERING MECHANICS

STATICS
Special Thanks
to
Mr. Vibhor Sharma, Assistant Professor, Transportation,
MIT Institute of Design, Pune
for
drawing artful illustrations and figures using Photoshop software
which has enhanced the effectiveness of the text.
ENGINEERING MECHANICS
STATICS

Vikrant Sharma
Atul Kumar
N S Baruaole†
Mukesh Kumar

α
Alpha Science International Ltd.
Oxford, U.K.
Engineering Mechanics Statics
292 pgs. | 478 figs. | 11 tbls.

Vikrant Sharma
Atul Kumar
N S Baruaole†
Mukesh Kumar
Mechanical and Engineering Department
Mody University
Lakshmangarh

Copyright © 2018
ALPHA SCIENCE INTERNATIONAL LTD.
7200 The Quorum, Oxford Business Park North
Garsington Road, Oxford OX4 2JZ, U.K.

www.alphasci.com

ISBN 978-1-78332-356-2

E-ISBN 978-1-78332-432-3

All rights reserved. No part of this publication may be reproduced, stored in


a retrieval system, or transmitted in any form or by any means, electronic,
mechanical, photocopying, recording or otherwise, without prior written
permission of the publisher.
Sincerely Dedicated
to
The Lord Ganesha
&
to our loving parents, family and friends
In the loving memory of our Co-Author
& former Head of Department
(Late) Sh. Nitin S. Baruaole
Preface
Engineering Mechanics: Statics, deals with the behavior of bodies under the action of
forces. It is the foundation of engineering disciplines hence, necessary for the students
of engineering. The objective of this book is to develop a fundamental knowledge of
‘Statics’ and ability to analyze problems in real life with a simple and logical manner
for carrying out substantial engineering design.
Based on our classroom experiences, we have designed this textbook in a clear
and concise language. The purpose is to present the concepts and principles of
‘Statics’ in a systematic approach to help the students understand the subject. We have
invested a great deal of attention to learn the fundamental principles of statics and
the development of problem-solving skills. The book would be useful for engineering
students and the professionals as well.
Large numbers of solved problems are selected to help students understand the
intricacies of engineering problems and to analysis. Practice problems are provided
for each unit, based on principles of statics and real-life engineering problems that
are applicable to engineering designs. Each unit also lists a summary of key concepts
to facilitate further learning.
We wish to express our thanks to all colleagues and friends who have provided
valuable inputs during preparation of this text book. Our special thanks to Dr. V. K.
Jain, Dean, Mr. Satyajeet Anand and Mr. Mukul Kant Paliwal, Assistant Professors,
CET Mody University for their valuable guidance. We also thank all of our students
at CET, Mody University.
Finally, our sincere thanks to Narosa Publishing House, for taking interest and
publishing this book in the best possible form.
Vikrant Sharma
Atul Kumar
N S Baruaole†
Mukesh Kumar
Contents
Preface vii

0. Introduction 0.1—0.5
0.1 Mechanics 0.3
0.2 Basic Concept 0.3
0.3 Fundamental Laws 0.4
0.4 Units 0.5

1. Co-planer System of Forces 1.1—1.76


1.1 Introduction to Force 1.3
1.2 Classification of Force 1.4
1.2.1 System of Forces 1.4
1.3 Resultant Force 1.7
1.3.1 Resultant of Collinear Forces 1.7
1.3.2 Resultant of Concurrent Forces 1.7
1.4 Resolution of a Force 1.10
1.5 Moment 1.13
1.5.1 Resultant Moment 1.14
1.5.2 Varignon’s Theorem (Principle of Moment) 1.14
1.5.3 Resultant of Parallel Force System 1.15
1.6 Couple 1.16
1.6.1 Moment of a Couple 1.17
1.6.2 Force–Couple System 1.17
1.7 Equivalent Systems of Forces 1.18
Solved Examples Based on Parallelogram Law of Forces 1.18
Solved Examples Based on Resolution of Forces 1.26
Solved Examples Based on Co-planer Concurrent Forces 1.35
Solved Examples Based on Moment & Non-Concurrent Forces 1.43
Solved Examples Based on Parallel Forces 1.55
Solved Examples Based on Force-Couple System and Equivalent System 1.59
Summary 1.70
Problems 1.71

2. Equilibrium of System of Forces 2.1—2.38


2.1 Equilibrium 2.3
2.2 Free Body Diagram (F.B.D.) 2.3
2.2.1 General Procedure for Constructing a Free Body Diagram 2.6
2.2.2 Examples on Free Body Diagram (FBD) 2.6
2.3 Equilibrium for Different Force System (Based on Number of Forces) 2.8
x  Engineering Mechanics Statics

Solved Examples Based on Equilibrium of Forces 2.10


Summary 2.34
Problems 2.35

3. Analysis of Structure 3.1—3.49


3.1 Trusses 3.3
3.2 Types of Truss 3.3
3.3 Analysis of Truss 3.4
3.3.1 Assumptions in Truss Analysis 3.4
3.3.2 Methods of Truss Analysis 3.4
Solved Examples Based on Method of Joints 3.5
Solved Examples Based on Method of Sections 3.13
Summary 3.19
3.4 Beam 3.19
3.5 Types of Beams 3.19
3.6 Types of Loading 3.20
3.7 Shear Force (V) and Bending Moment (M) 3.21
3.7.1 Sign Convention 3.22
3.8 Shear Force and Bending Moment Diagrams 3.22
Solved Examples Based on Simply Supported Beam 3.22
Solved Examples Based on Cantilever Beam 3.34
Solved Examples Based on Overhang Beam 3.44
Summary 3.48
Problem 3.48

4. Centroid and Moment of Inertia 4.1—4.34


4.1 Introduction 4.3
4.2 Determination of Centroid 4.3
4.3 Centroid of Wire 4.5
4.4 Procedure to Determine Coordinates of Centroid of Composite Areas 4.5
4.5 Centroid of Common Areas Using Method of Integration 4.5
4.6 Centroid and Area of Some Common Plane Areas 4.7
Solved Examples Based on Centroid 4.8
Summary 4.18
Problems 4.19
4.7 Area Moment of Inertia 4.19
4.8 Perpendicular Axis Theorem 4.20
4.9 Parallel Axis Theorem 4.21
4.10 Radius of Gyration 4.22
4.11 Area Moment of Inertia of Common Areas 4.22
Solved Examples Based on Area Moment of Inertia 4.26
Summary 4.32
Problems 4.33
Contents  xi

5. Friction 5.1—5.40
5.1 Introduction 5.3
5.2 Coulomb’s Theory of Dry Friction 5.3
5.3 Coefficient of Friction 5.4
5.4 Angle of Friction 5.5
5.5 Angle of Repose 5.5
5.6 Applications of Friction 5.6
5.6.1 Ladder Friction 5.6
5.6.2 Wedge Friction 5.7
5.6.3 Belt Friction 5.7
Solved Examples Based on Block Friction 5.9
Solved Examples Based on Ladder Friction 5.20
Solved Examples Based on Wedge Friction 5.26
Solved Examples Based on Belt Friction 5.34
Summary 5.38
Problem 5.38

6. Simple Stress & Strain 6.1—6.20


6.1 Stress 6.3
6.1.1 Types of Stress 6.3
6.2 Strain 6.4
6.2.1 Shear Strain 6.5
6.2.2 Longitudinal and Lateral Strains 6.5
6.3 Poisson’s Ratio 6.6
6.4 Volumetric Strain 6.6
6.5 Hooke’s Law and Elastic Moduli 6.6
6.6 Bar of Varying Cross-Section 6.6
6.7 Stress-Strain Diagram for Mild Steel 6.7
6.8 Temperature Stress and Strain 6.8
6.9 Composite System 6.9
6.10 Factor of Safety 6.9
Solved Examples Based on Stress and Strain 6.9
Summary 6.18
Problems 6.19
Objective Type Questions OTQ.1—OTQ.10
Review Problems R.1—R.3
Index I.1–I.2
Unit 0
Introduction
0.1 Mechanics
echanics is that branch of physics which deals with the state of rest or motion of
M
bodies under the action of forces. No other subject plays a bigger role in engineering
analysis than mechanics. Though the principles of
mechanics are few, they have varied application Mechanics is the study of
in engineering. A detailed understanding of this forces and their effects.
subject is an essential prerequisite for work in
many fields of engineering.
Mechanics is divided into three parts:
1. Mechanics of Rigid Bodies
2. Mechanics of Deformable Bodies and
3. Mechanics of Fluid.
In this book, we will concentrate on mechanics of rigid bodies.
Mechanics of rigid body deal with perfectly rigid bodies. It may be classified
as follow:

Statics deals with the equilibrium of bodies under action of forces and
Dynamics deals with the motion of bodies. Dynamics is further classified as,
Kinematics and Kinetics. Kinematics is the study of motion of bodies without any
reference to the cause of motion. (i.e. without any reference to effects of forces
and masses). Kinetics is the study of bodies with reference to effects of forces
and masses.
In this book, we will concentrate on Statics.

0.2  Basic Concept


The following concepts are important to understand before studying mechanics.
1. Space: It is the boundless three-dimensional region occupied by bodies
which have relative position and direction. The position of the bodies
0.4  Engineering Mechanics Statics

can be defined by two lengths (in two-dimensional problems) or three


lengths (in three dimensional problems). These lengths are known as the
coordinates of bodies.
Time: Time provides a measure of when an event or sequence of events
2.
occurs.
Mass: Mass is the amount of matter in an object. It is a measure of inertia
3.
of a body, which is resistance of matter to a change of velocity.
Particle: A particle is a body of negligible mass and dimensions. Objects
4.
that are small compared to other objects can be idealized as particles.
Rigid body: A rigid body is non-deformable body i.e. the distance between
5.
any points in the body remains fixed.

0.3  Fundamental Laws


The study of mechanics is based on following fundamental laws:
1. Newton’s Laws of Motion: Sir Isaac Newton in seventeenth century
stated three laws of motion as follow,
Law I: A particle remains at rest, or continuous to move in a straight line
with uniform velocity, if there is no unbalanced force acting on it.
Law II: The acceleration of a particle is proportional to the resultant force
acting on the particle and is in the direction of this force.
Law III: The forces of action and reaction between interacting bodies are
equal in magnitude, opposite in direction, and collinear.
2. Newton’s Law of Gravitation: This law states that two particles of mass
m1 and m2 are mutually attracted with equal and opposite forces P and – P
of magnitude P given by the formula,
m1m2
P = G
r2
Where, m1 and m2 = masses of two particles
r = distance between two particles
G = constant of gravitation
3. Law of Transmissibility: It state that, the point of application of a force
can be transmitted to any other point along its line of action within the
body. Force P acting at point O can be transmitted to point O’ along its line
of action as shown in Figure 1.
Introduction  0.5

Fig. 1

0.4 Units
In mechanics we use four fundamental measures. These are length, mass, force,
and time. Although there are a number of different systems of units, only the two
systems are most commonly used in engineering. They are shown in Table 1.
Table 1. Fundamental Units

Fundamental Measures Symbol SI Units U.S. Customary Units


Length L Meter (m) Foot (ft)
Mass M Kilogram (kg) Slug
Force P Newton (N) Pound (lb)
Time T Second (s) Second (s)
Unit 1

In this unit, we discuss forces and effect of forces on particle and


rigid bodies in detail. Particularly, we learn how to determine
resultant force of different force systems. The moment of force and
couple and concept of equivalent system are also introduced in this
unit. Systematically understanding of these concepts is important to
the design engineers.
Co-planer System of Forces
1.1  Introduction to Force
Study of Statics and the whole study of Mechanics is actually the study about the
actions of forces or force systems and the effect of these actions on bodies. So it
is important to understand the action of forces, characteristics of force systems,
and particular methods to analyze them.
What is force? Force is an action that
Force is the action of a body about another body, has the ability to change
and it changes or tends to changes the state of rest motion
or motion of a body. Forces exist because of an
interaction of one object with another object. Whenever there is an interaction
between two objects, there is a force upon each of the objects.
Force is measured using standard metric unit known as the Newton; Newton
is abbreviated as ‘N’, so 15 N means 15 Newton of force. One Newton equals 1
kilogram multiplied by 1 meter per second squared. This means that a force of one
Newton causes a 1-kilogram mass to have an acceleration of 1 m/sec2.
1 N = (1 kg) (1 m/s2) = 1 kg-m/s2.
The force is a vector quantity as its effect
depends on the direction as well as on the Force is a vector quantity
magnitude of the action. Thus, for example 15 N
is not a full description of the force acting on an object, in contrast 15 N acting in
downward direction (or any direction) is a complete description of the force acting
on an object.
Force acting on an object may cause the object to change its shape, to start
moving, to stop moving, to accelerate or decelerate.
As shown in figure 1.1,
the effect of the force
applied to the bracket
depends on P, the angle
θ and the location of the
point of application. If
any of these specifications
will change, it alters the
effect on the bracket. So Fig. 1.1
magnitude, direction, sense
and point of application are characteristics of the force.
Magnitude represents the value of force, like, 15 N, 10 KN etc. It is also
expressed in Pounds (lbs). The Magnitude can be represented graphically by
drawing a vector to scale.
1.4  Engineering Mechanics Statics

Direction of the force is represented by line of action and angle it forms with
some reference axis. The line of action is indefinitely long line on which the force
is laying.
Sense of the force is represented by an arrowhead; it specifies the direction
in which the force moves along the line of action. The direction relates to the line
of action of the force, and the sense is the way in which the force moves along
that line.
Point of application is the exact contact point (location) at which a force is
applied to a body. Graphically it is represented by the tip of the arrowhead (it may
be situated in the opposite end as the arrowhead) and it is unique to each force. It
may happen that different forces share same point of application.

1.2 Classification of Force
Forces are classified as either contact or body forces. Contact force is produced
by direct physical contact of two objects, and they are distributed over a surface
area of the body; examples of contact forces include applied force, frictional
forces, normal forces etc. Body force is produced when body (object) is located
in a force field such as a gravitational, electric or magnetic field. These types of
forces results when the two interacting objects are not in physical contact with
each other and they are distributed over the volume of the body.
Forces are also considered as an external force or internal force. External
force is applied externally to an object; they are either applied or reactive forces.
Internal force is developed inside the body to resist deformation of body.
Forces may be further classified as concentrated or distributed force. When
the area over which contact force is applied is very small like a point, the force
may be considered as concentrated on a point. The force which is distributed over
an area is considered as distributed force.

1.2.1 System of Forces
When two or more forces act on a body they are System of forces is simply
called to form a system of forces. Most practical a particular set of forces
problems of statics involve system of force.
System of forces is simply a term used to describe a group of forces. System of
forces may be classified as follows,
Co-planer System of Forces  1.5

Coplanar forces are those forces whose lines of action lies on the same plane.
Coplanar forces may be collinear, concurrent, non-concurrent, and parallel. Non-
Coplanar forces are those forces whose lines of action does not lie on the same
plane. Non-Coplanar forces may be concurrent, non-concurrent, and parallel.
Non-coplanar forces are also known as space forces or spatial force system, as
forces passes through space when they change their plane. Table 1.1 shows types
and characteristics of different system of forces.
Table 1.1.  Characteristics of Different System of Forces

Collinear Forces are those forces whose


lines of action lies on the same line. As
shown in figure, forces P1 and P2 acting
in same plane and having a common
line of action, this system is coplanar
collinear force system. Similarly forces
P3 and P4 are collinear forces.

Coplanar Concurrent Forces are those


forces whose lines of action passes
through the same point and also lie on
the same plane. The concurrent forces
may or may not be collinear. As shown
in figure, line of action of forces P1, P2
and P3 meet at common point O and
also these forces lie on the same plane,
this system is coplanar concurrent force
system.

Coplanar Non-Concurrent Forces


are those forces whose lines of action
does not pass through the same point
but lie on the same plane. As shown in
figure, forces P1, P2, P3 and P4 lie in the
same plane but their lines of action do
not meet at same point, this system is
coplanar non-concurrent force system.
1.6  Engineering Mechanics Statics

Coplanar Parallel Forces are those


forces whose lines of action are parallel
to each other and also lie on the same
plane. As shown in figure, forces P1, P2,
and P3 lie in the same plane and their
lines of action are parallel to each other,
this system is coplanar parallel force
system. Parallel forces can be in the
same or in opposite directions. Parallel
forces which are having same direction
are known as like parallel forces
and parallel forces which are having
different directions are known as unlike
parallel forces.
Non-Coplanar Concurrent Forces are
those forces whose lines of action passes
through the same point but do not lie on
the same plane. As shown in figure, line
of action of forces P1, P2 and P3 meet at
common point O but do not lie on the
same plane, this system is non-coplanar
concurrent force system.

Non-Coplanar Non-Concurrent Forces


are those forces whose lines of action
does not pass through the same point and
also not lie on the same plane. As shown
in figure, line of action of forces P1, P2
and P3 do not pass through the same point
and also not lie on the same plane, this
system is non-coplanar non-concurrent
force system.

Non-Coplanar Parallel Forces are


those forces whose lines of action are
parallel to each other but do not lie in the
same plane. As shown in figure, forces
P1, P2, and P3 are parallel to each other
but do not lie in the same plane.
Co-planer System of Forces  1.7

1.3 Resultant Force
Resultant is a single force
When number of forces are acting on an object then
which produces the same
it is possible to replace them with a single force
effect as produced by
which produces the same effect as produced by
number of forces when
all the given forces, this single force is known as
acting together
resultant force.
Composition is the
Resultant force is denoted by R. The procedure
procedure to find out
to find out single resultant force of a system of
single resultant force of a
forces is known as composition of forces.
system of forces

1.3.1 Resultant of Collinear Forces


The resultant of two or more collinear forces is simply their algebraic sum and
acts on the same line on which lines of action of all collinear forces lie. If two
forces P1 and P2 act in same direction as shown in figure 1.2, their resultant
R = P1 + P2. If two forces P1 and P2 act in opposite direction as shown in figure 1.3,
their resultant R = P1 – P2. As a matter of convenience, force acting towards right
is considered as positive and force acting toward left is considered as negative.

Fig. 1.2 Fig. 1.3

1.3.2 Resultant of Concurrent Forces


Here we may consider two cases of concurrent forces. Case I, resultant of two
concurrent forces and Case II, resultant of more than two concurrent forces.
CASE I. Resultant of two concurrent forces:
Resultant of two concurrent forces can be found out by means of the parallelogram
law or the triangle law. These methods are discussed in following section:
Parallelogram Law of Forces (Analytical method)
It state, “If two forces acting
simultaneously at a point be
represented in magnitude and
direction by two adjacent sides of
a parallelogram, their resultant
may be represented in magnitude
and direction by the diagonal
of the parallelogram passing
through that point.”
In figure 1.4, two forces P1 and
P2 acting at point O are represented Fig. 1.4
1.8  Engineering Mechanics Statics

in magnitude and direction by OA and OB respectively. By parallelogram law the


resultant R is represented in magnitude and direction by OC.
We now find magnitude and direction of resultant as below:
Let, θ be the angle between two forces P1 and P2 and α is the angle between
force P1 and resultant R.
Now from C draw a perpendicular CD to OA produced.
Now < DAC = < AOB = θ …(Corresponding angle)
In ∆DAC, CD = P2 sin θ and AD = P2 cos θ
In ∆DOC,
OC2 = OD2 + CD2 = (OA +AD)2 + CD2
\ R2 = (P1 + P2 cos θ)2 + (P2 sin θ)2
\ R2 = P12 + 2P1P2 cos θ + P22 cos2 θ + P22 sin2 θ
\ R2 = P12 + 2P1P2 cos θ + P22 (cos2 θ + sin2 θ)

\ R = P12 + P22 + 2 P1P2 cos θ ...  Magnitude of resultant ...(1.1)


Now in ∆DOC,
CD CD
tan α = =
OD OA + AD
P2 sin θ
\ tan α = ...  Direction of resultant ...(1.2)
P1 + P2 cos θ

Particular Cases
1. If θ = 90°, i.e. when the two forces act at right angle, then
P
R = P12 + P22 and tan α = 2 (since cos 90° = 0 and sin 90° = 1)
P1
2. If the two forces are equal, i.e. P1 = P2

R = P12 + P12 + 2 P12cos θ = 2 P12 (1 + cos θ)

θ 2 θ
= 2 P12 × 2cos 2 (since 1 + cos q = 2 cos )
2 2

θ
= 4 P12 cos 2
2
θ
∴ R = 2 P1 cos
2
Triangle Law of Forces (Graphical method)
It state, “If two forces acting simultaneously at a point be represented in
magnitude and direction by two sides of triangle taken in order, their resultant
may be represented in magnitude and direction by the third side of triangle taken
in opposite order”
Co-planer System of Forces  1.9

Triangle law is the equivalent statement of parallelogram law, which is shown


in figure 1.5; this figure is one-half of the parallelograms. Here the tail of P2 is
placed at the tip of P1 and R (resultant) is the vector that completes the triangle,
drawn from the tail of P1 to tip the tip of P2. The result is identical if the tail of
P1 is placed at the tip of P2 and R is drawn from the tail of P2 to the tip of P1.

Fig. 1.5

CASE II. Resultant of more than two concurrent forces:


If three or more forces are acting at a point then their resultant is found analytically
by a method known as rectangular components method. In this method all the
forces acting at a point are resolved (resolution of force is discussed in detail in
next section) into horizontal and vertical components and then algebraic sum of
horizontal components (written as SH) and vertical components (written as SV)
found separately.
Then the resultant R is calculated as,
R= (ΣH )2 + (ΣV )2 ...  Magnitude of resultant ...(1.3)
Let θ be the angle of resultant R with x-axis, then
ΣV
tan θ = ...  Direction of resultant ...(1.4)
ΣH
Polygon Law of forces (Graphical method)
The resultant of three or more forces acting at a point is found graphically by
polygon law of forces, it state,
“If a number of forces acting simultaneously at a point be represented in
magnitude and direction by the sides of a polygon taken in order, their resultant
may be represented in magnitude and direction by the closing side of the polygon
taken in opposite order”
If four forces P1, P2, P3 and P4 are acting at a point as shown in figure 1.6,
then the resultant of these forces is obtained by drawing a polygon of forces as
explained below and shown in figure 1.7.
1.10  Engineering Mechanics Statics

Fig. 1.6 Fig. 1.7

With suitable scale from any convenient point A, draw AB parallel and equal
to P1. From B draw BC parallel and equal to P2. From C draw CD parallel and
equal to P3. From D draw DE parallel and equal to P4. Draw AE from the tail of P1
to the tip of P4, then AE represents resultant of four forces P1, P2, P3 and P4. Also
AC represents resultant of P1 and P2 and AD represents resultant of AC and CD.
ABCDEA is the force polygon and it can be extended to any number of forces.

1.4 Resolution of a Force
Any force can be resolved into individual component forces in the same way as
individual component forces can be composed Resolution is the process
together. It is often convenient to decompose of splitting up the given
a single force into two distinct forces; they are force into components,
known as components. These forces, when without changing its
acting together, have the same external effect on effect on the body
a body as the original force.
There are mainly two methods of resolving a force.
1. Perpendicular resolution
2. Non-perpendicular resolution
1. Perpendicular Resolution: Perpendicular resolution is the general
method of splitting up a single force into two mutually perpendicular
directions. Two perpendicular components are acting along x-axis and
y-axis or any two perpendicular axes.
Table 1.2 illustrates the resolution of a force into perpendicular
components.
Following sign conventions are used in drawing components,
(a) Components acting horizontally towards right are positive and
components acting horizontally towards left are negative.
(b) Components acting vertically upward are positive and components
acting vertically downwards are negative.
Co-planer System of Forces  1.11

Table 1.2
1.12  Engineering Mechanics Statics

When body is placed on an inclined plane, its weight ‘mg’ can be resolved
into two components i.e. parallel to incline plane and perpendicular
to incline plane. Here we can select x-axis along the plane and y-axis
perpendicular to plane as shown in figure 1.8.

Fig. 1.8

2. Non-Perpendicular Resolution: In non-perpendicular resolution a force


is resolved into two directions which are not perpendicular to each other.
If force P is to be resolved into two non-perpendicular directions let
say along OA and OB as shown in
figure 1.9, where OA and OB are
not perpendicular to each other.
Now construct a parallelogram
by keeping two components (i.e.
components along OA and OB) along
two adjacent sides of parallelogram
and original given force along the
diagonal of parallelogram. Fig. 1.9
Now apply sine rule in triangle OAC,
P P P
OB = OA =
sin α sin β sin[180 – (α + β)]
Co-planer System of Forces  1.13

P sin β
\  POA = ... This is component of force P along OA
sin[180 - (α + β)]
P sin α
\  POB = ... This is component of force P along OB
sin[180 - (α + β)]

1.5 Moment
The rotational effect
When a force causes an object to turn, this turning
producead by force is
effect is called moments. So the moment of a
known as moment of the
force is a measure of its tendency to cause a body
force
to rotate around a specific point or axis. Moment
is the product of the force multiplied by the perpendicular distance from the line
of action of the force to the pivot or point where the object will turn. The point
about which moment is taken is known as moment center and the perpendicular
distance is known as moment arm, as shown in figure 1.10. If moment arm d is
zero, i.e. the line of action of the force P is passing through the moment center
then moment due to this force will be zero.

Fig. 1.10

Moment = Force × Distance


M = F × d ...(1.5)
Since moment is product of force and perpendicular distance, its S.I. unit is
N-m or KN-m.
The direction of the rotation resulting from a moment is either clockwise or
anticlockwise. When force causes an object to turn in clockwise direction, it is
called clockwise moment and when force causes an object to turn in anticlockwise
direction, it is called anticlockwise moment. Anticlockwise moments are regarded
as positive, while clockwise moments are negative. (We shall assume this sign
convention throughout this book)
Anticlockwise moment → Positive

Clockwise moment → Negative


1.14  Engineering Mechanics Statics

1.5.1 Resultant Moment
When two or more than two forces are acting about a point their combined effect
is represented by the “Resultant Moment”. To find out the resultant moment about
a point, first find the sum of the anticlockwise moments and clockwise moments
about the point, than take the lesser of these two moments from the greater and the
difference is the magnitude of the resultant moment. The direction of the resultant
moment will be that of the greater of the two component moments.

1.5.2  Varignon’s Theorem (Principle of Moment)


The Varignon theorem is a theorem given by French mathematician Pierre
Varignon in 1687. It states, “The moment of resultant of a force system about any
point is equal to the algebraic sum of moments of all other forces about the same
point”.
Let R be the resultant of two concurrent forces P1 and P2. Let R, P1 and
P2 make angles θ, α and β with axis respectively as shown in figure 1.11. The
moment of R about an arbitrary point A is, R × d, where d is the perpendicular
distance from line of action of R to point A. Similarly if we consider d1 and d2 are
perpendicular distances from line of action of P1 and P2 to point A, then the sum
of moments of P1 and P2 about point A are P1 × d1 + P2 × d2.

Fig. 1.11

Now, the sum of vertical component of P1 and P2 is equal to the vertical


component of their resultant R. (As R is the resultant of P1 and P2)
i.e., Ry = P1 + P2
y y
But, from figure, Ry = R cos θ, P1 = P1 cos α and P2 = P2 cos β
y y
\ R cos θ = P1 cos α + P2 cos β
Co-planer System of Forces  1.15

Now multiply on both sides by OA


R × OA cos θ = P1 × OA cos α + P2 × OA cos β
But, from figure, OA cos θ = d, OA cos α = d1 and OA cos β = d2
\ R × d = P1 × d1 + P2 × d2 ...(1.6)
Hence, it is proved that, the moment of resultant of a force system about any
point is equal to the algebraic sum of moments of all other forces about the same
point.

1.5.3 Resultant of Parallel Force System


Case I: Like Parallel Forces
When two parallel forces are acting in same direction they are known as like
parallel forces. Their resultant can be find out using following steps, consider
figure 1.12.
1. Find resultant R = P1 + P2
2. Take the algebraic sum moment of forces about point O,
Moment of P1 about O = P1 × AO (clockwise moment)
Moment of P2 about O = P2 × BO (anticlockwise moment)
3. Position of resultant R can be obtained by applying Varignon’s theorem
about point O.
– P1 × AO + P2 × BO = R × CO
In above equation, CO is the perpendicular distance between line of action of
resultant R and reference point O.

Fig. 1.12

Case II: Unlike Parallel Forces


When two parallel forces are acting in opposite direction they are known as unlike
parallel forces. Their resultant can be find out using following steps, consider
figure 1.13.
1. Find resultant R = P1 – P2
2. Take the algebraic sum moment of forces about point O,
Moment of P1 about O = P1 × AO (clockwise moment)
Moment of P2 about O = P2 × BO (clockwise moment)
1.16  Engineering Mechanics Statics

3. Position of resultant R can be obtained by applying Varignon’s theorem


about point O.
–P1 × AO – P2 × BO = –R × CO
In above equation, CO is the perpendicular distance between line of action of
resultant R and reference point O.

Fig. 1.13

1.6 Couple
A couple consists of a pair of two forces which
Two equal, unlike parallel,
has the following properties:
non-collinear forces form
• Equal magnitude and opposite in a couple
direction
• Act along parallel lines of action
• Separated by a perpendicular distance d, known as arm of couple. A
couple is shown in figure 1.14.
Since the two forces cancels out each other giving zero resultant, a couple
produces purely rotational effect in the body without translation motion. A couple
cannot be balanced by a single force but it can be balanced only by another couple
of opposite nature.

Fig. 1.14
Co-planer System of Forces  1.17

1.6.1  Moment of a Couple


Consider a couple as shown in figure 1.15.
The moment of couple about point A is,
MA = P × d2 – P × d1
MA = P (d2 – d1)
But from figure 1.15, d2 – d1 = d
MA = P × d
The moment of a couple is the product
of magnitude of one of the forces and arm Fig. 1.15
of couple.
d
Now consider point B at distance from force P, the moment of couple
about point B is, 2
d d
MB = P ´ + P´
2 2
æ d d ÷ö
MB = P ççç + ÷÷
è 2 2ø
MB = P × d ...(1.7)
From above we see that, the moment of couple about any pivot point is P × d.
It is important to note here that a couple does not have moment center, like
moment of force. A couple has the same moment about all points on a body.

1.6.2  Force–Couple System


The concept of couple is useful in application of parallel transfer of a force.
Refer to figure 1.16, consider a force P is acting at point A and it is required
to transfer from point A to point B without changing its magnitude and direction.
Now apply two forces of equal magnitude (their magnitude are same as that
of force P acting at point A) and opposite in direction at point B, as shown in
figure 1.17.

Fig. 1.16 Fig. 1.17 Fig. 1.18

Observing figure 1.17, we see that two forces are acting in opposite direction
at point A and B form a couple. Moment of this couple is P × d, anticlockwise.
Thus, to shift a force to point B, a couple is required to be added to the system.
1.18  Engineering Mechanics Statics

This system which consisting a force and a couple at point B, as shown in


figure 1.18 is known as force-couple system. So we can transfer a force parallel
to itself at any point with force-couple system.
It is also possible to replace a force-couple system into a single force by
following exactly the reverse procedure as discuss above.

1.7 Equivalent Systems of Forces


Two force systems that produce the same external effects on a rigid body are said
to be equivalent, i.e. the sums of the forces and sums of the moments about a
point are equal. Equivalent system consists of a single resultant force R through
the given point and single resultant moment.
Two systems are equivalent only when,
1. The sum of the forces in system 1 is equal to the sum of the forces in
system 2
2. The sum of the moments about any point O in system 1 is equal to the
sum of the moments about same point O in system 2.

Solved Examples Based on Parallelogram Law of Forces


Example 1. Two forces 20 N and 25 N are acting at a point with an angle of 60°
between them. Find the magnitude and direction of resultant.
Solution: Here P1 = 20 N, P2 = 25 N and θ = 60°
As we know from parallelogram law of forces,
R = P12 + P22 + 2 P1P2 cos θ

\ R = 202 + 252 + 2 × 20 × 25cos60


R = 39.05 N   ...Magnitude of resultant  (Ans.)
Now we know that
P2 sin θ
tan α =
P1 + P2 cos θ
25sin 60
\ tan α =
20 + 25cos60
\ α = 33.66°   ...Direction of resultant  (Ans.)
Example 2. Force P1 of magnitude 10 N is acting along horizontal direction and
force P2 acting along vertical direction. The resultant of P1 and P2 has magnitude
of 20 N. Determine the magnitude of force P2 and direction of resultant.
Solution: Here P1 = 10 N, R = 20 N and θ = 90°
As we know that when θ = 90° in parallelogram then,
R = P12 + P22
Co-planer System of Forces  1.19

∴ 20 = 102 + P22
\ 202 = 102 + P22
\ P2 = 17.32 N  (Ans.)
Now we also know that when θ = 90° in parallelogram then
P
tan α = 2
P1
17.32
\ tan α =
10
\ α = 59.5°  (Ans.)
Example 3. Two forces acting at point and angle between them is 120°.
The larger force is of magnitude 80 N and the resultant of these two forces is
perpendicular to the smaller force. Find magnitude of smaller force and resultant.
Solution: Here, P1 = 80 N, θ = 120°, and angle between smaller force i.e. P2 and
resultant is 90°.
As we know from parallelogram law of forces,
P2 sin θ
tan α =
P1 + P2 cos θ
In above equation α = 30°.
P2 sin120
\ tan 30° =
80 + P2 cos120
P2 0.8660 P 0.8660
\ 0.5773 = = 2
80 + P2 (–0.50) 80 – 0.5 P2
\ 80 – 0.5P2 = 1.5P2
\ 2P2 = 80
\ P2 = 40 N  (Ans.)
Now from parallelogram law of forces,
R = P12 + P22 + 2 P1P2 cos θ

\ R = 802 + 402 + 2 × 80 × 40 cos120


\ R = 69.28 N  (Ans.)
Example 4. The maximum resultant of two forces P1 and P2 is 1000 N and
minimum magnitude is 400 N. Find values of P1 and P2.
Solution: The resultant of two forces will be maximum only when the two forces
will act in same direction as shown in Figure Ex. 4 (a) and resultant is minimum
only when the two forces will act in opposite direction as shown in Figure Ex. 4(b).
1.20  Engineering Mechanics Statics

1000 N 400 N 1000 N


O 400 N O

Fig. Ex. 4(a) Fig. Ex. 4(b)

Now from figure Ex. 4(a), angle between two forces θ = 0


From parallelogram law of forces,
R = P12 + P22 + 2 P1P2 cos θ
Substituting θ = 0
R = P12 + P22 + 2 P1P2

\ R = ( P1 + P2 ) 2
\ R = P1 + P2
\ P1 + P2 = 1000   ...(Maximum resultant) ...(Eq. 1)
Now from figure Ex. 4(b), angle between two forces θ = 180°
From parallelogram law of forces,
R = P12 + P22 + 2 P1P2 cos θ
Substituting θ = 180°
R = P12 + P22 – 2 P1P2

\ R = ( P1 – P2 ) 2
\ R = P1 – P2
\ P1 – P2 = 400   ...(Minimum resultant) ...(Eq. 2)
Solving Eq. 1 and Eq. 2, we get
P1 = 700 N and P2 = 300 N  (Ans.)
Example 5. Two forces of 375 N and 250 N are acting on a hook and passing
through point O as shown in figure Ex. 5. Determine magnitude and direction of
resultant of these two forces.

250 N 375 N

45°
30°

Fig. Ex. 5
Co-planer System of Forces  1.21

Solution: Here, P1 = 375 N, P2 = 250 N and θ = 105°.


As we know from parallelogram law of forces,
R = P12 + P22 + 2 P1P2 cos θ

\ R = 3752 + 2502 + 2 × 375× 250 cos105°


\ R = 393.18 N  ...Magnitude of resultant  (Ans.)
Now, as we know from parallelogram law of forces,
P2 sin θ
tan α = R 375 N
P1 + P2 cos θ
250 N 105°
250 sin105°
\ tan α =
375 + 250 cos105°
\ α = 37.9°  ...Direction of resultant  (Ans.)
Example 6. Find the magnitude of the two forces, such that if they act at right
angles, their resultant is 5 N but when they act at 60°, their resultant is 37 N.
Solution: First consider two forces P1 and P2 acting at right angle,
\ angle between two forces θ = 90°.
As we know that when θ = 90° in parallelogram then
R = P12 + P2 2
\ 5 = P12 + P2 2
\ 25 = P12 + P22 ...(Eq. 1)
Similarly, when the angle between the two forces is 60°, then resultant force
from parallelogram law is,
R = P12 + P22 + 2 P1P2 cos θ

\ 37 = P12 + P22 + 2 P1P2 cos 60°


\ 37 = P12 + P22 + 2P1P2 × 0.5
Substituting P12 + P22 = 25 from Eq. 1
\ 37 = 25 + 2P1P2 × 0.5
\ P1P2 = 37 – 25   \ P1P2 = 12 ...(Eq. 2)
Now we know that,
(P1 + P2)2 = P12 + P22 + 2P1P2
Substituting values from Eq. 1 and Eq. 2
(P1 + P2)2 = 25 + 2 × 12 = 49
\ P1 + P2 = 49 = 7 ...(Eq. 3)
1.22  Engineering Mechanics Statics

Similarly, we know that,


(P1 – P2)2 = P12 + P22 – 2P1P2
Substituting values from Eq. 1 and Eq. 2
(P1 – P2)2 = 25 – 2 × 12 = 1
\ P1 – P2 = 1 = 1 ... (Eq. 4)
Solving Eq. 3 and Eq. 4 we get,
P1 = 4 N and P2 = 3 N  (Ans.)
Example 7. A disabled automobile is pulled using ropes subjected to the two
forces as shown in figure Ex. 7. Determine the magnitude and direction of their
resultant.
8 kN

30°
25°

4 kN
Fig. Ex. 7
Solution: Here, P1= 4 kN, P2 = 8 kN and θ = 55°
8 KN

R 55°

4 KN
As we know from parallelogram law of forces,
R = P12 + P22 + 2 P1P2 cos θ
\ R = 42 + 82 + 2 × 4 × 8cos55°
\ R = 10.80 kN   ...Magnitude of resultant  (Ans.)
Similarly we know from parallelogram law of forces,
P2 sin θ
tan α =
P1 + P2 cos θ
8sin 55°
tan α =
8 + 4cos55°
α = 32.49°  ...Direction of resultant  (Ans.)
Co-planer System of Forces  1.23

Example 8. Two forces of 240 N and 200 N are acting at point O as shown in
figure Ex. 8. Determine magnitude of resultant and direction of resultant from
positive x-axis.
Y
200 N

240 N
29°

21°
O X

Fig. Ex. 8

Solution: Here, P1 = 240 N, P2 = 200 N and θ = 90° – 21° + 29° = 98°.


As we know from parallelogram law of forces,
R = P12 + P22 + 2 P1P2 cos θ

\ R = 2402 + 2002 + 2 × 240 × 200 cos98°


\ R = 290.24 N  ...Magnitude of resultant  (Ans.)

200 N
R

240 N
98°

Now, as we know from parallelogram law of forces,


P2 sin θ
tan α =
P1 + P2 cos θ
200sin 98°
\ tan α =
240 + 200cos98°
\ α = 43.02°
Now direction of resultant from positive x axis is
21° + 43.02° = 64.02°  ...Direction of resultant  (Ans.)
Example 9. Two forces of 600 N and 700 N are acting at point O as shown in
figure Ex. 9. Determine magnitude of resultant and direction of resultant from
x-axis.
1.24  Engineering Mechanics Statics

600 N

40°

60°

700 N

Fig. Ex. 9
Solution: Here, P1 = 700 N, P2 = 600 N and θ = 100°.
As we know from parallelogram law of forces,
700 N
R = P12 + P22 + 2 P1P2 cos θ

\ R = 7002 + 6002 + 2 × 700 × 600 cos100°


\ R = 839.12 N
...Magnitude of resultant  (Ans.) R
Now, as we know from parallelogram law of
forces, 100°

P2 sin θ
tan α =
P1 + P2 cos θ
600 sin 100° 600 N
\ tan α =
700 + 600 cos100°
\ α = 44.76°
Now direction of resultant from x axis is
60° – 44.76° = 15.24°  (Ans.)
Example 10. Two forces of 600 N and 500 N are acting at point O as shown in
figure Ex. 10. Determine magnitude of resultant and direction of resultant from x axis.

Fig. Ex. 10
Co-planer System of Forces  1.25

4 2
Solution: Here, P1 = 600 N, P2 = 500 N and θ = tan –1 – tan –1 = 54.16°
1 5
As we know from parallelogram law of forces,
R = P12 + P22 + 2 P1P2 cos θ
\ R = 6002 + 5002 + 2 × 600 × 500 cos 54.16°
\ R = 980.47 N   ...Magnitude of resultant  (Ans.)
Now, as we know from parallelogram law of forces,
P2 sin θ
tan α =
P1 + P2 cos θ
500 sin 54.16° 500 N
\ tan α =
600 + 500 cos 54.16°
R
\ α = 24.41°
Now direction of resultant from x-axis is 600 N
–1 2
24.41° + tan = 46.22°  (Ans.)
5
Example 11. Two forces of 82.5 kN and 74 kN are acting on a bracket as shown
in figure Ex. 11. Determine magnitude of resultant and direction of resultant from
x-axis.

Fig. Ex. 11

Solution: Here, P1 = 82.5 kN, P2 = 74 kN


65 70
and θ = 180° – tan –1 – tan –1 = 121.30° .
120 120
1.26  Engineering Mechanics Statics

As we know from parallelogram law of forces,


R = P12 + P22 + 2 P1P2 cos θ

\ R = 82.52 + 742 + 2 × 82.5× 74 cos121.30°


\ R = 77.06 kN  ...Magnitude of resultant  (Ans.)
Now, as we know from parallelogram law of forces,
82.5 N
P2 sin θ
tan α =
P1 + P2 cos θ
74 sin 121.30°
\ tan α =
82.5 + 74 cos121.30° R

\ α = 55.13°
Now direction of resultant from x-axis is
65
55.13° + tan –1 = 83.6°  (Ans.)
120
74 N

Solved Examples Based on Resolution of Forces


Example 1. Resolve 100 N force into components along (a) x and y directions
and (b) u and v directions. Refer figure Ex. 1 Y
u
Solution: (a) Components along x and y:
Component along x = 100 cos 40° = 76.60 N  (Ans.) 100 N
Component along y = 100 sin 40° = 64.28 N  (Ans.)
30°
(b) Component along u and v:
40°
As angle between u and v is 100°, so this is non- X
30°
perpendicular resolution.
Construct parallelogram by considering 100 N force v
as a resultant and its component along u and v as shown Fig. Ex. 1
in figure Ex. 1(a)
B

C
Pu

30°

30° 100N

O 70°

80°
Pv
A
Fig. Ex. 1(a)
Co-planer System of Forces  1.27

Now in triangle OAC, apply sine rule


100 Pu Pv
= =
sin80° sin 70° sin 30°
100 sin 70°
Pu = = 95.42 N   (Ans)
sin 80°
100 sin 30°
Pv = = 50.77 N   (Ans)
sin 80°

Example 2. If the force P as shown in figure Ex. 2 is resolved into components


parallel to the bars AB and BC, the magnitude of the component parallel to bar BC
is 4 kN. What is the magnitudes of P and its component along AB?
P

1.5m
A  C


1m 4m

Fig. Ex. 2

Solution: Here taking a force P as a resultant, construct a parallelogram


taking two side of parallelogram as two component of force P along AB and BC
as shown in figure Ex. 2(a).
Now, from figure Ex. 2
1.5
tan α = \ α = 56.31°
1.0
1.5
tan β = \ β = 20.56°
4.0
From figure Ex. 2(a)
Fig. Ex. 2(a)
θ = 90 – α \ θ = 90 – 56.31° = 33.69°
Φ = 90 – β \ Φ = 90 – 20.56° = 69.54°
Now apply sine rule to figure Ex. 2(a)
P P P
= AB = BC
sin (180 – θ – Φ ) sin Φ sin θ
1.28  Engineering Mechanics Statics

P 4
=
sin (180 – 33.69 - 69.54) sin 33.69
4sin 76.87
\ P = \ P = 7 kN  (Ans)
sin 33.69
PAB 4
=
sin 69.54 sin 33.69
4sin 69.54
\ PAB = \ PAB = 6.75 kN  (Ans)
sin 33.69
Example 3. A force P is inclined at 60° to the horizontal. If the horizontal
component of force is 50 N find out vertical component of force.
Solution: Here the force P is inclined to the horizontal at an angle θ = 60°.
Horizontal component of force P = P cos θ
50
P cos 60° = 50  \ P = = 100 N
cos60°
Now vertical component of P = P sin θ = 100 sin 60° = 86.60 N  (Ans.)
Example 4. A box is to be moved in the direction of OO’ as shown in figure
Ex.4. It is identified that a 20 kN force in this direction is required but obstruction
prevent direct application of such a force. Accordingly, the forces P1 and P2 are
applied as shown. Determine the magnitude of P1 and P2.
Solution: Here it is desired that 20 kN force in the
direction of OO’ is the resultant of P1 and P2.
Now the parallelogram may be constructed
taking a 20 kN force as a resultant and by drawing
lines parallel to the specified direction of P1 and P2 as
shown in figure Ex. 4(a).
Now in triangle OAC, apply sine rule Fig. Ex. 4
Figure Ex. 4(a)
P1
20 P1 P2
= = 20 kN
sin 45° sin 60° sin 75° C
20sin 60° 75°
\ P1 = = 24.5 kN  (Ans)
sin 45° 45°
60°
O
20 sin 75°
\ P2 = = 27.32 kN   (Ans) A
P2
sin 45° Fig. Ex. 4(a)

Example 5. Crank ABC is subjected to force P as shown in figure Ex. 5.


Knowing that P must have a 200 N component perpendicular to arm AB of the
crank, determine the magnitude of force P and its component along AB.
Co-planer System of Forces  1.29

35°

40° A

B C

Fig. Ex. 5

Solution: Here component of force P along AB PAB


and perpendicular to AB are as shown in figure
Ex. 5(a). 75°
Now we know that component of force P
perpendicular to AB is 200 N
\ P sin 75° = 200 N 20
0N
P
200
\ P = = 207 N   (Ans)
sin 75° Fig. Ex. 5(a)
Now component of P along AB = P cos 75°
= 207 cos 75° = 53.58 N (Ans.)
Example 6. A vertical force of 50 N acts downward at A as shown in figure
Ex. 6. Determine the magnitudes of the two components of 50 N force along AB
and AC.

Fig. Ex. 6

Solution: Here taking a 50 N force as a resultant, construct a parallelogram taking


two adjacent sides of parallelogram as two component of force P along AB and
AC as shown in figure Ex. 6(a).
Now in triangle OAC, apply sine rule
50 PAB P
= = AC
sin 75° sin 60° sin 45°
1.30  Engineering Mechanics Statics

50 sin 60° O
\ PAB = = 44.82 N   (Ans) 30° 45°
sin 75° PAC
45°
PAB
50 sin 45°
\ PAC = = 36.6 N   (Ans) 50N 75°
A
sin 75°
60°

Example 7. Resolve 800 N force into C


components along u and v directions as shown Fig. Ex. 6(a)
in figure Ex. 7.

Fig. Ex. 7

Solution: Construct parallelogram by considering 800 N force as a resultant and


its component along u and v as shown in figure Ex. 7(a). Pu

Now in triangle OAC, apply sine rule


800 Pu Pv
= = O
sin 75° sin 75° sin 30° 75°
C
800N 30°
800 sin 75°
\ Pu = = 800 N   (Ans) Pv 75°
sin 75°
800 sin 30° A
\ Pv = = 414.11 N   (Ans)
sin 75° Fig. Ex. 7(a)
Example 8. Two forces P1 and P2 are applied to a hook as shown in figure
Ex. 8. The resultant of the two forces has a magnitude of 300 N and makes an angle
of 30° with positive x-axis (anticlockwise direction). Determine the magnitudes
of P1 and P2.
P2 y P1
45° 30°

Fig. Ex. 8
Co-planer System of Forces  1.31

Solution: Here taking a 300 N force as a resultant, construct a parallelogram


taking P1 and P2 as two adjacent sides of parallelogram as shown in figure
Ex. 8(a).
Now in triangle OAC, apply sine rule
300 P1 P2
= =
sin 75° sin 75° sin 30°
300 sin 75°
\ P1 = = 300 N   (Ans)
sin 75°
300 sin 30°
\ P2 = = 155.3 N   (Ans) Fig. Ex. 8(a)
sin 75°
Example 9. A bracket is subjected to two forces P1 and P2 as shown in figure
Ex. 9. If the resultant of the two forces has a magnitude of 525 N and direction as
shown in the figure, determine magnitudes of P1 and P2.
P1
525N

2 2 P2
1 1
3 4

Fig. Ex. 9

Solution: Here first consider angle of P2, P1 and resultant force are respectively
θ1, θ2, and θ3 from x-axis.
1 2 2
Now θ1 = tan –1 = 14.03° θ2 = tan –1 = 63.43° θ3 = tan –1 = 33.70°
4 1 3

49.4° A

C
47.73° P1

525 N 82.87°
O
P2

Fig. Ex. 9(a)

Now taking a 525 N force as a resultant, construct a parallelogram taking P1


and P2 as two adjacent sides of parallelogram as shown in figure Ex.9 (a).
1.32  Engineering Mechanics Statics

Now in triangle OAC, apply sine rule


525 P1 P2
= =
sin 49.4° sin 47.73° sin 82.87°
525 sin 82.87°
\ P2 = = 686.11 N
sin 49.4°
525 sin 47.73°
\ P1 = = 511.66 N
sin 49.4°
Example 10. A frame is subjected to a horizontal force of 200 N as shown
in figure Ex. 10. If the component of force along AC is 250 N, directed from
A towards C, determine the component of force along member AB and angle
θ (00 ≤ θ ≤ 900) of member AB.

Fig. Ex. 10

Solution: Here taking a force 200 N as a resultant, construct a parallelogram


taking two adjacent sides of parallelogram as two component of force along AC
and AB as shown in Figure Ex. 10(a) PAB
Now in triangle OAC, apply sine rule B
40°
200 250 PAB A 
= = O
200N
sin 40° sin θ sin (140 – θ)°
PAC (140–)
C
250 sin 40°
\ sin θ = = 0.8034 Fig. Ex. 10(a)
200
\ θ = 53.46°  (Ans.)
250 sin (140 - 53.46)°
\ PAB = = 310.6 N   (Ans.)
sin 53.46°

Example 11. A wooden log is to be hoisted using two chains as shown in figure
Ex. 11. If the resultant force is to be 1200 N in vertical upward direction determine
the magnitudes of P1 and P2 acting on each chain and angle θ of P2 so that
magnitude of P2 is to be minimum.
Co-planer System of Forces  1.33

P2 P1
30°


Fig. Ex. 11
Solution: Here taking a force 1200 N as a resultant, construct a parallelogram
taking two adjacent sides of parallelogram as two component of force along AC
and AB as shown in figure Ex.11 (a).
Now in triangle OAC, apply sine rule 1200N
C
1200 P P2
= 1 =  (150–)
sin (150 – θ)° sin θ sin 30° A
P1
1200 sin 30°
\ P2 = P2 30°
sin (150 – θ)°
Now, for P2 to be minimum the denominator O
sin (150 – θ) must be maximum. Fig. Ex. 11(a)
\ (150 – θ) = 90°  \ θ = 60°
1200 sin 30°
\ P2 = = 600 N   (Ans.)
sin (150 – 60)°
P2 sin 60° 600 sin 60°
Now P1 = = = 1039.23 N   (Ans.)
sin 30° sin 30°
Example 12. The line of action of the 1000 N force is passing through the
point A (–6, –2) and B (7,5) as shown in Figure Ex. 12. Determine the x and y
components of 1000 N force.
Y
1000N
B(7,5)

A(–6,–2)

Fig. Ex. 12

–1 7
Solution: Here angle made by 1000 N force with x axis is θ = tan \ θ = 28.3°
13
Now x component of 1000 N force = 1000 cos 28.3° = 880.48 N  (Ans.)
y component of 1000 N force = 1000 sin 28.3° = 474 N  (Ans.)
1.34  Engineering Mechanics Statics

Example 13. Resolve 500 N force acting on bar into components along OA and
perpendicular to OA. Refer figure Ex. 13.
A
20°

500N

30°
O
Fig. Ex. 13

Solution: The component of 500 N force along OA and perpendicular to OA are,


Component along OA = 500 cos 50° = 321.4 N  (Ans.)
Component perpendicular to OA = 500 sin 50° = 383 N  (Ans.)
Example 14. A frame is subjected to two forces as shown in figure Ex. 14 .
Replace them by equivalent forces along x and a direction.

400N
105°

   450N
Fig. Ex. 14          Fig. Ex. 14(a)

Solution: Here first find out resultant of given two forces using parallelogram
law. Refer figure Ex. 14 (a)
Now take P1 = 450 N, P2 = 400 N and angle θ between P1 and P2 = 105°
As we know from parallelogram law of forces,
R = P12 + P22 + 2 P1P2 cos θ

\ R = 4502 + 4002 + 2 × 450 × 400 cos105°


\ R = 519 N
Now, as we know from parallelogram law of forces,
P2 sin θ
tan α =
P1 + P2 cos θ
400 sin105°
\ tan α = \ α = 48.11°
450 + 400 cos105°
Co-planer System of Forces  1.35

Now taking a 519 N force as a resultant, construct a parallelogram taking two


adjacent sides of parallelogram as two component of force along x and a direction
as shown in figure Ex. 14(b).
PX O
A
45°113.1°

21.9°
PA

C
519N
Fig. Ex.14 (b)

Now in triangle OAC, apply sine rule


519 Pa Px
= =
sin 45° sin113.1° sin 21.9°
519 sin113.1°
\ Pa = = 675.13 N   (Ans.)
sin 45°
519 sin 21.9°
\ Px = = 273.76 N   (Ans.)
sin 45°

Solved Examples Based on Co-planer Concurrent Forces


Example 1. Determine resultant of the force system shown in the figure Ex. 1

Fig. Ex. 1

Solution: Resolving the forces along x and y-axes, and then determine algebraic
sum of horizontal components and vertical components.
+
→ΣH = 5 cos 30° + 10 cos 60° + 12 cos 40°
= 18.52 kN
1.36  Engineering Mechanics Statics

+ ↑ ΣV = 5 sin 30° + 10 sin 60° –12 sin 40°


= 3.45 kN
Now we know that resultant R is calculated as,
R = ( ΣH ) 2 + ( Σ V ) 2

\ R = (18.52) 2 + (3.45) 2
\ R = 18.84 kN  (Ans.)
Let θ be the angle of resultant R with x-axis, then
ΣV
tan θ =
ΣH
3.45
\ tan θ = θ = 10.55°  (Ans.)
18.52
Example 2. A hook is subjected to three forces as shown in the figure Ex. 2.
Determine the magnitude of the resultant force and its direction from x-axis.
200N

2
500N 1 300N

1 1
2 2

Fig. Ex. 2

Solution: Consider θ1, θ2 and θ3 are the angles of forces 300 N, 200 N and 500
N respectively.
–1 1
Now, θ1 = tan = 26.57°
2
–1 2
θ2 = tan = 63.43°
1
–1 1
θ3 = tan = 26.57°
2
Now resolving the forces along x and y-axes, and then determine algebraic
sum of horizontal components and vertical components.
+
→ΣH = 300 cos 26.57° – 200 cos 63.43° – 500 cos 26.57°
Co-planer System of Forces  1.37

\ SH = – 268.33 N
(here ‘–’ sign indicate that SH is acting toward negative x direction)
+↑SV = 300 sin 26.57° + 200 sin 63.43° + 500 sin 26.57°
\ SV = 536.71 N
Now we know that resultant R is calculated as,
R = ( ΣH ) 2 + ( Σ V ) 2
\ R = (268.33) 2 + (536.71) 2
\ R = 600 N
Let θ be the angle of resultant R with x-axis, then
ΣV
tan θ =
ΣH
536.71
\ tan θ = \ q = 63.43°  (Ans.)
268.33
Example 3. Determine resultant of the force system shown in the figure Ex. 3.

Fig. Ex. 3

Solution: Resolving the forces along x and y-axes, and then determine algebraic
sum of horizontal components and vertical components.
+
→ΣH = –300 sin 20° + 350 cos 20° + 250 cos 30° – 600 sin 30°
= 142.79 N
+↑SV = –300 cos 20° + 350 sin 20° – 250 sin 30° + 600 cos 30°
= 232.41 N
Now we know that resultant R is calculated as,
R = ( ΣH ) 2 + ( Σ V ) 2
\ R = (142.79) 2 + (232.41)2
\ R = 272.77 N  (Ans.)
1.38  Engineering Mechanics Statics

Let θ be the angle of resultant R with x-axis, then


ΣV
tan θ =
ΣH
232.41
\ tan θ = \θ = 58.43°  (Ans.)
142.79
Example 4. The following forces act at a point: 30 N towards East, 20 N towards
North, 35 N towards North West, 25 N inclines at 40° towards South of West,
40 N towards South. Determine the magnitude of the resultant force and its
direction from East.

Fig. Ex. 4

Solution: The system of given forces is shown in figure Ex. 4.


Resolving the forces along x and y-axes, and then determine algebraic sum of
horizontal components and vertical components:
+
→ΣH = 30 – 35 cos 45° – 25 cos 40°
= –13.90 N (here ‘–’ sign indicate that SH is acting toward negative x direction
i.e. towards West)
+↑SV = 20 + 35 sin 45° – 25 sin 40° – 40
= –11.32 N (here ‘–’ sign indicate that SV is acting toward negative y direction
i.e. towards South)
Now we know that resultant R is calculated as,
R = ( ΣH ) 2 + ( Σ V ) 2

\ R = (13.90) 2 + (11.32) 2
\ R = 17.92 N  (Ans.)
Let θ be the angle of resultant R with x-axis, then
ΣV
tan θ =
ΣH
Co-planer System of Forces  1.39

11.32
\ tan θ = \ θ = 39.15°
13.90
Now angle of resultant with East = 180° + 39.15° = 219.15°  (Ans.)
Example 5. Three forces act as shown in figure Ex. 5. Determine the magnitude
of the force P if the resultant is along x-axis. Also determine magnitude of resultant.
Y
75N

45N
25°
25°
O X
35°

P
Fig. Ex. 5

Solution: As resultant is acting along x-axis, its vertical component become zero
i.e. SV = 0.
\ Resolving the forces along y-axes, and then equating sum of vertical
components to zero,
\ 75 sin 50° + 45 sin 25° – P sin 35° = 0
\ P = 133.32 N  (Ans.)
Now as resultant is acting along x-axis, its magnitude is equal to SH
R = SH = 75 cos 50° + 45 cos 25° + 133.32 cos 35°
\ R = 198.20 N  (Ans.)
Example 6. Three forces are acting on a bracket as shown in figure Ex. 6.
Determine the magnitude and direction of resultant force.
200N 150N

3
4 30°
200mm

400mm

100N

Fig. Ex. 6

Solution: Consider θ1, and θ2 are the angles of forces 100 N, and 200 N respectively
from horizontal.
1.40  Engineering Mechanics Statics

–1 400
Now θ1 = tan = 63.43°
200
–1 3
θ2 = tan = 36.86°
4
Resolving the forces along x and y-axes, and then determine algebraic sum of
horizontal components and vertical components:
+
→ΣH = 100 cos 63.46° + 150 cos 30° – 200 cos 36.86°
= 14.56 N
+↑SV = –100 sin 63.43° + 150 sin 30° + 200 sin 36.86°
= 105.53 N
Now we know that resultant R is calculated as,
R = ( ΣH ) 2 + ( Σ V ) 2

\ R = (14.56) 2 + (105.53) 2
\ R = 106.52 N  (Ans.)
Let θ be the angle of resultant R with x-axis, then
ΣV
tan θ =
ΣH
105.53
\ tan θ =   \ q = 82.14°  (Ans.)
14.56
Example 7. Four concurrent forces acts on the centre of mass during landing of
airplane as shown in figure Ex. 7. Determine the magnitude of the resultant force
and its direction from x-axis
2kN 8.9kN
45°

4.5 kN

30°
8.9 kN

Fig. Ex. 7

Solution: Resolving the forces along x and y-axes, and then determine algebraic
sum of horizontal components and vertical components.
+
→ΣH = 2 sin 45° – 4.5 – 8.9 sin 30°
Co-planer System of Forces  1.41

= –7.53 kN (here ‘–’ sign indicate that SH is acting toward negative x direction)
+↑SV = –2 cos 45° – 8.9 + 8.9 cos 30°
= –2.60 kN (here ‘–’ sign indicate that SV is acting toward negative y direction)
Now we know that resultant R is calculated as,
R = ( ΣH ) 2 + ( Σ V ) 2

\ R = (7.53) 2 + (2.60) 2
\ R = 7.97 kN  (Ans.)
Let θ be the angle of resultant R with x-axis, then
ΣV
tan θ =
ΣH
2.60
\ tan θ = \ θ = 19°  (Ans.)
7.53
Example 8. Bracket AB is supported by cable BC as shown in figure Ex. 8. If
the tension in cable BC is 150 N, determine magnitude of the resultant of the three
forces and direction of resultant from x-axis.
C

T = 150N

45° B
A
40°
40°

100N 125N
Fig. Ex. 8

Solution: Resolving the forces along x and y-axes, and then determine algebraic
sum of horizontal components and vertical components:
+
→ΣH = –150 cos 45° – 100 sin 40° + 125 cos 40°
= –74.58 N (here ‘–’ sign indicate that SH is acting toward negative x direction)
+↑SV = 150 sin 45° – 100 cos 40° – 125 sin 40°
= –50.88 N (here ‘–’ sign indicate that SV is acting toward negative y direction)
Now we know that resultant R is calculated as,
R = ( ΣH ) 2 + ( Σ V ) 2

\ R = (74.58) 2 + (50.88) 2
1.42  Engineering Mechanics Statics

\ R = 90.28 N (Ans.)
Let θ be the angle of resultant R with x-axis, then
ΣV
tan θ =
ΣH
50.88
\ tan θ = \ θ = 34.30°  (Ans.)
74.58
Example 9. Three forces are acting on a hook as shown in figure Ex. 9. If
the magnitude of the resultant force is 400 N and acting along positive y-axis,
determine the magnitude of force P and the angle α.

Fig. Ex. 9

Solution: Consider θ is the angle of 200 N force with x axis,


–1 3
Now θ = tan = 36.86°
4
As resultant is acting along y-axis, its horizontal component become zero i.e.
SH = 0.
\ Resolving the forces along x-axes, and then equating sum of horizontal
components to zero,
\ – P sin α + 250 cos 30° + 200 cos 36.86° = 0
\ P sin α = 376.52 (Eq. 1)
Now resultant is acting along y axis, its magnitude is equal to SV
\ 400 = SV
\ 400 = P cos α + 250 sin 30° – 200 sin 36.86°
P cos α = 394.98 (Eq. 2)
Solving Eq. 1 and Eq. 2,
\ a = 43.62°  (Ans.)
Now from (Eq. 1), P sin 43.63° = 376.52 P = 545.68 N  (Ans.)
Example 10. Three forces are acting on a bracket as shown in figure Ex. 10. If
the magnitude of the resultant force is to be 250 N directed along positive u axis,
determine force P and angle α.
Co-planer System of Forces  1.43

y P

 u
30°
x
65° 100N

225N
Fig. Ex. 10

Solution: Here find SH and equate with horizontal component of the resultant
force,
+
\ →ΣH = 250 cos 30°
\ P sin α + 100 + 225 cos 65° = 250 cos 30°
\ P sin α = 21.41 (Eq.1)
Now find SV and equate with vertical component of the resultant force,
\ +↑SV = 250 sin 30°
\ P cos α – 225 sin 65° = 250 sin 30°
\ P cos α = 328.91 (Eq.2)
Solving Eq. 1 and Eq. 2,
\ α = 3.72°  (Ans.)
Now from (Eq. 1), P sin 3.72° = 21.41 P = 330 N  (Ans.)

Solved Examples Based on Moment & Non-Concurrent Forces


Example 1. Determine the moment of the 100 N force about point O as shown
in the figure Ex. 1.
100N
1.5m

Fig. Ex. 1
Solution: Since the perpendicular distance d from the force to the point O is
1.5 m, we use
MO = P × d
Treating anticlockwise moment as positive
MO = –100 × 1.5
(Negative sign is used from our assumed sign convention of moment, as
100 N force tends to rotate the object in clockwise direction)
1.44  Engineering Mechanics Statics

\ MO = – 150 N.m
MO = 150 N.m (Clockwise)  (Ans.)
Example 2. Determine the moment of the 100 N force about point O as shown
in the figure Ex. 2
100N
1.5m

60°
0

Fig. Ex. 2

Solution: Here first determine perpendicular distance from line of action of 100
N force to point O as shown in figure Ex. 2(a).
MO = P × d
\ MO = –100 × 1.5 sin 60°
100N
1.5m

60°
O
60°
d

Fig. Ex. 2(a)

MO = –129.9 N.m = 129.9 N.m (clockwise)


Alternate Approach: Many times it is convenient to resolve the given force
into its components and then take algebraic sum of moment of these components
about given point. In above example given 100 N force can be resolve into
horizontal and vertical components as shown in figure Ex. 2(b).

Fig. Ex. 2(b)

Treating anticlockwise moment as positive, sum of moment due to these


components about point O is find out as below. (Note that, the line of action of
horizontal component is passing through point O, so it produces zero moment)
\ MO = –100 sin 60° × 1.5
MO = –129.9 N.m = 129.9 N.m (clockwise)  (Ans.)
Co-planer System of Forces  1.45

Example 3. Determine the moment of the 100 N force about point O as shown
in the figure Ex. 3.
200 N
5
3
4

1m
O
3m

Fig. Ex. 3

200 N 200 sin36.86°


5
3
4
200 cos 36.86°
1m
O
3m

Fig. Ex. 3(a)

Solution: Here resolved 200 N force into horizontal and vertical components as
shown in figure Ex. 3(a).
Treating anticlockwise moment as positive, sum of moment due to these
components about point O is,
MO = –200 cos 36.86° × 1 – 200 sin 36.86° × 3
\ MO = –520 N.m = 520 N.m (clockwise)  (Ans.)
Example 4. Three forces are acting in a plane as shown in figure Ex. 4. Determine
moment of each forces about point A and B.
B
40
50N 70N
30

A
20
100N

10

0 10 20 30 40
Fig. Ex. 4

Solution: Treating anticlockwise moment as positive, moment of 50 N force


about point A and B are,
1.46  Engineering Mechanics Statics

MA= 50 × 10 = 500 N.mm (anticlockwise)  (Ans.)


MB= –50 × 30 = –1500 N.mm = 1500 N.mm (clockwise)  (Ans.)
Moment of 70 N force about point A and B are,
MA = –70 × 10 = –700 N.mm = 700 N.mm (clockwise)  (Ans.)
MB = 70 × 10 = 700 N.mm (anticlockwise)  (Ans.)
Now resolved 100 N force into horizontal and vertical components and then
find out moment due to these components about point A and B as below,
Horizontal component of 100 N force = 100 cos 45° = 70.71 N
Vertical component of 100 N force = 100 sin 45° = 70.71 N
MA = (70.71 × 20) + (70.71 × 20) = 2828.4 N.mm (anticlockwise)  (Ans.)
MB = (70.71 × 40) – (70.71 × 20) = 1414.2 N.mm (anticlockwise)  (Ans.)
Example 5. A bracket is subjected to two forces as shown in figure Ex. 5.
Determine (a) moment of 160 N force about point A, (b) moment of 210 N force
about point A and B.

Fig. Ex. 5
Solution: Treating anticlockwise moment as positive, moment of 160 N force
about point A is,
MA = –160 × 0.7 = –112 N.m = 112 N.m (clockwise)  (Ans.)
Moment of 210 N force about point A is,
MA = –210 × 0.6 = –126 N.m = 126 N.m (clockwise)  (Ans.)
MB = 210 × 0.4 = 84 N.m (anticlockwise)  (Ans.)
Example 6. Force P is acting in a plane as shown in figure Ex. 6. If the magnitude
of the moment due to the force P about O is 300 N-m, determine magnitude of
force P.
Y
P

50°
(5,1)

O X

Fig. Ex. 6
Co-planer System of Forces  1.47

Solution: Here first find out horizontal and vertical components of force P as
shown in figure Ex. 6(a).
Y
P P sin 50°

50°
(5,1)
P cos 50°
O X

Fig. Ex. 6(a)

Treating anticlockwise moment as positive, sum of moment due to these


components about point O is,
MO = (P sin 50° × 5) + (P cos 50° × 1)
Now it is given that moment due to the force P about O is 300 N-m
\ 300 = (P sin 50° × 5) + (P cos 50° × 1)
\ 300 = P [(0.7660 × 5) + ( 0.6427 × 1)]
\ P = 67 N  (Ans.)
Example 7. Two equal forces of 150 N are applied as shown in figure Ex. 7. If
the length of the bar AB is 500 mm and radius of pulley is 100 mm, determine sum
of the moments of the forces (a) about point A and (b) about point B.

Fig. Ex. 7

Solution: Treating anticlockwise moment as positive, sum of moment due to


these forces about point A is,
SMA = (150 × 100) – (150 × 100)
\ MA = 0  (Ans.)
Now sum of moment about B is
SMB = (150 × 100) – (150 cos 35°) (500 sin 45° + 100 cos 35°) – (150 sin
35°) (500 cos 45° + 100 sin 35°)
\ MB = – 73860.57 N.mm = –73.86 N.m = 73.86 N.m (clockwise)  (Ans.)
1.48  Engineering Mechanics Statics

Example 8. A 200 N force is acting on a bracket as shown in figure Ex. 8.


Determine the value of θ for which the moment about O is (a) zero and (b) a
maximum.
3m

A


5m
2000N

Fig. Ex. 8

Solution: For moment about O to be zero, the line of action of


the given force must pass through point O as shown in figure
Ex. 8 (a).
–1 5
\ θ = tan = 59°   (Ans.)
3
Now for moment about O to be a maximum, the line of
action of the given force must perpendicular to distance OA as Fig. Ex. 8(a)
shown in figure Ex. 8 (b).
 –1 5 
\ q =  tan  + (90°) = 149°   (Ans.)
 3

Example 9. Two forces acting on a bracket as shown in


figure Ex. 9. Determine the sum of moment of these two
forces about point O.
400N
600N Fig. Ex. 8(b)

60°

1m 1m
2m

O
Fig. Ex. 9

Solution: Treating anticlockwise moment as positive, sum of moment due to


these forces about point O is,
Co-planer System of Forces  1.49

SMO = – (600 × 1) + (400 sin 60° × 1) + (400 cos 60° × 2)


\ MO = 146.41 (anticlockwise)  (Ans.)
Example 10. Cable AB and AC are used to support a pole as shown in figure
Ex. 10. If the tension in cable AC is 250 N and the sum of the moments about O
by the two cables is zero, determine the tension in cable AB.
A

9m

5m 4m
B O C
Fig. Ex. 10

Solution: Let θ1 and θ2 are the angles of cable AB and AC with horizontal
respectively.
–1 9 –1 9
θ1 = tan = 61° and θ2 = tan = 66°
5 4
Now find out components of the cable tensions at point A.
As sum of the moments about point O is zero, SMO = 0
Treating anticlockwise moment as positive and T be the tension in cable AB,
T cos 61° × 9 – 250 cos 66° × 9 = 0
\ T = 209.74 N (Ans.)
Example 11. Square ABCD is subjected to four forces and clockwise moment
as shown in figure Ex. 11. Determine (a) magnitude and direction of the resultant
force and (b) position of the resultant force from point A.

Fig. Ex. 11
1.50  Engineering Mechanics Statics

Solution: Here first resolve the forces along x and y-axes, and then determine
algebraic sum of horizontal components and vertical components,
+
→ΣH = 10 – 30 = – 20 N (here ‘–’ sign indicate that SH is acting toward
negative x Direction)
+↑SV = 20 – 40 = – 20 N (here ‘–’ sign indicate that SV is acting toward
negative y Direction)
Now we know that resultant R is calculated as,
R = ( ΣH ) 2 + ( ΣV ) 2

\ R = (20) 2 + (20) 2
\ R = 28.28 N (Ans.)
Let θ be the angle of resultant R with x-axis, then
ΣV
tan θ =
ΣH
20
\ tan θ = \ θ = 45°  (Ans.)
20
Now position of resultant force is determine by applying Varignon’s theorem
about point A. Let x be the perpendicular distance between line of action of the
resultant force and point A.
\ Rx = SMA, treating anticlockwise moment as positive
\ 28.28 x = 30 × 0.05 + 20 × 0.05 – 1.5 (note that the moment of 10 N and
40 N forces about point A is zero, as the line of action of these forces are passing
through point A)
D C

45°
35
.36

28.28N
mm

A B
Fig. Ex. 11(a)

\ x = 0.0353 m = 35.36 mm from point A , refer figure Ex. 11(a)  (Ans.)


Example 12. A rectangular block ABCD is subjected to four forces as shown
in figure Ex. 12. Determine (a) magnitude and direction of the resultant force and
(b) position of the resultant force from point A.
Co-planer System of Forces  1.51

Fig. Ex. 12
Solution: Here first resolve the forces along x and y-axes, and then determine
algebraic sum of horizontal components and vertical components,
+
→ΣH = 100 – 400 – 100 cos 30° + 200 cos 30° = – 213.39 N
(here ‘–’ sign indicate that SH is acting toward negative x Direction)
+↑SV = –200 sin 30° – 100 sin 30° = –150 N
(here ‘–’ sign indicate that SV is acting toward negative y Direction)
Now we know that resultant R is calculated as,
R = (ΣH )2 + (ΣV )2

\ R = (213.39) 2 + (150) 2
\ R = 260.83 N (Ans.)
Let θ be the angle of resultant R with x-axis, then
ΣV
tan θ =
ΣH
150
\ tan θ = \ θ = 35.10°  (Ans.)
213.39
Now position of resultant force is determine by applying Varignon’s theorem
about point A. Let x be the perpendicular distance between line of action of the
resultant force and point A.
D C
35.1°

260.83N
15
0m
m

A B
Fig. Ex. 12(a)
1.52  Engineering Mechanics Statics

\ Rx = SMA, treating anticlockwise moment as positive


\ 260.83 x = (400 × 200) – (100 × 300) – (100 sin 30° × 100)
– (200 cos 30° × 400) – (200 sin 30° × 150)
\ x = 0.150 m = 150 mm from point A, refer figure Ex. 12(a)  (Ans.)
Example 13. Three forces are acting on member AB as shown in figure Ex. 13.
Determine (a) magnitude and direction of the resultant force, (b) point where line
of action of resultant intersect member AB.
Solution: Here first resolve the forces along x and y-axes, and then determine
algebraic sum of horizontal components and vertical components,
+
→ΣH = – 25 cos 45° + 20 sin 25° = –9.22 N
(here ‘–’ sign indicate that SH is acting toward negative x Direction)
+↑SV = – 30 – 25 sin 45° – 20 cos 25° = – 65.80 N
(here ‘–’ sign indicate that SV is acting toward negative y Direction)
Now we know that resultant R is calculated as, 30N 20N25°

R = ( ΣH ) 2 + ( Σ V ) 2 25N
45°
2 2 A B
\ R = (9.22) + (65.80)
1m 1m 2m
\ R = 66.44 N  (Ans.)
Fig. Ex. 13
Let θ be the angle of resultant R with x-axis, then
ΣV
tan θ = 82°
ΣH A B
2.07m C
65.80
\ tan θ = \ θ = 82°  (Ans.)
9.22
Now position of resultant force is determine by 66.44N
applying Varignon’s theorem about point A. Let x be
Fig. Ex. 13(a)
the perpendicular distance between line of action of the
resultant force and point A.
\ R x = SMA, treating anticlockwise moment as positive
\ 66.44 x = (–30 × 1) – (25 sin 45° × 2) – (20 cos 25° × 4)
\ x = 2.07 m from point A, refer figure Ex.13(a)
Now point where line of action of resultant intersect member AB from A as
shown in figure Ex.13(a) is determine by again applying Varignon’s theorem as
below,
SV × horizontal distance AC = SMA
65.80 × distance AC = (–30 × 1) – (25 sin 45° × 2) – (20 cos 25° × 4)
\ distance AC = 2.09 m  (Ans.)
Co-planer System of Forces  1.53

Example 14. A bracket is subjected to four forces as shown in figure Ex. 14.
Determine (a) magnitude and direction of the resultant force and (b) position
of the resultant force from point O.
0.5 kN
2 kN

3
4
1.5 kN
1m

O
1m 1m 1m
1.5m

45°
1kN
Fig. Ex. 14

Solution: Let θ be the angle of 2 kN force with horizontal.


–1 3
θ = tan = 36.86°
4
Now resolving the forces along x and y-axes, and then determine algebraic
sum of horizontal components and vertical components,
+
→ΣH = 2 cos 36.86° – 1 sin 45° = 0.8931 kN
+↑SV = 1.5 + 0.5 + 2 sin 36.86° – 1 cos 45° = 2.49 kN
Now we know that resultant R is calculated as,
R = (ΣH )2 + (ΣV )2

\ R = (0.8931)2 + (2.49)2
\ R = 2.65 kN  (Ans.)
Let θ be the angle of resultant R with x-axis, then
ΣV
tan θ =
ΣH
2.49
\ tan θ = \ q = 70.26°  (Ans.)
0.8931
Now position of resultant force is determine by applying Varignon’s theorem
about point O. Let x be the perpendicular distance between line of action of the
resultant force and point O.
\ R x = SMA, treating anticlockwise moment as positive
1.54  Engineering Mechanics Statics

\ 2.65 x = (1.5 × 1) + (0.5 × 2) + (2 sin 36.86° × 2) – (2 cos 36.86° × 1)


– (1 cos 45° × 3) – (1 sin 45° × 1.5)
\ x = 0.0442 m from point O  (Ans.)
Example 15. A bracket is subjected to four forces and a moment as shown in
figure Ex. 15. Determine (a) magnitude and direction of the resultant force, (b)
point where line of action of resultant intersect member OA measured from O.

Fig. Ex. 15

Solution: Resolving the forces along x and y-axes, and then determine algebraic
sum of horizontal components and vertical components,
+
→ΣH = 300 N
+↑SV = –200 – 100 – 150 = –450 N
(here ‘–’ sign indicate that is acting toward negative y direction)
Now we know that resultant R is calculated as,
R = ( ΣH ) 2 + ( Σ V ) 2

\ R = (300) 2 + (450) 2
\ R = 541 N  (Ans.)
Let θ be the angle of resultant R with x-axis, then
ΣV
tan θ = Fig. Ex. 15(a)
ΣH
450
\
tan θ = \ θ = 56.30°  (Ans.)
300
Now let x be the distance where line of action of resultant intersect member
OA from O as shown in figure Ex.15(a) is determine by applying Varignon’s
theorem as below,
SH x = ∑ MO treating anticlockwise moment as positive
300 x = – (300 × 2) – (100 × 2) – (150 × 4) + 200
\ x = 4 m from O  (Ans.)
Co-planer System of Forces  1.55

Solved Examples Based on Parallel Forces


Example 1. Determine the magnitude of the resultant of parallel force system
shown in figure Ex. 1. Also determine the position of the resultant force from
point O.
30N 60N 20N 40N

O
2m 3m 2m 4m

Fig. Ex. 1

Solution: Here magnitude of resultant is algebraic sum of all parallel forces.


R = +↑SV = – 30 – 60 + 20 – 40
\ R = –110 N = 110 N downward  (Ans.)
Now position of resultant is finding out by applying Varignon’s theorem at
point O.
\ Moment due to resultant R about point O = Sum of moment due to all
forces about point O
R

O
Fig. Ex. 1(a)

\ –R × d = – (30 × 2) – (60 × 5) + (20 × 7) – (40 × 11)


(Treating anticlockwise moment as positive)
–110 × d = –660
\ d = 6 m Refer figure Ex.1(a)
Example 2. A pole is subjected to parallel forces as shown in figure Ex. 2.
Determine (a) magnitude and direction of the resultant force and (b) position of
the resultant force from point O.

Fig. Ex. 2
1.56  Engineering Mechanics Statics

Solution: Here magnitude of resultant is algebraic sum of all parallel forces.


+
R = →ΣH = 200 – 800 – 100 + 150
\ R = –550 N = 550 N acting towards left  (Ans.)
Now position of resultant is finding out by applying Varignon’s theorem at
point O.

Fig. Ex. 2(a)


\ Moment due to resultant R about point O = Sum of moment due to all
forces about point O
\ 550 × d = –150 × 2 + 100 × 3 + 800 × 4 – 200 × 5
(Treating anticlockwise moment as positive)
\ d = 4 m from O Refer figure Ex.2(a)  (Ans.)
Example 3. Three parallel forces are acting on the lever as shown in figure Ex.3.
Determine (a) magnitude and direction of the resultant force and (b) position of
the resultant force from point O.

Fig. Ex. 3
Solution: Here magnitude of resultant is algebraic sum of all parallel forces.
R = +↑SV = – 180 – 100 + 40
\ R = –240 N = 240 N downward  (Ans.)
Now position of resultant is finding out by applying Varignon’s theorem at
point O. 240N
\ Moment due to resultant R about point O 2.125m
= Sum of moment due to all forces about point O
\ –R × d = –(180 × 2) – (100 × 3.5) + (40 × 5) O
(Treating anticlockwise moment as positive)
Fig. Ex. 3(a)
–240 × d = –510
\ d = 2.125 m Refer figure Ex.3(a)  (Ans.)
Co-planer System of Forces  1.57

Example 4. Four parallel forces are acting on the lever as shown in figure Ex.4.
Determine (a) magnitude and direction of the resultant force and (b) position of
the resultant force from point O.
25N 25N 15N 20N

3m 2m 2m 3m

Fig. Ex. 4

Solution: Here magnitude of resultant is algebraic sum of all parallel forces.


R = +↑SV = –25 + 25 + 15 – 20
\ R = – 5 N = 5 N downward  (Ans.)
Now position of resultant is finding out by applying Varignon’s theorem at
point O.
\ Moment due to resultant R about point O = Sum of moment due to all
forces about point O
5N
1m

Fig. Ex. 4(a)

\ R × d = (25 × 5) – (25 × 2) + (15 × 2) – (20 × 5)


(Treating anticlockwise moment as positive)
5 × d = 5
\ d = 1 m to the left of point O Refer figure Ex.4(a)  (Ans.)
Example 5. Four parallel forces are acting on lever as shown in figure Ex.5.
Determine magnitude of P1 and P2 so that the four forces produce downward
resultant of 150 N acting at 4 m from O

Fig. Ex. 5
1.58  Engineering Mechanics Statics

Solution: Here magnitude of resultant is algebraic sum of all parallel forces.


R = +↑SV = 100 – P2 + P1 – 50
Now it is given that magnitude of resultant force is 150 N
\ – 150 = – 50 + P1 – P2 + 100
\ P1 – P2 = – 200  \ P1 = P2 – 200 (Eq. 1)
Now by applying Varignon’s theorem at point O,
\ Moment due to resultant R about point O = Sum of moment due to all
forces about point O
\ 150 × 4 = – 2P1 + 5P2 – 700
(Treating anticlockwise moment as positive)
2P1 – 5P2 + 1300 = 0
2(P2 – 200) – 5P2 + 1300 = 0 Substituting value of P1 from (Eq. 1)
2P2 – 400 – 5P2 + 1300 = 0
900 = 5P2 – 2P2
900 = 3P2
\ P2 = 300 N  (Ans.)
Substituting value of P2 in (Eq. 1)
P1 = 300 – 200 P1 = 100 N  (Ans.)
Example 6. Four parallel forces are acting on circle of diameter 2 m as shown
in figure Ex.6. Determine (a) magnitude and direction of the resultant force and
(b) position of the resultant force from point O.
30N 50N

45° 30°
60°
30°

80N 40N
Fig. Ex. 6
Solution: Here magnitude of resultant is algebraic sum of all parallel forces.
R = +↑SV = 50 + 30 + 40 – 80
\ R = 40 N upward  (Ans.)
Now position of resultant is finding out by applying Varignon’s theorem at
point O.
\ Moment due to resultant R about point O = Sum of moment due to all
forces about point O
\ 40 × d = (50 × 1 cos 30°) – (30 × 1 cos 45°) + (80 × 1 cos 60°) + (40 × 1
sin 30°) (Treating anticlockwise moment as positive)
40 × d = 82
\ d = 2.05 m from point O  (Ans.)
Co-planer System of Forces  1.59

Solved Examples Based on Force-Coule System and Equivalent System

Example 1. Replace 100 N force shown in figure Ex. 1 by equivalent force


couple at point A.

Fig. Ex. 1

Solution: Here transfer the force parallel to itself along with its moment at A as
below:
Introduce a system of 100 N forces at point A as shown in figure Ex. 1(a)
Now force 100 N acting at point O is replaced at point A by keeping 100 N
force as it is and a couple moment as shown in figure Ex. 1(b)
M = 100 × 5 = 500 N.m anticlockwise.  (Ans.)

Fig. Ex. 1(a) Fig. Ex. 1(b)

Example 2. Two systems of forces are shown in figure Ex. 2. Are they equivalent?
50N 25N

2m 2m 4m

System 1 System 2
25N
Fig. Ex. 2
1.60  Engineering Mechanics Statics

Solution: As we know that two systems are equivalent only when the sum of the
forces and sum of the moments about an arbitrary point are equal.
Now check for sum of the forces of two systems.
+↑SFSystem 1 = 25 – 50 = –25 N
+ ↑SFSystem 2 = –25 N
Now check for sums of the moments of two systems about left end. Treating
anticlockwise moment as positive
SMSystem 1 = –50 × 2 = –100 N.m = 100 N.m clockwise
SMSystem 2 = –25 × 4 = –100 N.m = 100 N.m clockwise
As sums of the forces of two systems are equal and sum of the moments of
two systems about an arbitrary point are equal. Two systems are equivalent.
Example 3. Replace the 200 N force shown in figure Ex. 3 by equivalent force
couple at point A.
200N

50° B A

2m

Fig. Ex. 3

Solution: Here transfer the force parallel to itself along with its moment at A as below:
Introduce a system of 200 N forces at point A as shown in figure Ex. 3(a)
200N 200N 200 N

B A

200N      460 Nm
Fig. Ex. 3(a) Fig. Ex. (b)

Now force 200 N acting at point B is replaced at point A by keeping 200 N


force as it is and a couple moment as shown in figure Ex. 3(b)
M = 200 sin 50° × 3 = 460 N.m clockwise.  (Ans.)
Example 4. The 50 N force is acting on lever as shown in figure Ex. 4. Replace
the force by force and couple moment at point O that will have an equivalent
effect.
Co-planer System of Forces  1.61

50N

2m
60°
O

Fig. Ex. 4
Solution: Here transfer the force parallel to itself along with its moment at O as
below:
Introduce a system of 50 N forces at point O as shown in figure Ex. 4(a)
50N

50N 50N
   
Fig. Ex. 4(a)              Fig. Ex. 4(b)
Now force 200 N acting at point A is replaced at point O by keeping 50 N
force as it is and a couple moment as shown in figure Ex. 4(b)
M = 50 × 2 sin 60° = 86.60 N.m anticlockwise.  (Ans.)
Example 5. A plate is subjected to two couples as shown in figure Ex.5.
Determine the sum of the moments exerted on the plate by the two couples.

Fig. Ex. 5
Solution: Here, 60 N couple acts in clockwise direction and 40 N couple acts in
anticlockwise direction. Now treating anticlockwise as positive,
\ SM = –60 × 100 + 40 × 180
= – 6000 + 7200
\ SM = 1200 N.mm anticlockwise  (Ans.)
1.62  Engineering Mechanics Statics

Example 6. A plate is subjected to two couples as shown in figure Ex.6.


Determine the magnitude of force P if the resultant couple moment is 300 N.m
anticlockwise.

Fig. Ex. 6

Solution: Let θ be the angle of force P with horizontal.


1
\ tan θ =   \ θ = 26.57°
2
Now resultant couple moment = sum of the moments exerted on the plate by
the two couples.
Treating anticlockwise moment as positive,
\ 300 = (P cos 26.57° × 2) + (P sin 26.57° × 1) – (150 × 0.5)
\ P = 168.16 N  (Ans.)
Example 7. A rectangular plate is subjected to a force – couple moment system
as shown in figure Ex.7. Replace this system by a single force and determine its
position from x-axis.

Fig. Ex. 7

Solution: Here first replace clockwise couple moment by two equal and opposite
forces of magnitude 1500 N as shown in left hand part of figure Ex.7(a). The 1500
N force acting towards left at point O and another 1500 N force acting towards
right at point A, forms a couple. Here arm of couple is d and moment of couple is
given by M = P × d, where M =180 N.m clockwise, P = 1500 N and d is unknown.
Co-planer System of Forces  1.63

Fig. Ex. 7(a)

M 150
\d= = = 0.1 m = 100 mm from x-axis as shown in the right hand
P 1500
part of figure Ex.7(a).  (Ans.)
Example 8. Replace the 100 N force actin at A on a square plate as shown in
figure Ex. 8 by equivalent force and couple moment at point O.

Fig. Ex. 8

Solution: Here transfer the force parallel to itself along with its moment at O as
below:
Introduce 100 N forces at point O as shown in the left hand part of figure Ex. 8(a).
Now calculate the moment of a 100 N force about point O, (treating
anticlockwise as positive)

50°
100N
100N
281.75Nm
O
50° 50° 100N
100N
Fig. Ex. 8(a)

M = – 100 cos 50° × 2 – 100 sin 50° × 2


M = –281.75 = 281.75 N.m clockwise.  (Ans.)
equivalent force and couple moment at point O is shown in the right hand
part of figure Ex.8(a).
1.64  Engineering Mechanics Statics

Example 9. Two couples act on a pole as shown in figure Ex. 9. Determine the
magnitude of P so that resultant couple moment is 300 N.mm clockwise.

Fig. Ex. 9

Solution: The resultant couple moment is 300 N.mm clockwise,


\ – 300 = sum of moments of two couples (treating anticlockwise as positive)
\ – 300 = 150 × 500 – P × 800 sin 30° ( here 800 sin 30° is the arm of couple P)
\ P = 188.25 N  (Ans.)
Example 10. Replace the force system as shown in figure Ex. 10 by resultant
force and couple moment at point O that will have an equivalent effect.
300N

0.75m
O
300N 1m 1m

900N
Fig. Ex. 10
Solution: Here first determine the resultant force as below,
+
→ΣH = –300 + 300 = 0
+↑SV = –900 N (here ‘–’ sign indicate that SV is acting toward negative y direction)
Now we know that resultant R is calculated as,
R = ( ΣH ) 2 + ( Σ V ) 2 O

\ R = 2
(0) + (900) 2 1125Nm

\ R = 900 N
900N
Now determine sum of moments of all forces
Fig. Ex. 10(a)
about point O, (treating anticlockwise as positive)
SMO = 300 × 0.75 + 900 × 1 = 1125 N.m anticlockwise
Co-planer System of Forces  1.65

Resultant force and couple moment at point O of given system is shown in


figure Ex.10(a).
Example 11. Replace the force system acting on a bracket as shown in figure
Ex. 11 by resultant force and couple moment at point A that will have an equivalent
effect.

Fig. Ex. 11

Solution: Here first resolve the forces along x and y-axes, and then determine
algebraic sum of horizontal components and vertical components,
+
→ΣH = 200 cos 20° – 100 sin 30° = 138 N
+↑SH = 200 sin 20° – 100 cos 30° = –18.19 N
(here ‘–’ sign indicate that SV is acting toward negative y direction)
Now we know that resultant R is calculated as,
R = ( ΣH ) 2 + ( Σ V ) 2 7.50°
1036.34
Nm 139.20N
\ R = (138) 2 + (18.19) 2
\ R = 139.19 N
Let θ be the angle of resultant R with x-axis, then
ΣV
tan θ =
ΣH
18.19
\ tan θ = \ θ = 7.50° Fig. Ex. 11(a)
138
Now determine sum of moments of all forces about point A, (treating
anticlockwise as positive)
SMA = 350 + 100 cos 30° × 2.5 + 200 cos 20° × 2.5 = 1036.34 N.m anticlockwise
Resultant force and couple moment at point A of given system is shown in
figure Ex.11 (a).
1.66  Engineering Mechanics Statics

Example 12. A square plate of 800 mm side is subjected to a couple and force as
shown in figure Ex.12. Replace the force system acting on plate with an equivalent
force – couple system at O. (O is center of square plate)

Fig. Ex. 12

Solution: It is identifying from given figure that two 60 N forces form a couple,
therefore resultant force of the given force system is the remaining force i.e. 50 N.
\ R = 50 N and its inclination θ with horizontal axis is 20°.
For moment purpose first determine distance ab and cd. Refer figure Ex.12.
In triangle oab
oa cos 40° = 400 \ oa = 522.16 mm
ab = 522.16 sin 40° \ ab = 335.64 mm
In triangle ocd
oc cos 30° = 400 \ oc = 461.89 mm
cd = 461.89 sin 30° \ cd = 230.94 mm
Now determine sum of moments of all forces about O, (treating anticlockwise
as positive),

50N

866.73
Nm

Fig. Ex. 12(a)

SMO = – (50 cos 20° × 335.64) + (50 sin 20° × 400) – (60 sin 45° × 400)
+ (60 cos 45° × 230.90) + (60 cos 45° × 400)
= 866.73 N.m anticlockwise
Resultant force and couple moment at point O of given system is shown in
figure Ex.12 (a).
Co-planer System of Forces  1.67

Example 13. A bracket is subjected to three forces as shown in figure Ex.13.


Replace the force system with an equivalent force – couple system at B.
6m
A
200N
1m

30°
400N
2m
45°
300N

B 3m

Fig. Ex. 13

Solution: Here first resolve the forces along x and y-axes, and then determine
algebraic sum of horizontal components and vertical components,
+
→ΣH = 400 cos 30° + 300 sin 45° = 558.54 N
+↑SV = 200 – 400 sin 30° – 300 cos 45° = – 212.13 N
(here ‘–’ sign indicate that SV is acting toward negative y direction)
Now we know that resultant R is calculated as,
A
R = ( ΣH ) 2 + ( Σ V ) 2

\ R = (558.54) 2 + (212.13) 2
\ R = 597.47 N
Let θ be the angle of resultant R with x-axis, then
ΣV
tan θ =
ΣH
212.13 1541.34 N.M
\ tan θ = \ θ = 20.80
558.54 B
Now determine sum of moments of all forces 597.47N
about B, (treating anticlockwise as positive)
Fig. Ex. 13(a)
SMB = (200 × 6) – (400 cos 30° × 2)
3
– (400 sin 30° × 6) – (300 sin 45° ×  2 – × 2 – (300 cos 45° × 3)
 6 
= –1541.34 N.m = 1541.34 N.m clockwise
Resultant force and couple moment at point A of given system is shown in
figure Ex.13 (a).
1.68  Engineering Mechanics Statics

Example 14. Two systems of forces are shown in figure Ex. 14. Determine the
value of R and P if these two systems are equivalent.
A 150 A

3m 1m 3m 1m
200N C C

B B R

2m 2m

D E
250N D E
1m 1m
System 1 System 2
P P

Fig. Ex. 14

Solution: As we know that two systems are equivalent only when the sum of the
forces and sum of the moments about an arbitrary point are equal.
SHSystem 1 = SHSystem 2
\ –150 + 200 – 250 = – R \ R = 200 N (Ans.)
Now determine sums of the moments of two systems about B and equate
them. (Treating anticlockwise moment as positive)
SMSystem 1B = SMSystem 2B
150 × 1 – 250 × 2 = – P × 1 \ P = 350 N  (Ans.)
Example 15. Two systems of forces are shown in figure Ex. 15. Are they
equivalent?

Fig. Ex. 15

Solution: As we know that two systems are equivalent only when the sum of the
forces and sum of the moments about an arbitrary point are equal.
Co-planer System of Forces  1.69

Now check for sum of the forces of two systems.


+
→ ΣFSystem1 = 100 – 200 = –100 N
+
→ ΣFSystem 2 = –200 + 100 = –100 N
Now check for sums of the moments of two systems about O. (Treating
anticlockwise moment as positive)
SMSystem 1 = –100 × 3 + 500 = 200 N.m anticlockwise
SMSystem 2 = 200 × 3 – 100 × 1.5 – 300 = 150 N.m anticlockwise
As sums of the forces of two systems are equal but sum of the moments of two
systems about an arbitrary point are not equal. Two systems are not equivalent.
Example 16. Two equivalent system of forces and moments act on the plate are
shown in figure Ex. 16. Determine the force P and couple M.

Fig. Ex. 16

Solution: As we know that two systems are equivalent only when the sum of the
forces and sum of the moments about an arbitrary point are equal.
+ +
→ ΣH System1 = → ΣH System 2
60 N = 60 N
+↑SVSystem 1 = +↑SVSystem 2
\ 100 – 20 = P – 60 \ P = 140 N  (Ans.)
Now determine sums of the moments of two systems about A and equate
them. (Treating Anticlockwise moment as positive)
+ +
→ΣM System1A = →ΣM System 2 A

–60 × 3 – 20 × 4 + M = 200 \ M = 460 N. m  (Ans.)


1.70  Engineering Mechanics Statics

SUMMARY
• Force is the action of a body about another body.
♦ Unit of the force is N, kN
♦ 1 N = (1 kg) (1 m/s2) = 1 kg-m/s2
• Magnitude, Direction, Sense and Point of application are characteristics
of the force.
• When two or more forces act on a body they are called to form a system
of forces.
♦ Coplanar forces are those forces whose lines of action lie on the same
plane.
♦ Non-Coplanar forces are those forces whose lines of action do not lie
on the same plane
• Resultant is a single force which produces the same effect as produced by
number of forces when acting together.
• The resultant of two or more collinear forces is simply their algebraic
sum.
• The resultant of two concurrent forces can be found out by means of the
parallelogram law or the triangle law.
R = P12 + P22 + 2 P1P2 cos θ .......... Magnitude of resultant
P2 sin θ
tan α = .......... Direction of resultant
P1 + P2 cos θ
• The resultant of more than two concurrent forces can be found out by
means of rectangular components method or the polygon law.
R = (ΣH )2 + (ΣV )2 .......... Magnitude of resultant
ΣV
tan θ = .......... Direction of resultant
ΣH
• Resolution is the process of splitting up the given force into components,
without changing its effect on the body.
• The rotational effect produced by force is known as moment of the force.
Moment = Force × Distance
M = F × d
• The location of the resultant of parallel force system is found out by
means of Varignon’s Theorem (Principle of Moment)
• Two equal, unlike parallel, non-collinear forces form a couple.
• Force can be transfer parallel to itself by means of force couple system.
• Two force systems that produce the same external effects on a rigid body
are said to be equivalent.
Co-planer System of Forces  1.71

PROBLEMS

1. Two forces of 400 N and 150 N are


acting on a hook as shown in figure
Prob.1.Determine magnitude resultant of
these two forces.

Fig. Prob. 1
2. Two forces of 375 N and P are acting
on a hook as shown in figure Prob.2. If
the magnitude of the resultant force is to
be 200 N, directed along positive y-axis,
determine the magnitude of force P.

Fig. Prob. 2
3. Two forces of 150 N and 200 N are y
acting on a plate as shown in figure Prob.3.
Determine magnitude and direction of
resultant of these two forces. 150 N 50° 200 N
30°
x

Fig. Prob. 3
4. Two forces of 160 N and 200 N are acting 160 N
on a bracket as shown in figure Prob.4. 1m
Determine magnitude and direction of O
resultant of these two forces. 0.5m

200 N
2m

Fig. Prob. 4
1.72  Engineering Mechanics Statics

5. Resolve 300 N force in to components v


along u and v directions. Refer figure
Prob.5. 70° 300 N
30°
u

Fig. Prob. 5
6. Resolve 300 N force in to components
along u and v directions. Refer figure 60° 30°
Prob.6. O
u

v
Fig. Prob. 6
7. Two forces of 150 N and 110 N are V Y
acting on a hook as shown in figure Prob.7. 20°
U
Determine (a) magnitude and direction 40°
of resultant of these two forces. (b) The 150 N
magnitudes of two other forces Pu and Pv 110 N 20° 20°
that would have the same resultant. X

Fig. Prob. 7
8. Determine magnitude of the resultant
force and its direction from x-axis of
three concurrent forces as shown in figure
Prob.8.

Fig. Prob. 8
9. A rectangular plate is subjected to three
concurrent forces as shown in figure Prob.9. 60 N 50 N
Determine magnitude of the resultant force 35°
and its direction from x-axis. 45°
55°
50 N

Fig. Prob. 9
Co-planer System of Forces  1.73

10. Determine magnitude of the resultant 150 N


force and its direction from x-axis of
three concurrent forces as shown in figure 4
Prob.10. 250 N
3
1
O 3

1
1

400 N

Fig. Prob. 10
11. Determine magnitude of the resultant
force and its direction from x-axis of
four concurrent forces as shown in figure
Prob.11.

Fig. Prob. 11
12. The magnitude of the resultant of three Y
concurrent forces is 45 N and directed 60 N
along positive y axis. Determine magnitude 30°
P
of force P. Refer figure Prob.12
30°
X
30°

50 N
Fig. Prob. 12
13. Determine the moments of the 120 N A B C
force shown in figure Prob.13 about points
A, B and C.

70mm 80mm 100mm 120 N

Fig. Prob. 13
14. Determine moment of each forces
shown in figure Prob.14 about points O.

Fig. Prob. 14
1.74  Engineering Mechanics Statics

15. Two forces are applied to a bracket 1.5 m


as shown in figure Prob.15. Determine 150 N
the sum of the moments of the two forces

1.5 m
about point A and point B.
100 N
A

1.5 m
B
Fig. Prob. 15
16. Three forces are applied to a bracket 200 N 30°
as shown in figure Prob.16. Determine the
400 N
sum of the moments of the three forces
about point O. 0.5 m
O
0.5 m 0.5 m 1m

200 N
Fig. Prob. 16
17. The moment exerted by the weight W
about point B is 300 N.m. Determine the
moment by the weight W about point A.
Refer figure Prob.17.

Fig. Prob. 17
18. Two forces are applied to a circular 30° 50 N
plate as shown in figure Prob.18. determine
moment of each forces about point O.

R = 0.8 m O 45°
30 N

Fig. Prob. 18
19. Determine the resultant of the three 30 N 50 N
1.4 m
forces and the couple C acting on a
65° 50°
rectangular plate as shown in figure
Prob.19.
1,2 m

40 N

45 Nm
Fig. Prob. 19
Co-planer System of Forces  1.75

20. Determine the resultant of the three


forces acting on a beam as shown in figure
Prob.20.

Fig. Prob. 20
21. Three parallel forces are acting on 4 kN
3.5 kN
4.5 kN
a beam as shown in figure Prob.21.
Determine the resultant force and its A B
location with respect to support A.

1.4 m 1.5 m 1 m 1.2 m

Fig. Prob. 21
22. The resultant of the force system shown 300 mm
in figure Prob.22 is a 120 N.mm anticlock-
wise couple. Determine magnitude of P1, 40°
P2 and C. 200 mm
P1
C
50 N
200 mm

P2
40°

Fig. Prob. 22
23. Determine the moment of the couple y
shown in figure Prob.23. Also determine
perpendicular distance between the two 150 N 150 N
forces.

30° 30°
x

100 m m

Fig. Prob. 23
1.76  Engineering Mechanics Statics

24. If P = 80 N determine the couple


moment produced. Refer figure Prob.24. 0.1 m 0.1 m
P

Fig. Prob. 24
25. In Prob. No.24, If the couple moment produced by two forces P is 30 N.m. Determine
magnitude of P.
26. A rectangular plate is subjected to two
forces and the couple as shown in figure
Prob. 26. Replace the force system with an
equivalent force – couple system at point O

Fig. Prob. 26
27. A bracket is subjected to three forces as 6.5 kN
shown in figure Prob. 27. Replace the force 30°
system with an equivalent force – couple
system at point A.
1m

60°
1m
4.5 kN 3.5 kN
A
1m 1m

Fig. Prob. 27
Unit 2

In this unit, we discuss criteria for equilibrium of different force


system and concept of free body diagram, which is perhaps the most
important tool of mechanics. We then discuss analysis procedure for
finding out unknown forces and moments using free body diagram and
equilibrium equations.
Equilibrium of System of Forces
2.1 Equilibrium
Equilibrium is the state of rest of a body i.e. the body does not move and also
does not rotate about any point under the action of forces. For a rigid body in
static equilibrium, the external forces and moments are balanced and will impart
no translational or rotational motion to the body;
Static equilibrium is the
however it could be translating or rotating at a
state in which the sum of
constant velocity.
the forces and moments
When object is in equilibrium under the on each particle of the
action of system of forces, following conditions system is zero
are satisfied:
1. The algebraic sum of the components of the forces in any direction must
equal to zero.
2. The algebraic sum of the moments of the forces about any point must
equal zero.
Equilibrium conditions for different force system are shown in Table 2.1
Table 2.1 Equilibrium Conditions for Different Force Systems

Force System Equilibrium Condition


Collinear force system SP = 0
\ Resultant, R = 0
Concurrent force system SH = 0 (sum of horizontal components) and
SV = 0 (sum of vertical components)
\ Resultant, R = 0
Parallel force system SP = 0 and
SMO = 0
(O is any point in the plane of the forces)
Non-concurrent force system SH = 0 (sum of horizontal components)
SV = 0 (sum of vertical components) and
SMO = 0 (O is any point in the plane of the forces)

2.2  Free Body Diagram (F.B.D.)


Free body diagram is one of the most important tools of mechanics. Free-body
diagrams are fundamental to all engineering disciplines that are concerned with
the effects that forces have on bodies.
A free-body diagram of a body is a diagrammatic representation or a sketch
of a body in which the body is shown completely separated from all surrounding
bodies, including supports. The term free implies that all supports have been
2.4  Engineering Mechanics Statics

removed and replaced by the forces (reactions) that they exert on the body.
Forces that we show on a free body diagram can be divided into two categories:
(a) Reactive Forces: Reactive forces known as reactions are those forces
that are exerted on a body by the supports to which it is attached.
Reactions at supports and connections are of only two types:
1. If a support or connection prevents translation in some direction, then
a force may be developed in that direction.
2. If a support or connection prevents rotation about some axis, then a
moment may be developed around that axis
(b) Applied forces: Forces acting on a body that are not provided by the
supports are called applied forces.
Different supports and their reactions are shown in Table 2.2
Table 2.2 Supports and their Reactions

Free Body
Support Example Description
Diagram (F.B.D.)
Normal W Normal force
contact on oriented
horizontal perpendicular to
smooth W surface.
surface Direction of normal
force is towards the
N body.
Normal Normal force
contact on oriented
inclined perpendicular to
smooth W inclined surface.
surface Direction of normal
 force is towards the
body.
Rope, cable, Cable
T Tension oriented
wire  
along cable.
Direction of tension
is away from the
body.
Spring Spring force
oriented along
long axis of spring.
Spring Force is pull
P
if spring is in
tension, and force
is push if spring is
in compression.
Equilibrium of System of Forces  2.5

Normal W Normal force


contact on oriented
horizontal P W P perpendicular to
surface with surface.
friction Friction force
Rough Surface F
parallel to the
N surface.
Normal W Normal force
contact on oriented
inclined perpendicular to
surface with W inclined surface.
friction Rough Surface
 Friction force
F
parallel to the
N
inclined surface.
Link Force oriented
P along the link.
Link
Force can be push
or pull
Roller

Normal force
oriented
N perpendicular to the
surface supporting
the roller.


N
Pin/Hinge Force in terms
A of components
oriented along x
and y-axes.
Fixed Force in terms of
Ax
components
A M oriented along x
Ay
and y-axes. Moment
about z-axis.
Sliding in
guide
Normal force
oriented
perpendicular to
guide.

N
2.6  Engineering Mechanics Statics

2.2.1  General Procedure for Constructing a Free Body


Diagram
1. Choose the free body to be used, isolate it from any other body and sketch
its outline.
2. Locate all external forces on the free body and clearly mark their
magnitude and direction. This should include the weight of the free body,
which is applied at the centre of gravity.
3. Locate and mark unknown external forces and reactions on the free-body
diagram.
4. Include all dimensions that indicate the location and direction of forces.
The free-body diagram of a rigid body can be reduced to that of a particle.
The free-body of a particle is used to represent a point and all forces acting on it.

2.2.2  Examples on Free Body Diagram (FBD)


Some examples on free body diagram are shown in Table 2.3
Table 2.3 Examples on Free Body Diagrams

Different Systems Free Body Diagram


W

2 1 2 1
N2 N1

T
W


W N

W
W W
N5 N6

W W N1 N4

N2 N3
Equilibrium of System of Forces  2.7

W
B
W
30° 
A Ax 30° NB


Ay

10 N
10 N 20 Nm
20 Nm A B
A B
Ax

3m 3m 4m 3m 3m 4m
Ay By

T
 A
Ax
1m 2m
Ay

W
(Smooth
Surface)

2.8  Engineering Mechanics Statics

10 N 15 N
1m 2m
10 N 15 N
C
A B
30° 1m Ax
30°
PCD
D Ay

10 N N
1.5m 1m 1m

Ax
M
25 Nm
Ay

W B
A
2m 1m

10 kN

2.3  Equilibrium for Different Force System


   (Based on number of forces)
1. Equilibrium under Two Force System: If a body is in equilibrium
under the action of two forces, then two forces must be collinear, of equal
magnitude and act in opposite direction as shown in figure 1.

P
Fig. 1

A body will not be in equilibrium under the action of two equal and
opposite parallel forces, because these forces produce a couple.
Equilibrium of System of Forces  2.9

2. Equilibrium under Three Force System: If a body is in equilibrium


under the action of three forces, then three forces acting on a body must
be concurrent or parallel.
To satisfy the condition SM = 0 the line of action of three non-parallel forces
P1, P2 and P3 must pass through the same point i.e. P1
the three force must be concurrent as shown in
figure 2.
When the three forces P1, P2 and P3 acting on
a body are parallel and acts in same direction then
body will not be in equilibrium, as their resultant
R = P1 + P2 + P3. If the three forces are acting P3
in opposite direction and their magnitude is so P2
adjusted that the resultant force is zero and sum of Fig. 2
moment of three forces about any point is zero then
body will be in equilibrium.

Lami’s Theorem:
It states, “If a body is in equilibrium under the action of three concurrent forces,
then each force is proportional to the sine of angle between the other two forces”
P1



P3

P2

Let P1, P2 and P3 are three forces acting at point O. Let the angle between
P1 and P2 is α, angle between P2 and P3 is γ and P3 and P1 is β, then according
to Lami’s theorem,
P1 P P
= 2 = 3 (Eq 2.1)
sin γ sin β sin ∝
Below steps are following in equilibrium analysis of a body,
1. Draw a free-body diagram of the body which shows all of the forces and
moment that act on the body.
2. Write the equilibrium equations for all the forces and moments that
appear on the free-body diagram.
3. Solve the equilibrium equations for the unknowns.
2.10  Engineering Mechanics Statics

Solved Examples Based on Equilibrium of Forces


Example 1. The system of forces shown in figure Ex. 1 is in equilibrium. String
BC is horizontal. Determine tension in string AB (T1), tension in string BC (T2)
and tension in string CD (T3).

A B
35° 30°

B C

40 N 50 N

Fig. Ex. 1
Solution: Consider the free body diagram at point B and C as shown in figure Ex.
1(a). Since the system is in equilibrium and three concurrent forces are acting at
point B and C, we can use Lami’s theorem at point B and C.

Figure Ex. 1(a)

Consider point B from F.B.D. and apply Lami’s theorem.


40 T1 T2
= =
sin 125° sin 90° sin 145°
40 sin 90°
\ T1 =   \ T1 = 48.83 N  (Ans.)
sin 125°
40 sin 145°
T2 =   \ T2 = 28 N  (Ans.)
sin 125°
Consider point C from F.B.D. and apply Lami’s theorem.
28 T3 50
= =
sin150° sin 90° sin120°
28sin 90°
\ T3 =   \ T3= 56 N  (Ans.)
sin150°
Equilibrium of System of Forces  2.11

Example 2. Three concurrent forces acting at point O as shown in figure Ex 2


are in equilibrium. Find out the magnitude of force P and angle θ.

Fig. Ex. 2

Solution: Since the system is in equilibrium and three concurrent forces are acting
at point O we can use Lami’s theorem at point O.
P 15 20
= = ...(Eq. 1)
sin 90° sin (180 − θ) sin (90 + θ)
Now from Trigonometry formulas we know,
sin (180 – θ) = sin θ and sin (90 + θ) = cos θ, putting these in Eq. 1,
P 15 20
\ = = ...(Eq. 2)
sin 90° sin θ cos θ
15 sin θ 15
\ =   \ tan θ =   \ θ = 37°  (Ans.)
20 sin θ 20
Now from equation (2),
P 15
=
sin 90° sin 37°
15sin 90°
\ P =   \ P = 25 N  (Ans.)
sin 37°
Alternate Approach: The above problem can also be solved by applying
conditions of equilibrium,
+
i.e. →ΣH = 0 and +↑SV = 0
SH = 0: P cos θ – 20 = 0
20
\ P = ...(Eq. 1)
cos θ
SV = 0: P sin θ – 15 = 0
15
\ P = ...(Eq. 2)
sin θ
2.12  Engineering Mechanics Statics

Now from equation Eq.1 and Eq.2


15 20
=
sin θ cosθ
15 sin θ 15
\ = \ tan θ = \ θ = 37°  (Ans.)
20 cos θ 20
Now from Eq. 1
20
P =   \ P = 25 N  (Ans.)
cos37°
Example 3. Check whether two systems of forces shown in figure Ex. 3 are in
equilibrium or not.

Fig. Ex. 3

Solution: A system is in equilibrium only when it satisfies all equations of


equilibrium, i.e.
+
→ΣH = 0, +↑SV = 0 and SM = 0 (Treating anticlockwise moment as positive)
Now consider system 1 and apply equation of equilibrium,
 4  3
SH =   60 –   80
 5  5
= 48 – 48  \ SH = 0
 4  3
SV = 100 –   80 –   60
 5  5
= 100 – 64 – 36  \ SV = 0
As system 1 is concurrent force system, the sums of moment about point O
is zero,
i. e. SM = 0. So system 1 satisfies all equations of equilibrium,
\ System 1 is in equilibrium  (Ans.)
Now consider system 2 and apply equation of equilibrium,
4 3
SH =   60 –   80
5 5
= 48 – 48  \ SH = 0
Equilibrium of System of Forces  2.13

4 3
SV = 100 –   80 –   60
5 5
= 100 – 64 – 36  \ SV = 0
As system 2 is non-concurrent force system, taking the sum of moments
about point O. Here since line of action of forces 60 N and 80 N passing through
point O, they produces zero moment, but the 100 N force will cause a clockwise
moment of 200 N.m.
\ SMO = 200 N.m clockwise.
So system 2 satisfies sum of forces equations of equilibrium but it does not
satisfy sum of moment equation of equilibrium,
\ System 2 is not in equilibrium  (Ans.)
Example 4. A 250 N block rest on an incline smooth surface as shown in figure
Ex.4. Determine normal reaction at contact and force P to maintain equilibrium.

Fig. Ex. 4

Solution: Applying conditions of equilibrium i.e. +SH = 0 and + SV = 0 to


the free body diagram shown in figure Ex. 4 (a), 250 N
P
SH = 0: P – 250 sin 30° = 0 30°
\ P = 125 N  (Ans.)
SV = 0: N – 250 cos 30° = 0
\ N = 216.5 N  (Ans.)
Alternate Approach: Since the system is in
30°
equilibrium and three concurrent forces are acting at point N
O as shown in figure Ex. 4 (b), we can use Lami’s theorem
Fig. Ex. 4(a)
at point O.
N

30°
P

30°
O

250 N

Fig. Ex. 4(b)


2.14  Engineering Mechanics Statics

P 250 N
= =
sin150° sin 90° sin120°
P 250
=
sin150° sin 90°
\ P = 125 N  (Ans.)
250 N
=
sin 90° sin120°
\ N = 216.5 N  (Ans.)
Example 5. A 100 N block rest on an incline smooth surface as shown in figure
Ex.5. If θ = 40°, determine normal reaction at contact and force P to maintain
equilibrium.

P

100 N

20°

Fig. Ex. 5

Solution: Applying conditions of equilibrium i.e. +SH = 0 and +SV = 0


to the free body diagram shown in figure Ex. 5(a),
SH = 0: P cos 40° – 100 sin 20° = 0
\ P = 44.65 N  (Ans.)
SV = 0: N – P sin 40° – 100 cos 20° = 0
N = 44.65 sin 40° + 100 cos 20°
\ N = 122.66 N  (Ans.)
Alternate Approach: Since the system is in
equilibrium and three concurrent forces are acting Fig. Ex. 5(a)
at point O as shown in figure Ex. 5 (b), we can use
Lami’s theorem at point O.
P 100 N
= =
sin160° sin130° sin 70°
P 100
=
sin160° sin130°
\ P = 44.65 N  (Ans.)
100 N
=
sin130° sin 70°
Fig. Ex. 5(b)
\ N = 122.67 N  (Ans.)
Equilibrium of System of Forces  2.15

Example 6. In Example 5 if P = 60 N, determine the angle θ to maintain


equilibrium.
Solution: Applying conditions of equilibrium i.e. +SH = 0 and +SV = 0 to
the free body diagram shown in figure Ex. 5(a),
SH = 0: 60 cos θ – 100 sin 20° = 0  \ θ = 55.24°  (Ans.)
Example 7. Determine the tension in each cord for equilibrium of 1000 N and
3 m long wooden log as shown in figure Ex.7.

Fig. Ex. 7

Solution: In order to maintain equilibrium the force developed in AB must be


equal to weight of wooden log, i.e. 1000 N. Consider the free body diagram at
point B figure Ex. 7(a). Since the system is in equilibrium and three concurrent
forces are acting at point B, we can use Lami’s theorem, 1000 N
1000 PBC PBD
= =
sin 75° sin150° sin135°
1000 PBC
=
sin 75° sin150° 45° 60°
\ PBC = 517.64 N  (Ans.)
1000 PBD PBC
= PBD
sin 75° sin135°
\ PBD = 732.05 N  (Ans.) Fig. Ex. 7(a)

Example 8. Four forces are acting at point O as shown in figure Ex.8. Determine
magnitude of P1 and P2 for equilibrium.

Fig. Ex. 8
2.16  Engineering Mechanics Statics

+
Solution: Applying conditions of equilibrium i.e. →ΣH = 0 and +↑SV = 0
SH = 0: 80 + 60 cos 40° – P2 cos 30° = 0  \ P2 = 145.44 N  (Ans.)
SV = 0: P1 – P2 sin 30° – 60 sin 40° = 0
P1 = 145.44 sin 30° + 60 sin 40°  \ P1 = 111.29 N  (Ans.)
Example 9. A circular plate is subjected to three concurrent forces as shown in
figure Ex.9. Determine magnitude of force P1 and P2 for equilibrium.

100 N

P1 2
3
P2

Fig. Ex. 9

Solution: Since the system is in equilibrium and three concurrent forces are acting
at point O we can use Lami’s theorem at point O. Refer figure Ex.9(a)
2
Let θ be the angle of 90 N force with horizontal, tan θ =
3
\ θ = 33.69° 100 N
100 P1 P2
= =
sin 90° sin146.3° sin123.69°
100 P1 33.69 ° 56.3 °
=
sin 90° sin146.3°
\ P1 = 55.48 N  (Ans.) P1 P2

100 P2 Fig. Ex. 9(a)


=   \ P2 = 83.20 N  (Ans.)
sin 90° sin123.69°
Alternate Approach: The above problem can also be solved by applying
conditions of equilibrium,
+
i.e. →ΣH = 0 and +↑SV = 0
SH = 0: P2 cos 56.30° – P1 cos 33.69° = 0
P2 cos 56.30° = P1 cos 33.69°
\ P2 = 1.499 P1 ...(Eq.1)
SV = 0: 100 – P2 sin 56.30° – P1 sin 33.69° = 0
100 = P2 sin 56.30° + P1 sin 33.69°
Substitute value of P2 from Eq. 1
Equilibrium of System of Forces  2.17

\ 100 = 1.499 P1 sin 56.30° + P1 sin 33.69°


100 = 1.8023 P1  \ P1 = 55.48 N  (Ans.)
Now from equation Eq. 1
P2 = 1.499 × 55.48  \ P2 = 83.17 N  (Ans.)
Example 10. A 100 kg mass is supported by cables as shown in figure Ex.10.
determine tension in each cables.

Fig. Ex. 10

Solution: Consider the free body diagram as shown in figure Ex. 10 (a)

Fig. Ex. 10(a)

Applying conditions of equilibrium +↑SV = 0 to 100 kg block of free body


diagram,
SV = 0 : TAB – 100 × 9.81 = 0  \ TAB = 981 N  (Ans.)
Now since the system is in equilibrium and three concurrent forces are acting
at point B and D, we can use Lami’s theorem at point B and D. First applying
Lami’s theorem at point B,
TAB TBC T
= = BD
sin120° sin150° sin 90°
2.18  Engineering Mechanics Statics

981 TBC
=   \ TBC = 566.38 N  (Ans.)
sin120° sin150°
981 TBD
=   \ TBD = 1132.76 N  (Ans.)
sin120° sin 90°
Now applying Lami’s theorem at point D,
TBD TDE T
= = DF
sin105° sin165° sin 90°
1132.76 TDE
=   \ TDE = 303.52 N  (Ans.)
sin105° sin165°
1132.76 TDF
=   \ TDF = 1172.71 N  (Ans.)
sin105° sin 90°
Example 11. Block A is supported by two weights of 100 N and 50 N as shown
in figure Ex.11. Determine the weight of block A and angle θ for equilibrium.

50N 50° 
100N

Fig. Ex. 11

+
Solution: Applying conditions of equilibrium i.e. →ΣH = 0 and +↑SV = 0 to
free body diagram as shown in figure Ex.11(a)
50N
100N

50° 

W
Fig. Ex. 11(a)

SH = 0: 100 cos θ – 50 cos 50° = 0  \ θ = 71.25°  (Ans.)


SV = 0: 50 sin 50° + 100 sin 71.25° – W = 0  \ W = 133 N  (Ans.)
Equilibrium of System of Forces  2.19

Example 12. A roller 400 N rests against vertical and inclined smooth surfaces
as shown in figure Ex.12. Determine the normal reactions at A and B.

A
30°

Fig. Ex. 12

Solution: Consider free body diagram as shown in figure Ex.12 (a). Since the
system is in equilibrium and three concurrent forces are acting at the centre of
roller, we can use Lami’s theorem at point O.
400 N

30° NA
NB
NB

400N
30°
NA

Fig. Ex. 12(a)

400 NA NB
= =
sin120° sin 90° sin150°
400 NA
=   \ NA = 461.88 N  (Ans.)
sin120° sin 90°
400 NB
=   \ NB = 230.94 N  (Ans.)
sin120° sin150°
Alternate Approach: The above problem can also be solved by applying
conditions of equilibrium,
+
i.e. →ΣH = 0 and +↑SV = 0
SH = 0: NA sin 30° – NB = 0
NB = NA sin 30° ...(Eq. 1)
SV = 0: NA cos 30° – 400 = 0  \ NA = 461.88 N  (Ans.)
Now from Eq. 1
NB = 461.88 sin 30°  \ NB = 230.94 N  (Ans.)
2.20  Engineering Mechanics Statics

Example 13. Two smooth sphere of weight 150 N and radius 200 mm each rest
in a channel as shown in figure Ex.13. Assuming all contact surfaces as smooth,
determine reactions at points of contacts.

Fig. Ex. 13

Solution: First determine angle α. Refer figure Ex.13 (a),


O1O2 = 200 + 200 = 400 mm, m O2
0m
40
O1E = 750 – 200 – 200 = 350 mm
 E
O1
O E 350
cos α = 1 =   \ a = 29° 350mm
O1O2 400
Fig. Ex. 13(a)
Now applying conditions of equilibrium i.e.
+
→ΣH = 0 and +↑SV = 0 to the right hand side roller
of free body diagram shown in figure Ex. 13(b),

Fig. Ex. 13(b)

SH = 0: NC cos 29° – ND = 0  \ ND = NC cos 29° ..(Eq.1)


SV = 0: NC sin 29° – 150 = 0  \ NC = 309.39 N  (Ans.)
From Eq. 1, ND = 309.39 cos 29°  \ ND = 270.60 N  (Ans.)
+
Applying conditions of equilibrium i.e. →ΣH = 0 and +↑SV = 0 to the left
hand side roller of free body diagram shown in figure Ex. 13(b),
Equilibrium of System of Forces  2.21

SH = 0: NA – NC cos 29° = 0
\ NA = 309.39 cos 29°  \ NA = 270.6 N  (Ans.)
SV = 0: NB – 150 – NC sin 29° = 0
\ NB = 150 – 309.39 sin 29°  \ NB = 0 N  (Ans.)
Example 14. Two smooth sphere of weight 400 N and 200 N respectively rest
on an inclined surface as shown in figure Ex.14. Assuming all contact surfaces as
smooth, determine reactions at points of contacts.

Fig. Ex. 14

+
Solution: Applying conditions of equilibrium i.e. →ΣH = 0 and +↑SV = 0 to the
left hand side roller of free body diagram shown in figure Ex. 14(a),
200 N

400 N
NB 
NB
 40°
NC

NA
10°
Fig. Ex. 14(a)

SH = 0: NA sin 10° – NB cos α = 0 ...(Eq.1)


SV = 0: NA cos 10° – 400 – NB sin α = 0 ...(Eq.2)
+
Applying conditions of equilibrium i.e. →ΣH = 0 and +↑SV = 0 to the right
hand side roller of free body diagram shown in figure Ex. 14(a),
SH = 0: NB cos α – NC sin 40° = 0 ...(Eq.3)
SV = 0: NB sin α – 200 + NC cos 40° = 0 ...(Eq.4)
Now from Eq. 1 and Eq. 3
NA sin 10° – NC sin 40° = 0
NA sin 10° = NC sin 40°  \ NA = 3.7016 NC ...(Eq.5)
2.22  Engineering Mechanics Statics

From Eq. 2 and Eq. 4


NA cos 10° + NC cos 40° – 200 – 400 = 0
NA cos 10° + NC cos 40° = 600
Substitute value of NA from Eq. 5,
3.7016 NC × cos 10° + NC cos 40° = 600  \ NC = 136 N  (Ans.)
From Eq. 5,
NA = 3.7016 × 136  \ NA = 503.41 N  (Ans.)
Now from Eq. 3,
NB cos α = 136 sin 40° = 87.419 ...(Eq. 6)
From Eq. 4,
NB sin α = 200 – 136 cos 40° = 95.817 ...(Eq.7)
Dividing Eq. 7 by Eq. 6,
N B sin α 95.817
=
N B cos α 87.419
\ tan α = 1.096  \ α = 47.62°
Now from Eq. 6,
NB cos 47.62° = 87.419  \ NB = 129.69 N  (Ans.)
Example 15. A roller of weight 800 N and radius 240 mm is pushed by force
P over a step at A as shown in figure Ex.15. Determine magnitude of force P to
just start the roller over the step.
P
240mm

25°
O
30mm

Fig. Ex. 15

Solution: Consider free body diagram of roller as shown in figure Ex.15(a).


Consider the equilibrium state when roller just start over the step. Now since the
system is in equilibrium, three forces acting on the roller must be concurrent at
the centre of roller, and we can use Lami’s theorem.

Fig. Ex. 15(a)


Equilibrium of System of Forces  2.23

Referring to free body diagram, first find out angle α as below,


OA = 240 mm and OB = 240 – 30 = 210 mm
Now in ∆OAB,

AB = OA2 − OB 2 = 2402 − 2102 = 116.18 N


AB 116.18
Now tan α =   \ tan α =   \ α = 28.950
OB 210
Now using Lami’s theorem,
800 P NA
= =
sin143.95° sin151.05° sin 65°
800 NA
=   \ NA = 1232 N
sin143.95° sin 65°
800 P
=   \ P = 658 N  (Ans.)
sin143.95° sin151.05°
Example 16. The boom OA and cable AC support a load of 300 N as shown in
figure Ex.16. Knowing that boom AO exerts on pin A a force along OA, determine
magnitude of force along OA and tension in cable AC.

Fig. Ex. 16

Solution: As three concurrent forces are acting at pin A, we can apply Lami’s
theorem at A. Consider free body diagram at A as shown in figure Ex.16 (a)
300 PAO TAC
= = PAO TAC
sin110° sin 120° sin 130°
40° 30°
300 PAO
=
sin 110° sin 120°
\ PAO = 276.48 N  (Ans.)
300 N
300 TAC
= Fig. Ex. 16(a)
sin 110° sin130°
\ TAC = 244.56 N  (Ans.)
Example 17. A man is holding up the 600 N bar AB by applying perpendicular
force of 400 N as shown in figure Ex.17. Determine angle θ at which he can
support the bar.
2.24  Engineering Mechanics Statics

B 3.5
m

1.5
m


A
Fig. Ex. 17

Solution: Consider free body diagram of bar as shown in figure Ex.17 (a).
B
600 N
2.5
m
1.5
m

 Ax
400 N A

Ay

Fig. Ex. 17(a)

Applying conditions of equilibrium i.e. SMA = 0, treating anticlockwise


moment as positive,
SMA = 0: 600 × 2.5 cos θ – 400 × 1.5 = 0
600 × 2.5 cos θ = 400 × 1.5  \ θ = 66.42°  (Ans.)
Example 18. The homogeneous bar AB of 300 N is resting as shown in figure
Ex.18. Assuming contact surface as smooth, determine the forces acting at A and B.

Fig. Ex. 18
Equilibrium of System of Forces  2.25

+
Solution: Applying conditions of equilibrium i.e. →ΣH = 0, +↑SV = 0 and SM
= 0 to free body diagram shown in figure Ex. 18(a),
NB
B

2.5
300 N

m
60°
Ax A

Ay

Fig. Ex. 18(a)

+
→ΣH = 0: NB + Ax = 0 ...(Eq.1)
+↑ SV = 0: Ay – 300 = 0  \ Ay = 300 N  (Ans.)
SMA = 0: 300 × 2.5 cos 60° – NB × 5 sin 60° = 0
300 × 2.5 cos 60° = NB × 5 sin 60°  \ NB = 86.60 N  (Ans.)
Now from Eq. 1, 86.60 + AX = 0  \ AX = –86.60 N  (Ans.)
Here, ‘–ve’ sign indicate that force AX will act in negative x direction.
Example 19. The homogeneous bar AB of 1000 N is resting as shown in figure
Ex.19. Assuming contact surface as smooth, determine the tension in the rope CD.

Fig. Ex. 19
+
Solution: Applying conditions of equilibrium i.e. →ΣH = 0 and SM = 0 (Treating
anticlockwise moment as positive) to free body diagram shown in figure Ex. 19(a),
T C

NB 25°
25° A
1000 N
NA
Fig. Ex. 19(a)
2.26  Engineering Mechanics Statics

+
→ΣH = 0: NB sin 25° – T = 0
T = NB sin 25° ...(Eq.1)
SMA = 0: T × 1225 sin 25° – NB × 800 + 1000 × 612.5 cos 25° = 0
Substituting value of T from Eq. 1
NB sin 25° × 1225 sin 25° – × 800 + 1000 × 612.5 cos 25° = 0
\ NB = 955 N
Now from Eq. 1, T = 955 sin 25°  \ T = 403.60 N  (Ans.)
Example 20. The homogeneous bar AB of 600 N is resting as shown in figure
Ex.20. Assuming contact surface as smooth, determine the reactions at A and B.

Fig. Ex. 20

Solution: Applying conditions of


+
equilibrium i.e. →ΣH = 0, +↑SV = 0 and NA T
SM = 0 (Treating anticlockwise moment
as positive) to free body diagram shown 30
360mm

0m 20
0m
in figure Ex. 20(a),
m
m

SH = 0: NA – NB = 0 
\ NA = NB ...(Eq.1) 600 N
NB

SV = 0: T – 600 = 0 480 mm

\ T = 600 N ...(Eq.2)
Fig. Ex. 20(a)
360
Now from free body diagram tan θ =   \ θ = 36.86°
480
SMB = 0: –T × 200 cos 36.86° + 600 × 600 cos 36.86° – NA × 360 = 0
Substituting T = 600 N from Eq. 2,
–600 × 200 cos 36.86° + 600 × 600 cos 36.86° = NA × 360
\ NA = 533.40 N  (Ans.)
Now from Eq. 1, NA = NB  \ NB = 533.40 N  (Ans.)
Equilibrium of System of Forces  2.27

Example 21. The homogeneous bar ABC of 20 kg is supported as shown in


figure Ex.21. If weight of the bar is concentrate at G, determine the reactions at
A and tension T.
A
T

1m
0.5m
G
C
2m

Fig. Ex. 21

Solution: Applying conditions of equilibrium Ay


+
i.e. →ΣH = 0, +↑SV= 0 and SM = 0 (Treating A Ax
anticlockwise moment as positive) to free T
body diagram shown in figure Ex. 21(a), 1m
SH = 0: Ax = 0  (Ans.) G
C
SV = 0: Ay + T – 20 × 9.81 = 0 1.5m 0.5m

\ Ay + T = 196.2 N  ...(Eq.1) 20×9.81


SMA = 0: 20 × 9.81 × 0.5 – T × 2 = 0 Fig. Ex. 21(a)
\ T = 49 N  (Ans.)
Now from Eq. 1,
Ay + 49 = 196.2  \ Ay = 147.2 N  (Ans.)
Example 22. The homogeneous bar ABC of is attached to a pin at A and rest
on roller support at B as shown in figure Ex.22. Neglecting the weight of the bar,
determine the reactions at A and B.
150N

A B 1m

3m 1m

Fig. Ex. 22

+
Solution: Applying conditions of equilibrium i.e. →ΣH = 0, +↑SV = 0 and
SM = 0 (Treating anticlockwise moment as positive) to free body diagram shown
in figure Ex. 22(a),
2.28  Engineering Mechanics Statics

150N

A B 1m
Ax

3m
45°
Ay NB

Fig. Ex. 22(a)

SH = 0: Ax – NB sin 45° = 0
\ Ax = NB sin 45° ...(Eq.1)
SV = 0: Ay + NB cos 45° – 150 = 0 ...(Eq.2)
SMB = 0: –Ay × 3 – 150 × 1 = 0
\ Ay = –50 N  (Ans.)
Here, ‘–ve’ sign indicate that force Ay will act in negative y direction.
Now from Eq. 2,
–50 + NB cos 45° – 150 = 0
NB cos 45° = 200 \ NB = 282.84 N  (Ans.)
Now from Eq. 1,
Ax = 282.82 sin 45°  \ Ax = 200 N  (Ans.)
Example 23. The 100 N load is attached to one and of a rope and the load is
held at rest by the force T applied to the other end of the rope as shown in figure
Ex.23. Assuming pulley as frictionless, determine the reactions at A and force T.

Fig. Ex. 23
+
Solution: Applying conditions of equilibrium i.e. →ΣH = 0, +↑SV = 0 and
SM = 0 (Treating anticlockwise moment as positive) to free body diagram shown
in figure Ex. 23(a),
Equilibrium of System of Forces  2.29

Ay
30°

A Ax

100N

Fig. Ex. 23(a)

SH = 0: Ax + T cos 30° = 0
\ Ax = –T cos 30° ...(Eq.1)
SV = 0: –100 – T sin 30° + Ay = 0
Ay = 100 + T sin 30° ...(Eq.2)
SMA = 0: –T × 100 + 100 × 100 = 0  \ T = 100 N  (Ans.)
Now from Eq. 1,
Ax = – 100 cos 30°  \ Ax = –86.60 N  (Ans.)
Here, ‘–ve’ sign indicate that force AX will act in negative x direction.
Now from Eq. 2,
Ay = 100 + 100 sin 30°  \ Ay = 150 N  (Ans.)
Example 24. A 2 m long cantilever of negligible weight is used to support 200
N load as shown in figure Ex. 24. Determine the reactions at A.

A B
2m
200N
Fig. Ex. 24
+
Solution: Applying conditions of equilibrium i.e. →ΣH = 0, +↑SV = 0 and
SM = 0 (Treating anticlockwise moment as positive) to free body diagram shown
in figure Ex. 24(a),
SH = 0: Ax = 0  (Ans.) Ax
B
CA
SH = 0: Ay – 200 = 0
2m
\ Ay = 200 N  (Ans.) Ay 200N

SMA = 0: CA – 200 × 2 = 0 Fig. Ex. 24(a)


\ CA = 400 N.m (anticlockwise)  (Ans.)
2.30  Engineering Mechanics Statics

Example 25. A 2 m long cantilever of negligible weight is used to support


50 N/m distributed load as shown in figure Ex.25. Determine the reactions at A.

Fig. Ex .25
+
Solution: Applying conditions of equilibrium i.e. →ΣH = 0, +↑SV = 0 and
SM = 0 (Treating anticlockwise moment as positive) to free body diagram shown
in figure Ex. 25(a),

Fig. Ex. 25(a)

Here, load is distributed as 50 N/m over 2 m length total load acting in


downward direction is 50 × 2 = 100 N and for moment purpose the total load (as
a point load) is assume to act at the centre of distribution as shown by dash line
in free body diagram.
SH = 0:  Ax = 0  (Ans.)
SV = 0: Ay – 50 × 2 = 0  \ Ay = 100 N  (Ans.)
SMA = 0: CA – 50 × 2 × 1 = 0  \ CA = 100 N.m (anticlockwise)  (Ans.)
Example 26. The bar AB is pinned at A and rest on roller at B. The bar AB is
loaded as shown in figure Ex.26. Neglecting the weight of the bar, determine the
reactions at A and B.
10N
30° 15Nm
A B

1m 1m 1m

Fig. Ex. 26
+
Solution: Applying conditions of equilibrium i.e. →ΣH = 0, +↑SV = 0 and SM
= 0 (Treating anticlockwise moment as positive) to free body diagram shown in
figure Ex. 26(a),
Equilibrium of System of Forces  2.31

SH = 0: Ax – 10 cos 30° = 0
\ Ax = 8.66 N  (Ans.)
SV = 0: Ay + By – 10 sin 30° = 0 ...(Eq.1)
SMA = 0: By × 3 – 15 – 10 sin 30° × 1 = 0
Fig. Ex. 26(a)
\ By = 6.67 N  (Ans.)
Now from Eq. 1,
Ay + 6.67 – 10 sin 30° = 0  \ Ay = –1.67 N  (Ans.)
Here, ‘–ve’ sign indicate that force Ay will act in negative y direction.
Example 27. A bracket is pinned at A and rest on roller at B. It is subjected to
a 100 N force as shown in figure Ex.27. Neglecting the weight of the bracket,
determine the reactions at A and B.

Fig. Ex. 27

+
Solution: Applying conditions of equilibrium i.e. →ΣH = 0, +↑SV = 0 and SM
= 0 (Treating anticlockwise moment as positive) to free body diagram shown in
figure Ex. 27(a), 100N
SV = 0: Ax + 100 = 0
\ Ax = –100 N  (Ans.)
Here, ‘–ve’ sign indicate that force AX will act in
negative x direction.
SV = 0: Ay + By = 0 ...(Eq.1)
A
SMA = 0: By × 50 – 100 × 170 = 0 Ax
\ By = 340 N  (Ans.) Ay By

Now from Eq. 1, Fig. Ex. 27(a)


Ay + 340 = 0
\ Ay = –340 N  (Ans.)
Here, ‘–ve’ sign indicate that force Ay will act in negative y direction.
2.32  Engineering Mechanics Statics

Example 28. A bracket is fixed at A and loaded as shown in figure Ex.28.


Neglecting the weight of the bracket, determine the reactions at A.
400N
1.5m
300N

200Nm 1.5m

200Nm
2.5m

Fig. Ex. 28

+
Solution: Applying conditions of equilibrium i.e. →ΣH = 0, +↑SV = 0 and SM
= 0 (Treating anticlockwise moment as positive) to free body diagram shown in
figure Ex. 28(a),
SH = 0: Ax – 300 – 200 = 0
\ Ax = 500 N  (Ans.)
SV = 0: Ay – 400 = 0
\ Ay = 400 N  (Ans.)
SMA = 0:
CA – 200 + 200 × 2.5 + 300 × 4 – 400 × 1.5 = 0
\ CA = – 900 N.m
= 900 N. m (Clockwise)  (Ans.)
Fig. Ex. 28(a)
Here, ‘–ve’ sign indicate that moment reaction
CA will act in opposite direction.
Example 29. The weight of the pickup is 3600 N and acting at G as shown in
figure Ex.29. Determine the load W on the pickup for which normal reactions at
front and rear wheels are equal.

Fig. Ex. 29
Equilibrium of System of Forces  2.33

Solution: Applying conditions of equilibrium i.e. +↑SV = 0 and SM = 0 (Treating


anticlockwise moment as positive) to free body diagram shown in figure Ex. 29(a),
Here given that normal reactions at front and rear wheels are equal,
\ NA = NB = N
SV = 0: 2N – 3600 – W = 0

Fig. Ex. 29(a)

\ W = 2N – 3600 ...(Eq.1)
SMA = 0: 3600 × 450 – N × 1120 + W × 1280 = 0
Substituting value of W from Eq. 1
3600 × 450 – N × 1120 + (2N – 3600) × 1280 = 0
\ N = 2075 N
Now from Eq. 1
W = 2 × 2075 – 3600
\ W = 550  (Ans.)
Example 30. The motorcycle shown in figure Ex.30, has a mass of 150 kg and
distance between front and rear wheel is 1.6 m. If the rear wheel exerts a 1000 N
force on the ground, determine the location of motorcycle’s centre of gravity from
front wheel also determine reaction at front wheel.

A 1.6m B

Fig. Ex. 30

Solution: Applying conditions of equilibrium i.e. +↑SV = 0 and SM = 0 (Treating


anticlockwise moment as positive) to free body diagram shown in figure Ex. 30(a),
2.34  Engineering Mechanics Statics

Fig. Ex. 30(a)

SV = 0: NA + 1000 – 150 × 9.81 = 0  \ NA = 471.5 N  (Ans.)


SMA = 0: 1000 × 1.6 – 150 × 9.81 × x = 0  \ x = 1.08 m  (Ans.)

Summary
• Equilibrium is the state of rest of a body i.e. the body does not move and
also does not rotate about any point under the action of forces.
• For collinear force system equilibrium conditions are,
SP = 0  \ Resultant, R = 0
• For concurrent force system equilibrium conditions are,
SH = 0 and SV = 0  \ Resultant, R = 0
• For parallel force system equilibrium conditions are,
SP = 0 and SMO = 0
• For non-concurrent force system equilibrium conditions are,
SH = 0, SV = 0 and SMO = 0
• A free body diagram is a sketch of the body showing all the forces acting
on it.
• If a body is in equilibrium under the action of two forces then two forces
must be collinear, of equal magnitude and act in opposite direction.
• If a body is in equilibrium under the action of three forces, then three
forces acting on a body must be concurrent or parallel.
• Lami’s theorem is useful for three concurrent forces.
Equilibrium of System of Forces  2.35

Problems

1. The system of forces shown in figure Prob. 1 Y


is in equilibrium. Determine P1 and P2.
50 N
P1
40° 30°
50° 60°
70 N P2

Fig. Prob. 1
2. The system of forces shown in figure Prob. 2 15 N
is in equilibrium. Determine P1 and P2. P1 20° P2

40°
80N 40N
O

Fig. Prob. 2
3. Two cables are tied together and loaded at B
A
as shown in figure Prob. 3. Determine tension in 30° 40°
C
cable AB and cable CB.
B

50
Kg

Fig. Prob. 3
4. A 500 N block is supported by a strut and two
cables as shown in figure Prob. 4. Determine the A
force in strut CB and cable AB.

B 20°
60°

500 N C

Fig. Prob. 4
5. The force P is applied to ring at B as shown
in figure Prob. 5. If the tension in cable AB and
BC is 150 N, determine magnitude of force P.

Fig. Prob. 5
2.36  Engineering Mechanics Statics

6. Determine the tension developed in each


E
cable to maintain equilibrium of the 800 N load.
Refer figure Prob. 6 A
4
30°
C 3
B

D
800 45°
N

Fig. Prob. 6
7. The 500 N roller is supported by the cable AB A
rests against a smooth wall as shown in figure
Prob. 7. Determine force in cable and reaction
250mm
on roller from the wall. Radius of roller is
150 mm.
B

Fig. Prob. 7
8. Neglecting the weight of bracket, determine 2m
all reactions at O due to 160 N force acting on
bracket as shown in figure Prob. 8.

60°
3m

160 N

O
Fig. Prob. 8
9. Three cylinders, each of diameter 300 mm
and weight 500 N are stacked as shown in
figure Prob. 9. Determine reactions exerted on
cylinders by the inclined surfaces.

30° 30°

Fig. Prob. 9
Equilibrium of System of Forces  2.37

10. A cantilever is subjected to loading as shown


in figure Prob. 10. Determine reactions at the
fixed support.

Fig. Prob. 10
11. A bracket is subjected to the loading as 15 N
shown in figure Prob. 11. Determine force P and 30°
reactions at pin support O.

2m

3m

O
1m

Fig. Prob. 11
12. A beam is subjected to loading as shown in 20° 80 N
figure Prob. 12. Determine reactions at A and B.
Take AC = 1.5 m and CB = 2 m. 100 N

A B
C

Fig. Prob. 12
13. A bar of 200 N with roller end is supported
by wire CB and two surfaces as shown in figure
Prob. 13. Determine reactions at A and B and
tension in wire CB.

Fig. Prob. 13
2.38  Engineering Mechanics Statics

14 A bracket is subjected to the loading as 30 N 40 N


shown in figure Prob. 14. Determine reactions 400 400 400
at A and B. mm mm mm

A
300 mm
25 N
300 mm

B
500 mm
Fig. Prob. 14
15. A uniform bar AB of length 1.5 m and weight A
200 N resting on two inclined smooth surfaces
as shown in figure Prob. 15. Determine angle θ
B
for equilibrium of bar. 

45° 30°

Fig. Prob. 15
Unit 3

In this unit, we discuss analysis of structures. In first part of this unit,


we focus on analysis of truss where we determine forces acting on
interconnected members of truss. In second part of this unit, we focus
on analysis of beams where we determine internal forces induced in a
beam because of external transverse loading. Analysis of these forces
is important before design of any members.
Analysis of Structure
3.1 trusses
Truss is one of the important engineering structures. Truss is used to support the
load. A frame which is composed of members
A truss is a structure
joined at their ends to form a structure, called a
composed of
truss. Members of a truss are two-force members
interconnected members
that are subjected to two equal and opposite
joined at their ends.
forces directed along member. Truss structures
are used in buildings, bridges, roofs, transmission
towers etc.
Truss structure in two dimensions, is known as plane truss. Basic triangular
truss is as shown in figure 1, where members of a truss
are connected by a pin joints at their ends. For keeping
the truss structure stationary all degrees of freedom must
be restricted. A truss which does not collapse under
loading is known as rigid or stable truss.
For a stable truss, the following relation exists,
n = 2 j – 3
Where,  n  is the number of members and j  is the
Fig. 1
number of pin joints.
A truss for which above relationship does not satisfy is known as unstable
truss.

3.2  Types of Truss


Some standard types of truss structure are shown in figure 2.
Warren truss consists of longitudinal members joined by angled cross-
members that forms alternate inverted equilateral triangle-shaped spaces along
its length. Howe truss includes vertical members and diagonals that slope
upward towards the centre of its length. Pratt truss includes vertical members and
diagonals that slope downward towards the centre of its length. K truss includes
vertical members and two tilted members forming K.
3.4  Engineering Mechanics Statics

   
Fig. 2

3.3 Analysis of Truss


When a truss is loaded its members are subjected to either tensile or compressive
forces. In truss analysis we determine these forces induced in members. In
figure 3, a member of a truss is shown with tensile and compressive forces.

Fig. 3.

3.3.1 Assumptions in Truss Analysis


Following assumptions are made in analysis of a truss,
1. All members of a truss are connected by frictionless pins.
2. Load or forces may be applied at joints only.
3. Each member may have two joints only.
4. Weights of members are assumed to be negligible.
5. The truss is a stable truss.

3.3.2 Methods of Truss Analysis


There are commonly two methods that can be used for the analysis of a truss,
1. Method of joints
2. Method of sections
1. Method of Joints: In method of joints each joint of the members are
treated as a particle. Forces induced in each members of a truss are
determine as follow,
• First, if necessary, determine reactions at the supports considering
entire truss as a single object.
Analysis of Structure  3.5

• Isolate an individual joint and draw free body diagram by assuming


forces in members as a tensile force.
• Apply conditions of equilibrium i.e. SH = 0, and SV = 0 to the free
body diagram of the joint.
• If the calculated value from equilibrium equations is positive then
our assumption of tensile force in member is valid. If the calculated
value from equilibrium equations is negative then our assumption of
tensile force in member is not valid and nature of the force in member
is compressive.
• Repeat this process for all joints and determine forces in all the
members of a truss.
2. Method of Sections: In method of sections forces induced only in certain
members of a truss are determine as follow,
• First, if necessary, determine reactions at the supports considering
entire truss as a single object.
• Select a section of a truss by cutting maximum three members.
• Draw the free body diagram of the section by showing axial forces in
the members (away from joints).
• Apply conditions of equilibrium i.e. SH = 0, SV = 0 and SM = 0 to the
free body diagram of the section.

Solved Examples Based on Method of Joints


Example 1. Determine the forces in all the members of a truss with the loading
and support system shown in figure Ex. 1.

Fig. Ex. 1

Solution: To determine reactions, consider the free body diagram of the entire
truss as shown in figure Ex. 1(a).
The triangle PQR is a right angled triangle with angle PRQ = 90°
3.6  Engineering Mechanics Statics

PR = PQ cos 60°
= 6 × 0.5 = 3 m
Distance of line of action of 50 kN force
from P,
PS = PR cos 60°
= 3 × 0.5 = 1.5 m
Now apply conditions of equilibrium i.e.
SM = 0 about P, (Treating anticlockwise moment
as positive)
Fig. Ex. 1(a)
SMP = 0:  RQ × 6 – 50 × 1.5 = 0
\ RQ = 12.5 kN
Now apply conditions of equilibrium i.e. +↑SV = 0 to free body diagram
SV = 0 : RP + RQ = 50 kN
Substituting RQ = 12.5 kN, RP = 37.5 kN
+
Joint P: Now apply conditions of equilibrium i.e. →ΣH = 0, +↑SV = 0 to free
body diagram of joint P as shown in figure Ex. 1(b).
SH = 0: FPQ + FPR cos 60° = 0 ...(Eq. 1)
SV = 0: FPR sin 60° + Rp = 0
Rp
FPR = − = – 43.3 kN (Compression)  (Ans.)
sin 60°
Substituting in Eq. 1
FPQ + (–43.3) cos 60° = 0 Fig. Ex. 1(b)
FPQ = 21.65 kN (Tensile)  (Ans.)
Joint Q: Now apply conditions of equilibrium i.e. +↑SV = 0 to free body diagram
of joint Q as shown in figure Ex. 1(c).
SV = 0: FRQ sin 30° + 12.5 = 0
12.5
FQR = – = – 25 kN (Compression)  (Ans.)
sin 30°
Example 2. Use the method of joints to determine the force
in each member of the truss shown in figure Ex. 2. Fig. Ex. 1(c)

Fig. Ex. 2
Analysis of Structure  3.7

Solution: For this Simple truss, the member forces can be determined without
solving the support reactions.
+
Joint B: Apply conditions of equilibrium i.e. →ΣH = 0, +↑SV = 0 to free body
diagram of joint B as shown in figure Ex. 2(a).
SH = 0:
FBC cos 40° – 1500 – FAB cos 50° = 0 ...(Eq. 1)
SV = 0:
– FAB sin 50° – FBC sin 40° = 0
FAB sin 50° = –FBC sin 40°
FAB = –0.84 FBC ....(Eq. 2) Fig. Ex. 2(a)
Substituting the value of FAB in Eq. 1
FBC cos 40° – 1500 – (–0.84 FBC) cos 50° = 0
Solving, FBC = 1145.03 kN (Tensile)  (Ans.)
From Eq. 2,
FAB = –0.84 × 1145.03 = –961.83 kN (Compressive)  (Ans.)
+
Joint C: Apply conditions of equilibrium i.e. →ΣH = 0 to free body diagram of
joint C as shown in figure Ex. 2(b).
SH = 0: – FBC cos 40° – FAC = 0
–1145.03 cos 40° – FAC = 0
FAC = –877.14 kN (Compression)  (Ans.)
Example 3. Use the method of joints to determine the force
in each member of the truss shown in figure Ex. 3.
Fig. Ex. 2(b)

Fig. Ex. 3

Solution: For this simple truss the member forces can be determined without
solving the support reactions.
3.8  Engineering Mechanics Statics

+
Joint B: Apply conditions of equilibrium i.e. →ΣH = 0, +↑SV = 0 to free body
diagram of joint B as shown in figure Ex. 3(a).
4 3
SH = 0 : – FAB + FBC – 20 cos 45° = 0   ...(Eq. 1)
5 5
3 4
SV = 0 : – FAB – FBC – 20 sin 45° = 0   ...(Eq. 2)
5 5
Solving Eq. 1 and Eq. 2 yields Fig. Ex. 3(a)
FAB = –28.28 kN (compression)  (Ans.)
FBC = 20.8 kN (Tensile)  (Ans.)
+
Joint C: Apply conditions of equilibrium i.e. →ΣH = 0 to free body diagram of
joint C as shown in figure Ex. 3(b).
3
SH = 0 : – FBC – FAC = 0
5
3 3
FAC = – FBC = – × 20.8 = – 12.48kN
5 5
(Compression)  (Ans.)
Example 4. Use the method of joints to determine the Fig. Ex. 3(b)
force in each member of the truss shown in figure Ex. 4

Fig. Ex. 4

Solution: For this simple truss the member forces can


be determined without solving the support reactions.
+
Joint D: Apply conditions of equilibrium i.e. →ΣH
= 0, +↑SV = 0 to free body diagram of joint D as
shown in figure Ex. 4(a).
SH = 0: –FAD + FCD = 0 ...(Eq. 1)
SV = 0: FBD – 2000 = 0
FBD = 2000 kN (Tensile)  (Ans.) Fig. Ex. 4(a)
Analysis of Structure  3.9

+
Joint B: Apply conditions of equilibrium i.e. →ΣH = 0,
+↑SV = 0 to free body diagram of joint B as shown in
figure Ex. 4(b).
3 4
SH = 0 : FBC – FAB – 1500 = 0 ...(Eq. 2) Fig. Ex. 4(b)
5 5
4 3
SV = 0 : – FBC – FAB – 2000 = 0 ...(Eq. 3)
5 5
Solving Eq. 2 and Eq. 3 yields,
FAB = –2400 kN (Compressive)
FBC = –700 kN (Compressive)
+
Joint C: Apply conditions of equilibrium i.e. →ΣH = 0 to free body diagram of
joint D as shown in figure Ex. 4(c).
3
SH = 0 : – FCD – FBC = 0
5
3
– FCD – (–700) = 0
5
FCD = 420 kN (Tensile)  (Ans.)
Fig. Ex. 4(c)
Now from Eq. 1,
– FAD + 420 = 0 FAD = 420 kN (Tensile)  (Ans.)
Example 5. Use the method of joints to determine the force in each member of
the truss shown in figure Ex. 5.

Fig. Ex. 5

Solution: To determine reactions, consider the free body diagram of the entire
+
truss as shown in figure Ex. 5(a) and apply conditions of equilibrium i.e. →ΣH
= 0, +↑SV = 0 and SM = 0 about A, (Treating anticlockwise moment as positive)
3.10  Engineering Mechanics Statics

Fig. Ex. 5(a)

SH = 0: Ax = 0
SV = 0: Ay + Dy – 5 – 7 = 0
Ay + Dy = 12 ...(Eq. 1)
SMA = 0: Dy × 8 – 7 × 6 – 5 × 2 = 0
Dy = 6.5 kN
From Eq. 1,
Ay + 6.5 = 12  Ay = 5.5 kN
+
Joint D: Apply conditions of equilibrium i.e. →ΣH = 0, +↑SV = 0 to free body
diagram of joint D, as shown in figure Ex. 5(b).
SH = 0: –FDE – FCD cos 60° = 0 ...(Eq. 2)
SV = 0: FCD sin 60° + 6.5 = 0
FCD = –7.50 kN (Compressive)  (Ans.)
From Eq. 2,
–FDE – (–7.50) cos 60° = 0
Fig. Ex. 5(b)
FDE = 3.752 kN (Tensile)  (Ans.)
+
Joint A: Apply conditions of equilibrium i.e. →ΣH = 0, +↑SV = 0 to free body
diagram of joint A as shown in figure Ex. 5(c).
SH = 0: FAE + FAB cos 60° = 0  ...(Eq. 3)
SV = 0: FAB sin 60o + 5.5 = 0
FAB = –6.35 kN (Compressive)  (Ans.)
Substituting the value in Eq. 3,
FAE + (–6.35) cos 60° = 0
FAE = 3.175 kN (Tensile)  (Ans.)
Fig. Ex. 5(c)
Analysis of Structure  3.11

+
Joint B: Apply conditions of equilibrium i.e. →ΣH = 0, +↑SV = 0 to free body
diagram of joint B as shown in figure Ex. 5(d).
SH = 0: FBC + FBE cos 60° – FAB cos 60° = 0  ...(Eq. 4)
SV = 0: –FAB sin 60° – FBE sin 60° – 5 = 0
Substituting value of FAB,
–(-6.35) sin 60° – FBE sin 60° – 5 = 0
FBE = 0.576 kN (Tensile)  (Ans.)
Fig. Ex. 5(d)
Substituting value of FAB and FBE in Eq. 4,
FBC + (0.576) cos 60° – (–6.35) cos 60° = 0
FBC = –3.4763 kN (Compressive)  (Ans.)
+
Joint C: Apply conditions of equilibrium i.e. →ΣH = 0
to free body diagram of joint C as shown in figure Ex. 5(e).
SH = 0: FCD cos 60° – FCE cos 60° – FBC = 0
Substituting value of FCS and FBC
(–7.5) cos 60° – FCE cos 60° – (–3.476) = 0
Fig. Ex. 5(e)
FCE = – 0.548 kN (Compressive)  (Ans.)
Example 6. Use the method of joints to determine the force in each member of
the truss shown in figure Ex. 6.

Fig. Ex. 6

Solution: For this simple truss the member forces can be determined without
solving the support reactions.
+
Joint A: Apply conditions of equilibrium i.e. →ΣH = 0, +↑SV = 0 to free body
diagram of joint A as shown in figure Ex. 6(a).
SH = 0: FAE + FAB cos 30° = 0 ...(Eq. 1)
SV = 0: FAB sin 30° – 800 = 0
FAB = 1600 kN (Tensile)  (Ans.)
3.12  Engineering Mechanics Statics

Substituting the value in Eq. 1,


FAE + 1600 cos 30° = 0
FAE = –1385.64 kN (Compressive)  (Ans.)
Joint B: Apply conditions of equilibrium i.e., +SH = 0
and +SV = 0 to free body diagram of joint B as shown
in figure Ex. 6(b).
Fig. Ex. 6(a)
SH = 0: FBC – FAB = 0
FBC = FAB = 1600 kN (Tensile)  (Ans.)
SV = 0: –FBE – 650 = 0
FBE = –650 kN (Compressive)  (Ans.)
+
Joint E: Apply conditions of equilibrium i.e. →ΣH = 0
and +↑SV = 0 to free body diagram of joint E as shown in
Fig. Ex. 6(b)
figure Ex. 6(c).
SH = 0: FEF – FEA – FBE cos 60° + FCE cos 60° = 0 ...(Eq. 2)
SV = 0: FBE sin 60° + FCE sin 60° – 1000 = 0
Substituting value of FBE,
–650 sin 60° + FCE sin 60° = 1000
FCE = 1804.70 kN (Tensile)  (Ans.)
Substituting values of FEA, FBE and FCE in Eq. 2
FEF – (–1385.64) – (–650) cos 60° + (1804.70)
cos 60° = 0
FEF = –2613 kN (Compressive)  (Ans.) Fig. Ex. 6(c)
+
Joint C: Apply conditions of equilibrium i.e. →ΣH = 0
and +↑SV = 0 to free body diagram of joint C as shown in figure Ex. 6(d).

Fig. Ex. 6(d)

SH = 0: FCD + FCF cos 60° – FCE cos 60° – FCB cos 30° = 0 ...(Eq. 3)
SV = 0: –FCF sin 60° – FCE sin 60° – FCB sin 30° = 0
Substituting value of FCE and FCB,
–FCF sin 60° – 1804.70 sin 60° – 1600 sin 30° = 0
FCF = –2728.45 kN (Compressive)  (Ans.)
Substituting values of FCF, FCE and FCB in Eq. 3
FCD + (–2728.45) cos 60° – 1804.70 cos 60° – 1600 cos 30° = 0
FCD = 3652.21 kN (Tensile)  (Ans.)
Analysis of Structure  3.13

Solved Examples Based on Method of Sections


Example 1. The length of each truss member in figure Ex. 1 is 7 m. Find the
forces in members CD, DE & EF using method of sections.

Fig. Ex. 1

Solution: To determine reactions, consider the free body diagram of the entire
+
truss as shown in figure Ex. 1(a) and apply conditions of equilibrium i.e. →ΣH
= 0, +↑SV = 0 and SM = 0 about A, (Treating anticlockwise moment as positive)

Fig. Ex. 1(a)

SH = 0: Ax = 0
SV = 0: Ay + By –1200 – 1000 – 800 – 1000 = 0
Ay + By = 4000 ...(Eq.1)
SMA = 0: By × 35 – 1200 × 7 – 1000 × 14 – 800 × 21 – 1000 × 28 = 0
By = 1920 kN
From Eq. 1,
Ay + 1920 = 4000  Ay = 2080 kN
Now consider free body diagram of the cut part of the truss to the left section
of member DF as shown in figure Ex. 1(b) and apply conditions of equilibrium
i.e. +↑SV = 0 and SM = 0 (Treating anticlockwise moment as positive)
3.14  Engineering Mechanics Statics

Fig. Ex. 1(b)

SME = 0: 1200 × 7 – 2080 × 14 – FCD × (7 Sin 60°) = 0


FCD = –3418 kN (Compressive)  (Ans.)
SMD = 0: 1000 × 3.5 + 1200 × 10.5 – 2080 × 17.5 + FEF × (7 Sin 60°) = 0
FEF = 3348.6 kN (Tensile)  (Ans.)
SV = 0: FDE sin 60° – 1000 – 1200 + 2080 = 0
FDE = 138.56 kN (Tensile)  (Ans.)
Example 2. Find the forces in members CD, CF, and FG of the truss shown in
figure Ex. 2 using method of sections.

Fig. Ex. 2

Solution: To determine reactions, consider the free body diagram of the entire
truss as shown in figure Ex. 2(a) and apply conditions of equilibrium SM = 0
about A, (Treating anticlockwise moment as positive)
Analysis of Structure  3.15

SMA = 0: Ey × 30 – 1000 × 10 – 1500 × 20 = 0


Ey = 1333.33 kN
Now consider free body diagram of the cut part of the truss to the right section
of member CG as shown in figure Ex. 2(b) and apply conditions of equilibrium
i.e. SM = 0 (Treating anticlockwise moment as positive),

Fig. Ex. 2(b)

SME = 0: –FCF sin 60° × 10 + 1500 × 10 = 0


FCF = 1732.10 kN (Tensile)  (Ans.)
SMF = 0: FCD × (10 sin 30°) + 1333.33 × 10 = 0
FCD = –2666.66 kN (Compressive)  (Ans.)
SMC = 0: –FFG × (15 tan 30°) – 1500 × 5 + 1333.33 × 15 = 0
FFG = 1443.49 kN (Tensile)  (Ans.)
Example 3. Find the forces in members BC, BF and EF of the truss shown in
figure Ex. 3 using the member of sections.
3.16  Engineering Mechanics Statics

Fig. Ex. 3

Solution: For this truss, the forces in members can be determined without solving
for the support reactions.
Now consider free body diagram of the cut part of the truss to the right section
of member BE as shown in figure Ex. 3(a) and apply conditions of equilibrium
i.e. +↑SV = 0 and SM = 0 (Treating anticlockwise moment as positive),
−1 7
q = tan = 54.46°
5
SMB = 0: –FEF × 7 – 5 × 10 = 0
FEF = –7.14 kN (Compressive)  (Ans.)
SMF = 0: FBC × 7 – 5 × 5 = 0
FBC = 3.57 kN (Tensile)  (Ans.)
SV = 0: FBF sin 54.46° – 5 = 0
FBF = 6.14 kN (Tensile)  (Ans.) Fig. Ex. 3(a)
Example 4. Find the forces in members AB and FG of the truss shown in figure
Ex. 4, using method of sections.

Fig. Ex. 4
Analysis of Structure  3.17

Solution: For this truss, the forces in members can be determined without solving
for the support reactions. Now consider free body diagram of the cut part of the
truss to the right section as shown in figure Ex. 4(a) and apply conditions of
equilibrium i.e. SM = 0 (Treating anticlockwise moment as positive),

Fig. Ex. 4(a)

SMF = 0: FAB × 5 – 6 × 3 – 4 × 6 = 0
FAB = 8.4 kN (Tensile)  (Ans.)
SMB = 0: – FFG × 5 – 6 × 3 – 4 × 6 = 0
FFG = –8.4kN (Compression)  (Ans.)
Example 5. Find the forces in members BC, CF and EF of the truss shown in
figure Ex. 5. using method of sections.

Fig. Ex. 5

Solution: To determine reactions, consider the free body diagram of the entire
truss as shown in figure Ex. 5(a) and apply conditions of equilibrium SM = 0
about A, (Treating anticlockwise moment as positive)
3.18  Engineering Mechanics Statics

Fig. Ex. 5(a)

SM = 0: Dy × 48 – 50 × 12 – 35 × 36 = 0
Dy = 38.75 kN
Now consider free body diagram of the cut part of the truss to the right
of joint F as shown in figure Ex. 5(b) and apply conditions of equilibrium i.e.
SM = 0 (Treat ing anticlockwise moment as positive),

Fig. Ex. 5(b)

−1 6
q = tan = 26.57°
12
SMF = 0: FBC × 6 – 35 × 12 + 38.75 × 24 = 0
FBC = –85 kN (Compressive)  (Ans.)
SMC = 0: –FEF × (12 cos 26.57°) + 38.75 × 12 = 0
FEF = 43.33 kN (Tensile)  (Ans.)
SMB = 0: –FCF × (24 sin 26.57°) – 35 × 24 + 38.75 × 36 = 0
FCF = 51.73 kN (Tensile)  (Ans.)
Analysis of Structure  3.19

SUMMARY
• A truss is a structure composed of interconnected members joined at their
ends.
• Truss structure in two dimensions, is known as plane truss.
• A truss which does not collapse under loading is known as rigid or stable
truss.
• For a stable truss, the following relation exists,
n = 2 j – 3
Where, n is the number of members and j is the number of pin joints.
• A truss for which above relationship does not satisfy is known as unstable
truss.
• In truss analysis we determine forces induced in members due to loading.
• When a truss is loaded its members are subjected to either tensile or
compressive forces.
• There are commonly two methods that can be used for the analysis of a
truss.
Method of joints: This method is used to determine forces induced in all
members of a truss
Method of sections: This method is used to determine forces induced
only in certain members of a truss.
• A truss member that supports no forces is known as zero force member.

3.4 Beam
Beam is a slender* structural member subjected to lateral loads that is, line of
action of forces perpendicular to the axis. Internal forces are developed inside
structural member when they are subjected to external forces. Knowledge
of internal forces that structural member must support is important before the
members are designed.
* A structural member is said to be slender when the dimensions of its cross
section are small compared to its length.

3.5 Types of Beams
Beams are generally designated by the way in which they are supported.
Table 3.1 describe different types of beam and their free body diagram, which
shows reactions at support.
3.20  Engineering Mechanics Statics

Table 3.1 Types of Beam

Types of Beam Description Example and Free Body Diagram


Simply A beam with a pin support at
Supported one end (A) and a roller support
Beam at the other (B). Horizontal
and vertical reactions at pin
support and vertical reaction
at roller support.
Cantilever Beam A beam which is fixed at one
end (A) and free at the other.
Horizontal, vertical and
moment reactions at fixed
support.

Beam with an A beam which is simply


Overhang supported at one end (A)
intermediate support/supports
(B) and also projects beyond
the support.

3.6 Types of loading


All types of beam discussed in previous section are subjected to one or more than
one types of loading. Table 3.2 describe different types of loading on the beam.
Table 3.2 Types of Loading on Beam

Types of Description Example


Loading
Concentrated Concentrated load is acts over
Load or Point a very small area. Its intensity
Load is N or kN.

Uniformly Uniformly distributed load


Distributed Load acts uniformly over a finite
area of the beam. Its intensity
is expressed in per unit length
i.e. N/m, kN/m etc. For finding
reactions, this load is assumed
as total load (product of
intensity of distributed load
and spreading distance) acting
at the centre of gravity of
distribution of load.
Analysis of Structure  3.21

Uniformly Uniformly varying load varies


Varying Load linearly over a finite area of
the beam. Its intensity is zero
at one end, and maximum at
other end of distribution which
is expressed in per unit length
i.e. N/m, kN/m etc. For finding
reactions, this load is assumed
as total load (total area of
distribution i.e. area of the
triangle) acting at the centre of
gravity of distribution of load
i.e. centroid of the triangle.
Concentrated Concentrated moment is the
Moment external moment acts at certain
points on the beam.

3.7 Shear Force (V) and Bending Moment (M)


Shear force and bending moment are internal
forces, and internal moment induced within
beam because of loading. To determine the
forces and moments within the beam, we ‘cut’
the beam by plane at an arbitrary cross section
and isolate left-hand part of the beam as a free
body from right-hand part of the beam. Fig. 3.1(a)
As an illustration, consider a cantilever
beam AB loaded by a force P at its free
end as shown in Figure 3.1 (a). We cut the
beam at cross section C at distance x from
fixed end, and isolate left-hand part as free
body as shown in Figure 3.1 (b). To Fig. 3.1(b)
maintain equilibrium of the free body, it is
subjected to some system of forces and moments at the cut
cross section as shown.
The component V perpendicular to the beam’s axis is
called the shear force and the couple M is called the bending
moment. (Because the load P is perpendicular to the axis of Fig. 3.1(c)
the beam, no axial force i.e. force parallel to the beam’s axis
exists at the cross section). Both V and M act in the plane of the beam. The shear
force and bending moment on the part of the beam to the right of the cross section
C are shown in Figure 3.1 (c). Note that V and M are equal in magnitude but
opposite in direction to the internal forces and moment on the free body diagram
as shown in Figure 3.1 (b).
3.22  Engineering Mechanics Statics

3.7.1 Sign Convention
For the sake of consistency, it is required to adopt sign conventions for shear force
and bending moment. The direction of the shear force and bending moment shown
in Figure 3.1 (b) and Figure 3.1 (c) are the established positive directions of
these quantities. A positive shear force V tends to rotate beam element clockwise.
A positive bending moment M tends to bend beam element concave upward
(smiling of the beam).

3.8 Shear Force and Bending Moment Diagrams


Shear force and bending moment diagrams are plots of V and M as functions of
position x. These diagrams help us understand how the shear force and bending
moment change throughout a beam and show locations where these have maximum
and minimum values.

Solved Examples Based on Simply Supported Beam


Example 1. The simply supported beam AB is subjected to concentrated load as
shown in figure Ex. 1. Determine, (a) reactions at supports A and B, and (b) value
of shear force and bending moment at C and D.

Fig. Ex. 1

+
Solution: To determine reactions, apply conditions of equilibrium i.e. →ΣH = 0,
+↑SV = 0 and SM = 0 (Treating anticlockwise moment as positive) to the free
body diagram of the entire beam as shown in figure Ex. 1(a).
SH = 0: Ax = 0
SV = 0: Ay + By – 20 = 0  ...(1)
SMA = 0: By × 6 – 20 × 3 = 0
\ By = 10 kN  (Ans.)
From Eq. (1),
Ay + 10 = 20
Fig. Ex. 1(a)
\ Ay = 10 kN  (Ans.)
Now to determine shear force and bending moment at C, first cut the beam
by a plane at C and then apply equilibrium equations to the free body diagram of
the part of the beam to the left of the plane. Refer figure Ex. 1(b).
Analysis of Structure  3.23

SH = 0: Ax = 0
SV = 0: Ay – V = 0
10 – V = 0
\ V = 10 kN  (Ans.) Fig. Ex. 1(b)
SMC = 0: M – Ay × 2 = 0
M = Ay × 2 = 10 × 2
\ M = 20 kN.m  (Ans.)
Now to determine shear force and bending moment at D, first cut the beam
by a plane at D and then apply equilibrium equations to the free body diagram of
the part of the beam to the left of the plane. Refer figure Ex. 1(c).
SH = 0: Ax = 0
SV = 0: Ay – 20 – V = 0
V = Ay – 20 = 10 – 20
\ V = –10 kN  (Ans.)
SMD = 0: M + 20 × 2 – Ay × 5 = 0
M = Ay × 5 – 20 × 2 Fig. Ex. 1(c)
= 10 × 5 – 20 × 2  \ M = 10 kN.m  (Ans.)
Alternate Approach: Shear force and bending moment at C, also determine
by considering the free body diagram of the part of the beam to the right of the
plane. Refer figure Ex. 1(d).
SV = 0: V – 20 + By = 0
V = 20 – By = 20 – 10
\ V = 10 kN  (Ans.)
SMC = 0: –M – 20 × 1 + By × 4 = 0
M = –20 × 1 + By × 4
= –20 × 1 + 10 × 4 Fig. Ex. 1(d)
\ M = 20 kN.m  (Ans.)
Shear force and bending moment at D, also determine by considering the
free body diagram of the part of the beam to the right of the plane. Refer figure
Ex. 1(e).
SV = 0: V + By = 0
V = – By  \ V = –10 kN  (Ans.)
SMD = 0: –M + By × 1 = 0 Fig. Ex 1(e)
M = By × 1 = 10 × 1  \ M = 10 kN.m  (Ans.)
Example 2. The simply supported beam AB is subjected to two concentrated
load as shown in figure Ex. 2. Determine, (a) reactions at supports A and B, and
(b) draw the shear force and bending moment diagrams for this beam.
3.24  Engineering Mechanics Statics

Fig. Ex. 2

Solution: Part 1: To determine reactions, apply conditions of equilibrium i.e.


+
→ΣH = 0, +↑SV= 0 and SM = 0 (Treating anticlockwise moment as positive) to
the free body diagram of the entire beam as shown in figure Ex. 2(a).
SH = 0: Ax = 0
SV = 0: Ay + By – 20 – 30 = 0  ...(1)
SMA = 0: By × 5 – 30 × 3.5 – 20 × 1.5 = 0
\ By = 27 N  (Ans.)
From Eq. (1),
Ay + By = 50  \ Ay + 27 = 50 Fig. Ex. 2(a)
Ay = 23 N  (Ans.)
Part 2: First cut the beam by a plane at point E between A and C (0 < x < 1.5)
and then apply equilibrium equations to the free body diagram of the part of the
beam to the left of the plane. Refer figure Ex. 2(b).
SV = 0: 23 – V = 0
\ V = 23 N  (Ans.)
SME = 0: M – 23 x = 0
\ M = 23 x Fig. Ex. 2(b)
Substituting x = 0 m,
\ M = 0 N.m  (Ans.)
Substituting x = 1.5 m,
\ M = 34.5 N.m  (Ans.)
Now cut the beam by a plane at point F between C and D (1.5 < x < 3.5) and
then apply equilibrium equations to the free body diagram of the part of the beam
to the left of the plane. Refer figure Ex. 2(c).
SV = 0: 23 – 20 – V = 0
\ V = 3 N  (Ans.)
SMF = 0: M + 20 (x – 1.5) – 23 x = 0
\ M = 23 x - 20 (x – 1.5)
Substituting x = 3.5 m, Fig. Ex. 2(c)
M = 23 × 3.5 – 20 (3.5 – 1.5)
\ M = 40.5 N.m  (Ans.)
Analysis of Structure  3.25

Next cut the beam by a plane at point G between D and B (3.5 < x < 5) and
then apply equilibrium equations to the free body diagram of the part of the beam
to the left of the plane. Refer figure Ex. 2(d).
SV = 0: 23 – 20 – 30 – V = 0
\ V = –27 N  (Ans.)
SMG = 0:
M + 30 (x – 3.5) + 20 (x – 1.5) – 23 x = 0
Fig. Ex. 2(d)
\ M = 23 x – 20 (x – 1.5) – 30 (x – 3.5)
Substituting x = 5 m,
M = 23 × 5 – 20 (5 – 1.5) – 30 (5 – 3.5)  \ M = 0 N.m (Ans.)

Fig. Ex. 2(f)

The shear force and bending moment diagrams are shown in figure Ex. 2(e)
and figure Ex. 2(f). These are the plots of V and M as a function of position x for
different segments of beam derived in Part 2. Note that for each concentrated
force, the shear force diagram jumps by an amount equal to the force, and remain
constant between two concentrated forces. The bending moment diagram is an
inclined straight line between two concentrated forces.
3.26  Engineering Mechanics Statics

Example 3. The simply supported beam AB is subjected to a concentrated load


and a moment as shown in figure Ex. 3. Determine, (a) reactions at supports A
and B, and (b) draw the shear force and bending moment diagrams for this beam.

Fig. Ex. 3

Solution: Part 1
+
To determine reactions, apply conditions of equilibrium i.e. →ΣH = 0,
+↑SV = 0 and SM = 0 (Treating anticlockwise moment as positive) to the free
body diagram of the entire beam as shown in figure Ex. 3(a).
SH = 0: Ax = 0
SV = 0: Ay + By – 25 = 0  ...(1)
SMA = 0: By × 4 – 25 × 1 – 35 = 0
\ By = 15 N  (Ans.)
From Eq. (1),
Fig. Ex. 3(a)
Ay + By = 25
\ Ay + 15 = 25
\ Ay = 10 N  (Ans.)
Part 2: First cut the beam by a plane at point E between
A and C (0 < x < 1) and then apply equilibrium equations
to the free body diagram of the part of the beam to the left Fig. Ex. 3(b)
of the plane. Refer figure Ex. 3(b).
SV = 0: 10 – V = 0
\ V = 10 N  (Ans.)
SME = 0: M – 10 x = 0
\ M =10 x
Substituting x = 0 m,
\ M = 0 N.m  (Ans.)
Substituting x = 1 m,
\ M = 10 N.m  (Ans.) Fig. Ex. 3(c)
Analysis of Structure  3.27

Now cut the beam by a plane at point F


between C and D (1 < x < 2) and then apply
equilibrium equations to the free body diagram
of the part of the beam to the left of the plane.
Refer figure Ex. 3(c).

Fig. Ex. 3(d)

Fig. Ex. 3(f)

SV = 0: 10 – 25 – V = 0  \ V = –15 N  (Ans.)
SMF = 0: M + 25 (x – 1) – 10 x = 0  \ M = 10 x – 25 (x – 1)
Substituting x = 2 m,
M = 10 × 2 – 25 (2 – 1)  \ M = –5 N.m  (Ans.)
Next cut the beam by a plane at point G between D and B (2 < x < 4) and then
apply equilibrium equations to the free body diagram of the part of the beam to
the left of the plane. Refer figure Ex. 3(d).
3.28  Engineering Mechanics Statics

SV = 0: 10 – 25 – V = 0  \ V = –15 N  (Ans.)
SMG = 0: M – 35 + 25 (x – 1) – 10 x = 0
\ M = 35 – 25 (x – 1) + 10 x
Substituting x = 2 m,
M = 35 – 25 (2 – 1) + 10 × 2  \ M = 30 N.m  (Ans.)
Substituting x = 4 m,
M = 35 – 25 (4 – 1) + 10 × 4  \ M = 0 N.m  (Ans.)
The shear force and bending moment diagrams are shown in figure Ex. 3(e)
and figure Ex. 3(f). These are the plots of V and M as a function of position x
for different segments of beam derived in Part 2. Note that for a concentrated
moment, the bending moment diagram jumps by an amount equal to the moment.
Example 4. The simply supported beam AB is subjected to uniformly distributed
load as shown in figure Ex. 4. Determine, (a) reactions at supports A and B, and
(b) draw the shear force and bending moment diagrams for this beam.

Fig. Ex. 4

Solution: Part 1: To determine reactions, apply conditions of equilibrium i.e.


+
→ΣH = 0, +↑SV = 0 and SM = 0 (Treating anticlockwise moment as positive) to
the free body diagram of the entire beam as shown in figure Ex. 4(a).
As discussed in section 3.6, distributed load
is assumed as total load (product of intensity of
distributed load and spreading distance) acting
at the centre of gravity of distribution of load.
SH = 0: Ax = 0
SV = 0: Ay + By – 10 × 5 = 0  ...(1) Fig. Ex. 4(a)
SMA = 0: By × 5 – 10 × 5 × 2.5 = 0
\ By = 25 N  (Ans.)
From Eq. (1),
Ay + By = 50
\ Ay + 25 = 50
Fig. Ex. 4(b)
\ Ay = 25 N  (Ans.)
Part 2: Now cut the beam by a plane at point C between A and B (0 < x < 5) and
then apply equilibrium equations to the free body diagram of the part of the beam
Analysis of Structure  3.29

to the left of the plane. Refer figure Ex. 4(b).

Fig. Ex. 4(d)

SV = 0: 25 – 10x – V = 0
\ V = 25 – 10x ...(2)
Substituting x = 0 m,
\ V = 25 N  (Ans.)
Substituting x = 5 m,
\ V = –25 N  (Ans.)
The location of the section on beam where shear force is zero is found by
substituting V = 0 in Eq. (2),
x = 2.5 m
 x
Now, SMC = 0 : M + 10 x   – 25 x = 0
2
 x
\ M = 25 x – 10 x  
2
Substituting x = 0 m,  \ M = 0 N.m  (Ans.)
Substituting x = 5 m,  \ M = 0 N.m  (Ans.)
Substituting x = 2.5 m,
3.30  Engineering Mechanics Statics

 2.5 
M = 25 × 2.5 – 10 × 2.5     \ M = 18.75 N.m  (Ans.)
 2 
The shear force and bending moment diagrams are shown in figure Ex. 4(c)
and figure Ex. 4(d). These are the plots of V and M as a function of position x
for different segments of beam derived in Part 2. Note that for distributed load,
the shear force diagram is an inclined straight line between start and end of
distribution. The bending moment diagram is a parabola between start and end of
distribution. The maximum bending moment occurs where the shear force is zero.
Example 5. The simply supported beam AB is subjected to uniformly varying
load as shown in figure Ex. 5. Determine, (a) reactions at supports A and B, and
(b) draw the shear force and bending moment diagrams for this beam.

Fig. Ex.5

Solution: Part 1
+
To determine reactions, apply conditions of equilibrium i.e. →ΣH = 0,
+↑SV = 0 and SM = 0 (Treating anticlockwise moment as positive) to the free
body diagram of the entire beam as shown in
figure Ex. 5(a).
As discussed in section 3.6, uniformly
varying load is assumed as total load (total
area of distribution i.e. area of the triangle)
acting at the centre of gravity of distribution
of load i.e. centroid of the triangle.
Fig. Ex. 5(a)
SH = 0: Ax = 0
1
SV = 0 : Ay + By – × 3 × 24 = 0 ...(1)
2
1 2
SMA = 0 : By × 3 – × 3 × 24 × × 3 = 0
2 3
\ By = 24 N  (Ans.)

From Eq. (1),


Ay + By = 36
\ Ay + 24 = 36
\ Ay = 12 N  (Ans.) Fig. Ex. 5(b)
Analysis of Structure  3.31

Part 2: Now cut the beam by a plane at point C between A and B (0 < x < 3) and
then apply equilibrium equations to the free body diagram of the part of the beam
to the left of the plane. Refer figure Ex. 5(b).
w 24
Let w be the intensity at section C, we have from similar triangle, = or w = 8 x
x 3
1
SV = 0 : 12 – × x × 8 x – V = 0
2
\ V = 12 – 4x2 ...(2)
Substituting x = 0 m,  \ V = 12 N  (Ans.)
Substituting x = 3 m,  \ V = –24 N  (Ans.)
The location of the section on beam where shear force is zero is found by
substituting V = 0 in Eq. (2),

Fig. Ex. 5(d)

\ x = 1.73 m
 x
SMC = 0 : M + 4 x 2 ×   – 12 x = 0
3
2 x
\ M = 12 x – 4 x  
3
Substituting x = 0 m,  \ M = 0 N.m  (Ans.)
Substituting x = 3 m,  \ M = 0 N.m  (Ans.)
Substituting x = 1.73 m,
3.32  Engineering Mechanics Statics

 1.73 
\  M = 12 × 1.73 – 4 × 1.732 ×    \ M = 13.86 N.m  (Ans.)
 3 

The shear force and bending moment diagrams are shown in figure Ex. 5(c)
and figure Ex. 5(d). These are the plots of V and M as a function of position x for
different segments of beam derived in Part 2. Note that for uniformly varying
load, the shear force diagram is a parabola between start and end of distribution.
The bending moment diagram is a third-degree polynomial between start and end
of distribution. The maximum bending moment occurs where the shear force is
zero.
Example 6. The simply supported beam AB is subjected to uniformly distributed
load and concentrated load as shown in figure Ex. 6. Determine, (a) reactions at
supports A and B, and (b) draw the shear force and bending moment diagrams
for this beam.

Fig. Ex. 6

Solution: Part 1: To determine reactions, apply conditions of equilibrium i.e.


+
→ΣH = 0, +↑SV = 0 and SM = 0 (Treating
anticlockwise moment as positive) to the free
body diagram of the entire beam as shown in
figure Ex. 6(a).
As discussed in section 3.6, distributed load
is assumed as total load (product of intensity of Fig. Ex. 6(a)
distributed load and spreading distance) acting
at the centre of gravity of distribution of load.
SH = 0: Ax = 0
SV = 0: Ay + By – 30 – 15 × 2 = 0 ...(1)
SMA = 0: By × 5 – 30 × 2 – 15 × 2 × 4 = 0
Fig. Ex. 6(b)
\ By = 36 N  (Ans.)
From Eq. (1),
Ay + By = 60  \ Ay + 36 = 60
\ Ay = 24 N  (Ans.)
Part 2: First cut the beam by a plane at point E
between A and C (0 < x < 2) and then apply equilibrium
equations to the free body diagram of the part of the
beam to the left of the plane. Refer figure Ex. 6(b). Fig. Ex. 6(c)
Analysis of Structure  3.33

SV = 0: 24 – V = 0  \ V = 24 N  (Ans.)
SME = 0: M – 24x = 0  \ M = 24x
Substituting x = 0 m,  \ M = 0 N.m  (Ans.)
Substituting x = 2 m,  \ M = 48 N.m  (Ans.)
Now cut the beam by a plane at point
F between C and D (2 < x < 3) and then
apply equilibrium equations to the free
body diagram of the part of the beam to
the left of the plane. Refer figure Ex. 6(c).
SV = 0: 24 – 30 – V = 0
\ V = –6 N  (Ans.) Fig. Ex. 6(d)
SMF = 0: M + 30 (x – 2) – 24x = 0
\ M = 24x – 30 (x – 2)
Substituting x = 3 m,  \ M = 42 N.m  (Ans.)

Fig. Ex. 6(e)

Fig. Ex. 6(f)


3.34  Engineering Mechanics Statics

Next cut the beam by a plane at point G between D and B (3 < x < 5) and then
apply equilibrium equations to the free body diagram of the part of the beam to
the left of the plane. Refer figure Ex. 6(d).
SV = 0: 24 – 30 – 15 (x – 3) – V = 0
\ V = 24 – 30 – 15 (x – 3)
Substituting x = 5 m,  \ V = –36 N  (Ans.)
 x – 3
SMG = 0 : M + 15( x – 3)   + 30( x – 2) – 24 x = 0
 2 
 x – 3
\ M = 24 x – 30( x – 2) – 15( x – 3)  
 2 
Substituting x = 5 m,  \ M = 0 N.m  (Ans.)
The shear force and bending moment diagrams are shown in figure Ex. 6(e)
and figure Ex. 6(f). These are the plots of V and M as a function of position x for
different segments of beam derived in Part 2.

Solved Examples Based on Cantilever Beam


Example 1. A cantilever beam AB is subjected to concentrated load at free end
B as shown in figure Ex. 1. Determine, (a) reactions at fixed support A, and (b)
draw the shear force and bending moment diagrams for this beam.

Fig. Ex. 1

Solution: Part 1: To determine reactions,


+
apply conditions of equilibrium i.e. →ΣH = 0,
+↑SV= 0 and SM = 0 (Treating anticlockwise
moment as positive) to the free body diagram
of the entire beam as shown in figure Ex. 1(a). Fig. Ex. 1(a)
SH = 0: Ax = 0
SV = 0: Ay – 10 = 0
\ Ay = 10 kN  (Ans.)
SMA = 0: CA – 10 × 4 = 0
\ CA = 40 kN.m  (Ans.) Fig. Ex. 1(b)
Analysis of Structure  3.35

Part 2: Now cut the beam by a plane at point C between A and B (0 < x < 4) and
then apply equilibrium equations to the free body diagram of the part of the beam
to the left of the plane. Refer figure Ex. 1(b).

Fig. Ex. 1(d)


SV = 0: 10 – V = 0  \ V = 10 kN  (Ans.)
SMC = 0: M + 40 – 10 x = 0
\ M =10 x – 40
Substituting x = 0 m,  \ M = – 40 kN.m  (Ans.)
Substituting x = 4 m,  \ M = 0 N.m  (Ans.)
The shear force and bending moment diagrams are shown in figure Ex. 1(c)
and figure Ex. 1(d). These are the plots of V and M as a function of position x for
different segments of beam derived in Part 2.
Example 2. A cantilever beam AB is subjected to two concentrated load as
shown in figure Ex. 2. Determine, (a) reactions at fixed support A, and (b) draw
the shear force and bending moment diagrams for this beam.

Fig. Ex. 2
3.36  Engineering Mechanics Statics

Solution: Part 1: To determine reactions, apply conditions of equilibrium i.e.


+
→ΣH = 0, +↑SV = 0 and SM = 0 (Treating anticlockwise moment as positive) to
the free body diagram of the entire beam as shown in figure Ex. 2(a).
SH = 0: Ax = 0
SV = 0: Ay – 5 – 8 = 0
\ Ay = 13 kN  (Ans.)
SMA = 0: CA – 5 × 2 – 8 × 4 = 0
\ CA = 42 kN.m  (Ans.) Fig. Ex. 2(a)
Part 2: First cut the beam by a plane at point E between A and C (0 < x < 2) and
then apply equilibrium equations to the free body diagram of the part of the beam
to the left of the plane. Refer figure Ex. 2(b).
SV = 0: 13 – V = 0  \ V = 13 kN  (Ans.)
SME = 0: M + 42 – 13x = 0
\ M = 13x – 42
Substituting x = 0 m,
\ M = – 42 kN.m  (Ans.)
Substituting x = 2 m,
\ M = – 16 kN.m  (Ans.)


Fig. Ex. 2(b) Fig. Ex. 2(c)

Now cut the beam by a plane at point F between C and D (2 < x < 4) and then
apply equilibrium equations to the free body diagram of the part of the beam to
the left of the plane. Refer figure Ex. 2(c).

Fig. Ex. 2(d)


Analysis of Structure  3.37

Fig. Ex. 2(f)

SV = 0: 13 – 5 – V = 0  \ V = 8 kN  (Ans.)
SMF = 0: M + 42 + 5 (x – 2) – 13 x = 0
=13 x – 42 - 5 (x – 2)
\ M
Substituting x = 4 m,
\ M = 0 kN.m  (Ans.)
Next cut the beam by a plane at point G between D and B (4 < x < 5) and then
apply equilibrium equations to the free body diagram of the part of the beam to
the left of the plane. Refer figure Ex. 2(d).
SV = 0: 13 – 5 – 8 – V = 0  \ V = 0 kN  (Ans.)
SMG = 0: M + 42 + 8 (x – 4) + 5 (x – 2) – 13 x = 0
\ M =13 x – 42 – 8 (x – 4) – 5 (x – 2)
Substituting x = 5 m,
\ M = 0 kN.m  (Ans.)
The shear force and bending moment diagrams are shown in figure Ex. 2(e)
and figure Ex. 2(f). These are the plots of V and M as a function of position x for
different segments of beam derived in Part 2.
3.38  Engineering Mechanics Statics

Example 3. A cantilever beam AB is subjected to uniformly distributed load as


shown in figure Ex. 3. Determine, (a) reactions at supports A, and (b) draw the
shear force and bending moment diagrams for this beam.

Fig. Ex. 3

Solution: Part 1: To determine reactions, apply conditions of equilibrium i.e.


+
→ΣH = 0, +↑SV = 0 and SM = 0 (Treating anticlockwise moment as positive) to
the free body diagram of the entire beam as shown in figure Ex. 3(a).

Fig. Ex. 3(a)

As discussed in section 3.6, distributed load is assumed as total load (product


of intensity of distributed load and spreading distance) acting at the centre of
gravity of distribution of load.
SH = 0: Ax = 0
SV = 0: Ay – 20 × 5 = 0
\ Ay = 100 N  (Ans.)
SMA = 0: CA – 20 × 5 × 2.5 = 0
\ CA = 250 N.m  (Ans.)
Part 2: Now cut the beam by a plane at point C between A and B (0 < x < 5) and
then apply equilibrium equations to the free body diagram of the part of the beam
to the left of the plane. Refer figure Ex. 3(b).
SV = 0: 100 – 20 x – V = 0
\ V = 100 – 20 x
Substituting x = 0 m,
\ V = 100 N  (Ans.)
Fig. Ex. 3(b)
Analysis of Structure  3.39

Substituting x = 5 m,
\ V = 0 N  (Ans.)
 x
SMC = 0 : M + 250 + 20 x   – 100 x = 0
2
 x
\ M = 100 x – 20 x   – 250
 2
Substituting x = 0 m,
\ M = –250 N.m  (Ans.)
Substituting x = 5 m,
\ M = 0 N.m  (Ans.)

Fig. Ex. 3(d)

The shear force and bending moment diagrams are shown in figure Ex. 3(c)
and figure Ex. 3(d). These are the plots of V and M as a function of position x for
different segments of beam derived in Part 2.
3.40  Engineering Mechanics Statics

Example 4. A cantilever beam AB is subjected to uniformly varying load as


shown in figure Ex. 4. Determine, (a) reactions at supports A, and (b) draw the
shear force and bending moment diagrams for this beam.

Fig. Ex.4

Solution: Part 1
+
To determine reactions, apply conditions of equilibrium i.e. →ΣH = 0, +↑SV = 0
and SM = 0 (Treating anticlockwise moment as positive) to the free body diagram
of the entire beam as shown in figure Ex. 4(a).

Fig. Ex. 4(a)

As discussed in section 3.6, uniformly varying load is assumed as total load


(total area of distribution i.e. area of the triangle) acting at the centre of gravity of
distribution of load i.e. centroid of the triangle.
SH = 0: Ax = 0
1
SV = 0: Ay – × 12 × 60 = 0
2
\ Ay = 360 N  (Ans.)

1 1 Fig. Ex. 4(b)


SMA = 0: C A – × 12 × 60 × × 12 = 0
2 3
\ CA = 1440 N.m  (Ans.)
Analysis of Structure  3.41

Fig. Ex. 4(d)

Part 2: Now cut the beam by a plane at point C between A and B and then apply
equilibrium equations to the free body diagram of the part of the beam to the right
of the plane. Refer figure Ex. 4(b). Here we measure x from B towards C.
1
SV = 0: V – × 5 x × x = 0
2
1
\ V = × 5x × x
2
Substituting x = 0 m,
\ V = 0 N  (Ans.)
Substituting x = 12 m,
\ V = 360 N  (Ans.)
3.42  Engineering Mechanics Statics

1  x
SMC = 0: – M – × 5x × x   = 0
2 3
1  x
\ M = – × 5 x × x  
2 3
Substituting x = 0 m,
\ M = 0  (Ans.)
Substituting x = 12 m,
\ M = –1440 N.m  (Ans.)
The shear force and bending moment diagrams are shown in figure Ex. 4(c)
and figure Ex. 4(d). These are the plots of V and M as a function of position x for
different segments of beam derived in Part 2.
Example 5. A cantilever beam AB is subjected to uniformly distributed load and
a concentrated load as shown in figure Ex. 5. Determine, (a) reactions at supports
A, and (b) draw the shear force and bending moment diagrams for this beam.

Fig. Ex. 5

Solution: Part 1: To determine reactions, apply conditions of equilibrium i.e.


+
→ΣH = 0, +↑SV = 0 and SM = 0 (Treating anticlockwise moment as positive) to
the free body diagram of the entire beam as shown in figure Ex. 5(a).
As discussed in section 3.6, distributed
load is assumed as total load (product of
intensity of distributed load and spreading
distance) acting at the centre of gravity of
distribution of load.
SH = 0: Ax = 0
SV = 0: Ay – 10 × 3 – 5 = 0
\ Ay = 35 kN  (Ans.)
Fig. Ex. 5(a)
SMA = 0: CA – 10 × 3 × 1.5 – 5 × 6 = 0
\ CA = 75 kN.m  (Ans.)
Part 2: First cut the beam by a plane at point D between A and C (0 < x < 3) and
then apply equilibrium equations to the free body diagram of the part of the beam
to the left of the plane. Refer figure Ex. 5(b).
SV = 0: 35 – 10 x – V = 0 \ V = 35 – 10 x
Analysis of Structure  3.43

Substituting x = 0 m,
\ V = 35 kN  (Ans.)
Substituting x = 3 m,
\ V = 5 kN  (Ans.)
 x
SMD = 0: M + 75 + 10 x   – 35 x = 0
2 Fig. Ex. 5(b)
x
 
\ M = 35 x – 10 x   – 75
2
Substituting x = 0 m,
\ M = –75 kN.m  (Ans.)
Substituting x = 3 m,
\ M = –15 kN.m  (Ans.)
Now cut the beam by a plane at point E between A and B (3 < x < 6) and then
apply equilibrium equations to the free body diagram of the part of the beam to
the left of the plane. Refer figure Ex. 5(c).
SV = 0: 35 – 10 × 3 – V = 0  \ V = 5 kN (Ans.)
SME = 0:
M + 75 + 10 × 3 × (x – 1.5) – 35 x = 0
\ M = 35x – 75 – 10 × 3 × (x – 1.5)
Substituting x = 6 m,
\ M = 0 kN.m  (Ans.)
Fig. Ex. 5(c)

Fig. Ex. 5(e)


3.44  Engineering Mechanics Statics

The shear force and bending moment diagrams are shown in figure Ex. 5(d)
and figure Ex. 5(e). These are the plots of V and M as a function of position x for
different segments of beam derived in Part 2.

Solved Examples Based on Overhang Beam


Example 1. A overhanging beam AC is subjected to a concentrated load as
shown in figure Ex. 1. Determine, (a) reactions at supports A and B, and (b) draw
the shear force and bending moment diagrams for this beam.

Fig. Ex. 1

Solution: Part 1: To determine reactions, apply conditions of equilibrium i.e.


+
→ΣH = 0, +↑SV = 0 and SM = 0 (Treating anticlockwise moment as positive)
to the free body diagram of the entire beam as shown in figure Ex. 1(a).
SH = 0: Ax = 0
SV = 0: Ay + By – 10 = 0  ...(1)
SMA = 0: By × 3 – 10 × 5 = 0
\ By = 16.67 kN  (Ans.)
From Eq. (1),
Ay + By = 10 Fig. Ex. 1(a)
\ Ay + 16.67 = 10
\ Ay = –6.67 kN  (Ans.)
Here ‘–ve’ sign indicate that, reaction at A will act in
negative y direction.
Part 2: First cut the beam by a plane at point D between
A and B (0 < x < 3) and then apply equilibrium equations
to the free body diagram of the part of the beam to the left
Fig. Ex. 1(b)
of the plane. Refer figure Ex. 1(b).

Fig. Ex. 1(c)


Analysis of Structure  3.45

SV = 0: –6.67 – V = 0  \ V = –6.67 kN  (Ans.)


SMD = 0: M + 6.67 x = 0  \ M = –6.67 x
Substituting x = 0 m,
\ M = 0 kN.m  (Ans.)
Substituting x = 3 m,
\ M = –20 kN.m  (Ans.)

Fig. Ex. 1(e)

Now cut the beam by a plane at point E between B and C (3 < x < 6) and then
apply equilibrium equations to the free body diagram of the part of the beam to
the left of the plane. Refer figure Ex. 1(c).
SV = 0: –6.67 + 16.67 – V = 0
\ V = 10 kN  (Ans.)
SME = 0: M + 6.67 x – 16.67 (x – 3) = 0
\ M = 16.67 (x – 3) – 6.67 x
Substituting x = 5 m,
M = 16.67 (5 – 3) – 6.67 × 5
\ M = 0 kN.m  (Ans.)
The shear force and bending moment diagrams are shown in figure Ex. 1(d)
and figure Ex. 1(e). These are the plots of V and M as a function of position x for
different segments of beam derived in Part 2.
3.46  Engineering Mechanics Statics

Example 2. A overhanging beam AC is subjected to a concentrated load and


uniformly distributed load as shown in figure Ex.2. Determine, (a) reactions at
supports A and B, and (b) draw the shear force and bending moment diagrams
for this beam.

Fig. Ex.2

Solution: Part 1: To determine reactions,


+
apply conditions of equilibrium i.e. →ΣH
= 0, +↑SV = 0 and SM = 0 (Treating
anticlockwise moment as positive) to the
free body diagram of the entire beam as
shown in figure Ex. 2(a).
Fig. Ex. 2(a)
SH = 0: Ax = 0
SV = 0: Ay + By – 20 × 4 – 10 = 0 ...(1)
SMA = 0: By × 4 – 20 × 4 × 2 – 10 × 6 = 0
\ By = 55 N  (Ans.)
From Eq. (1),
Ay + By = 90
\ Ay + 55 = 90
\ Ay = 35 N  (Ans.)
Part 2: First cut the beam by a plane at point D between A and B (0 < x < 4) and
then apply equilibrium equations to the free body diagram of the part of the beam
to the left of the plane. Refer figure Ex. 2(b).
SV = 0: 35 – 20x – V = 0
\ V = 35 – 20x ...(2)
Substituting x = 0 m,
\ V = 35 N  (Ans.)
Substituting x = 4 m, Fig. Ex. 2(b)
\ V = –45 N  (Ans.)
The location of the section on beam where shear force is zero is found by
substituting V= 0 in Eq. (2),
\ x = 1.75 m
Analysis of Structure  3.47

 x
SMD = 0: M + 20 x   – 35 x = 0
2
 x
\ M = 35 x – 20 x  
2
Substituting x = 0 m,
\ M = 0 N.m  (Ans.)
Substituting x = 4 m,
\ M = –20 N.m  (Ans.)
Fig. Ex. 2(c)
Substituting x = 1.75 m,
\ M = 30.625 N.m  (Ans.)
Now cut the beam by a plane at point E between B and C (4 < x < 6) and then
apply equilibrium equations to the free body diagram of the part of the beam to
the left of the plane. Refer figure Ex. 2(c).

Fig. Ex. 2(e)


3.48  Engineering Mechanics Statics

SV = 0: 35 – 20 × 4 + 55 – V = 0  \ V = 10 N  (Ans.)
SME = 0: M – 55 (x – 4) + 20 × 4 (x – 2) – 35 x = 0
\ M = 35 x – 20 × 4 (x – 2) + 55 (x – 4)
Substituting x = 6 m,
\ M = 0 kN.m  (Ans.)
The shear force and bending moment diagrams are shown in figure Ex. 2(d)
and figure Ex. 2(e). These are the plots of V and M as a function of position x for
different segments of beam derived in Part 2.

SUMMARY
• Beam is a slender structural member subjected to lateral loads.
• Simply supported beam, Cantilever, beam with overhang, Fixed beam etc
are different types of beam.
• Shear Force (V) and Bending Moment (M) are internal forces and moment
induced in a beam.
• Internal forces are forces and moments develop within beam due to
external loading.
• A positive shear force V tends to rotate beam element clockwise. A positive
bending moment M tends to bend beam element concave upward.
• Shear force and bending moment diagrams are plots of V and M as
functions of position x.

Problem

Determine shear force and bending moment equations for different sections of the beam
and draw complete shear force and bending moment diagrams for the beam shown in
figure Prob. 1 to figure Prob. 8.

Fig. Prob. 1 Fig. Prob. 2

Fig. Prob. 3 Fig. Prob. 4


Analysis of Structure  3.49

Fig. Prob. 5 Fig. Prob. 6

Fig. Prob. 7
Fig. Prob. 8
Unit 4

In this unit, we discuss concept of centroid and moment of inertia.


In first part of this unit, we focus on center of gravity and centroid
and show how to determine location of centroid for composite areas.
In second part of this unit, we focus on area moment of inertia that
measures how an area is distributed about particular axes. These
concepts are useful in distributed force system. We show how to
determine area moment of inertia of composite areas.
Centroid and Moment of Inertia
4.1 Introduction
The force of attraction exerted by the earth on body is known as force of gravity
or weight of the body, denoted by W. This force
on free body diagram is represented at the center The weight of the body
of gravity of the body. Actually a rigid body is is distributed force.
collection of particles and earth exerts a force on
each of the particles. Thus weight of the body is distributed over the entire volume
of the object, but it is represented by single equivalent force.
It is convenient to find resultant of distributed force system so that all the
forces may be assumed to be concentrated on a point. Distributed quantities like
length, area, mass and weight may be assumed to be concentrated on a point for
the ease in the analysis. This point for length and area is called as Centroid, for
mass, is called as Center of Mass and for weight, is called as Center of Gravity.

Center of Gravity
Center of gravity of a body is a point through
which the resultant of the gravity forces act C.G. is the average
irrespective of the orientation of the body. For position of a distribution
symmetrical object C.G. would be in the exact of weight.
center of object. However for non-symmetrical
objects C. G. would be in any number of positions, depending on weight
distribution. The location of the C.G. remains fixed as long as the body does not
change shape. If an object’s shape changes, the location of the C.G. also changes.

Center of Mass
Center of mass of a body is a point where the entire mass of a body is concentrated.
In a uniform gravitational field the center of gravity coincides with center of mass.
The center of mass may lie outside the object.

Centroid
Centroid is geometrical center of area or shape. It is also defined as a point where
the whole area of the shape is assumed to be concentrated.

4.2 Determination of Centroid
To determine mathematically the coordinates of centroid, we apply principle of
moment.
Consider the area A shown in figure 1. Let area ‘A’ is composed of a number
of small areas a1, a2, a3, a4, .... etc.
4.4  Engineering Mechanics Statics

\ A = a1 + a2 + a3 + a4 + ... + an
Let, x1 = the distance of the centroid of the area a1 from y axis
x2 = the distance of the centroid of the area a2 from y axis
x3 = the distance of the centroid of the area a3 from y axis
x4 = the distance of the centroid of the area a4 from y axis
The moments of all small areas about y axis is
= a1 x1 + a2 x2 + a3 x3 + a4 x4 + ...
Let G be the centroid of the total area whose distance from the y axis is x
Y
a a4
a2 3
a1 AA
G
x1
x2
x3
x4
x
X

Fig. 1

Then the moment of total area about y axis = Ax


The moment of all small areas about the y axis must be equal to the moment
of total area about the same axis.
a1 x1 + a2 x2+ a3 x3 + a4 x4 + ... = Ax
a1 x1 + a2 x2 + a3 x3 + a4 x4 + ....
x =
A
In the same way if we apply principle of moment about the x axis, then

y = a1 y1 + a2 y2 + a3 y3 + a4 y4 + ....
A
\ Using following expressions centroid of any given irregular shape may
be found.
n

x =
∑ i=1 xi Ai ...(Eq. 4.1)
n
∑ i=1 Ai
n

y =
∑ i=1 yi Ai ...(Eq. 4.2)
n
∑ i=1 Ai
By taking the limits of Eq. 4.1 and Eq. 4.2, as Ai → 0, the summations become
integrals

x =
∫ x dA and y = ∫ y dA ...(Eq. 4.3)
∫ dA ∫ dA
Centroid and Moment of Inertia  4.5

Here x and y are the location of the centroid of area element dA. ∫ x dA
is known as first moment of area about the y axis and ∫ y dA is known as first
moment of area about the x axis.

4.3 Centroid of Wire
Centroid of a linear element like when wire bent into some shape may be found
using expression;
l x + l x + .... + ln xn ∑ lx
X = 1 1 2 2 = ...(Eq. 4.4)
l1 + l2 + ..... + ln ∑l
l1 y1 + l2 y2 + ..... + ln yn ∑ ly
Y = = ...(Eq. 4.5)
l1 + l2 + ..... + ln ∑l
where l1, l2, ..., ln are the lengths of such regular line elements for which centroid
coordinates (x1, y1), (x2, y2),..., (xn, yn) are known.

4.4  Procedure to Determine Coordinates of


Centroid of Composite Areas
Many engineering objects can be considered as composite bodies made up of
connected simple shapes like a rectangle, triangle, circle and semicircle. Knowing
the location of the centroid of the simple shapes, we can easily determine the
location of the centroid of complex composite areas.
1. Divide the given area into different simple shapes having known
centrodial distance.
2. Establish the coordinate axes and determine the coordinates x , y of the
centroid of each part.
3. Determine x , y by applying the center of gravity equations.
4. If section is symmetric about its own x axis we can find y coordinate of
centroid directly without any calculations as it lie on x axis. If section is
symmetric about its own y axis we can find x coordinate directly without
any calculations as it lie on y axis.

4.5 Centroid of Common Areas Using Method of


Integration
1. Centroid of a triangular area: Consider a triangle shown in figure 2. We
have to determine the x coordinate of centroid of this triangle.
Now consider area dA in the form of vertical strip of width dx as shown
h
in figure 3. Here height of the strip is x.
b
4.6  Engineering Mechanics Statics

y y

dA
h h

x x
b        x dx

Fig. 2                 Fig. 3

Now using Eq. 4.3,


b h
∫ x dA ∫o b
x dx
x = =
∫ dA ∫ dA
2
Solving, x = b
3
1
Similarly considering a horizontal strip we can determine y = h.
3
Here it is important to understand the orientation of the area. If the
orientation of triangular area will change, the coordinates of centroid will
also change.
2. Centroid of Semicircular Area: Consider a semicircular lamina with
radius r as shown in figure 4. Here semicircular area is symmetric about
y-axis, therefore x = 0.
Select an elementary area in form of strip as shown in figure 4.
The strip can be considered as a triangle whose base is rdq and altitude
is r.
The location of the centroid of the elementary sector is A.

Fig. 4
2r
\ Distance O A =
(Centroid of triangle)
3
2
\ y coordinate the centroid of element is y =
r sin θ
3
Centroid and Moment of Inertia  4.7

1 1
Area of the strip, dA = × r × rd θ = r 2 d θ
2 2
Using Eq. 4.3,
π π 2 1 r3 π
∫0 3 r sin θ ⋅ 2 r ∫0 sin θd θ
2

y =
∫0 y dA
= = 3 2
π π 1 r π
∫0 dA ∫0 2 r
2

2 ∫0

r3 r3
[ − cos θ]0π ×2
3 = 3
=
r2 π r2
[θ]0 π
3 2
4r
\ y =

4.6 Centroid and Area of Some Common Plane Areas


Centroid and area of some common plane areas are shown in table 4.1.
Table 4.1 Centroid and Area

Shape Area Centroid

b
h x =
y 2
b×d h
x y =
b 2

b
x =
1 3
bh
2
y = h
3

y pr2 x =0 y =0

x
4.8  Engineering Mechanics Statics

r π⋅ r2 4r
x = r, y =
y 2 3π
x
d

π⋅ r2 4r 4r
x = y =
4 3π 3π

Solved Examples Based on Centroid


Example 1. Determine coordinates of centroid of the shaded area shown in
figure Ex. 1. All dimensions are in mm.
Y
200

80

40

40
20
X
300
Fig. Ex. 1
Solution: First divide the given area into three parts i.e. rectangle, semicircle and
triangle.
Rectangle: a1 = 300 × 140 = 42000 mm2
300 140
x1 = = 150 mm, y1 = = 70 mm
2 2
πr 2 π × 202
Semicircle: a2 = = = 628.31 mm 2
2 2
4r
x2 = 300 – = 291.5 mm

40
y2 = 20 + = 40 mm
2
Centroid and Moment of Inertia  4.9

1 1
Triangle: a2 = b × h = × 100 × 100 = 5000 mm 2
2 2
b 100
x3 = = = 33.33 mm
3 3
h 100
y3 = 140 – = 140 – = 106.67 mm
3 3
Now coordinates of the centroid are
a x − a2 x2 – a3 x3
x = 1 1
a1 – a2 – a3
42000 ×150 − 628.31× 291.5 − 5000 × 33.33
= = 163.59 mm   (Ans.)
42000 − 628.31 − 5000
a1 y1 − a2 y2 – a3 y3
y =
a1 – a2 – a3
42000 × 70 − 628.31× 40 − 5000 ×106.67
= = 65.47 mm
42000 − 628.31 − 5000
Example 2. Determine coordinates of centroid of the shaded area shown in
figure Ex. 2. All dimensions are in mm.

Fig. Ex. 2

Solution: First divide the given area into two parts i.e. rectangle 1 and rectangle 2.
Rectangle 1: a1 = 30 × 160 = 4800 mm2
30
x1 = = 15 mm
2
160
y1 = = 80 mm
2
4.10  Engineering Mechanics Statics

Rectangle 2: a2 = 90 × 40 = 3600 mm2


90
x2 = 30 + = 30 + 45 = 75 mm
2
40
y2 = = 20 mm
2
Now coordinates of the centroid are
a x +a x 4800 × 15 + 3600 × 75
x = 11 2 2 = = 40.71 mm   (Ans.)
a1 + a2 4800 + 3600
a1 y1 + a2 y2 4800 × 80 + 3600 × 20
y = = = 54.28 mm   (Ans.)
a1 + a2 4800 + 3600
Example 3. Determine coordinates of centroid of the shaded area shown in
figure Ex. 3.
25 mm 25 mm
50 mm

m
m
30

X
O
50 mm 50 mm
Fig. Ex. 3

Solution: Here it is important to note that given area is symmetric about its own
vertical axis therefore x coordinate of centroid of area will lie on its own vertical
axis and we have to determine only y coordinate of centroid.
Now divide the given area into four parts i.e. rectangle, triangle 1, triangle 2
and semicircle.
Rectangle: a1 = 50 × 100 = 5000 mm2
50
y1 = = 25 mm
2
1
Triangle 1: a2 = × 25 × 50 = 625 mm
2
2
y2 = × 50 = 33.33 mm
3
1
Triangle 2: a3 = × 25 × 50 = 625 mm
2
2
y3 = × 50 = 33.33 mm
3
Centroid and Moment of Inertia  4.11

πr 2 3.14 × 302
Semicircle: a4 = = = 1413.71 mm 2
2 2
4r 4 × 30
y4 = = = 12.73 mm
3π 3 × 3.14
Now y coordinate of the centroid is
a y − a y − a3 y3 − a4 y4
y = 1 1 2 2
a1 − a2 − a3 − a4
5000 × 25 – 625× 33.33 − 625× 33.33 − 1413.71×12.73
= = 27.96 mm (Ans.)
5000 − 625 − 625 − 1413.71
Example 4. Determine coordinates of centroid of the shaded area shown in
figure Ex. 4.
Y

R 2 cm

O
R 1 cm
X
3 cm
10 cm

Fig. Ex. 4

Solution: First divide the given area into four parts i.e. rectangle, triangle,
semicircle and circle.
Rectangle: a1 = 7 × 4 = 28 cm2
7
x1 = 3 + = 6.5 cm
2
4
y1 = = 2 cm
2
1
Triangle: a2 = × 3 × 4 = 6 cm 2
2
2
x2 = × 3 = 2 cm
3
1
y2 = × 4 = 1.33 cm
3
πr 2 π × 22
Semicircle: a3 = = = 6.28 cm 2
2 2
4.12  Engineering Mechanics Statics

4r
x3 = 10 + = 10.84 cm

y3 = 2 cm
Circle: a4 = pr2 = 3.14 cm2
x4 = 10 cm
y4 = 2 cm
Now coordinates of the centroid are
a x + a x + a3 x3 − a4 x4
x = 1 1 2 2
a1 + a2 + a3 − a4
28× 6.5 + 6 × 2 + 6.28×10.84 − 3.14 ×10
= = 6.21cm   (Ans.)
28 + 6 + 6.28 − 3.14
a1 y1 + a2 y2 + a3 y3 − a4 y4
y =
a1 + a2 + a3 − a4
28× 2 + 6 ×1.33 + 6.28× 2 − 3.14 × 2
= = 1.89 cm   (Ans.)
28 + 6 + 6.28 − 3.14
Example 5. Determine coordinates of centroid of the shaded area shown in
figure Ex. 5. All dimensions are in mm.

Fig. Ex. 5

Solution: First divide the given area into two parts i.e. quarter circle and semicircle
πr 2 π × 302
Quarter circle: a1 = = = 706.5 mm 2
4 4
4r 4 × 30
x1 = = = 12.73 mm
3π 3 × 3.14
4r 4 × 30
y1 = = = 12.73 mm
3π 3 × 3.14
πr 2 3.14 × 152
Semicircle: a2 = = = 353.25 mm 2
2 2
Centroid and Moment of Inertia  4.13

x2 = 15 mm
4r 4 ×15
y2 = = = 6.37 mm
3π 3 × 3.14
Now coordinates of the centroid are
a x −a x
x = 1 1 2 2
a1 − a2
706.5 × 12.73 − 353.25 × 15
= = 10.46 mm   (Ans.)
706.5 − 353.25
a1 y1 − a2 y2
y =
a1 − a2
706.5 × 12.73 − 353.25 × 6.37
= = 19.1 mm   (Ans.)
706.5 − 353.25
Example 6. Determine coordinates of centroid of the shaded area shown in
figure Ex. 6.

18 cm

3 cm

2 cm 2 cm

9 cm
Fig. Ex. 6

Solution: Here it is important to note that given area is symmetric about its own
vertical axis therefore x coordinate of centroid of area will lie on its own vertical
axis and we have to determine only y coordinate of centroid.
Now divide the given area into two parts i.e. rectangle 1 and rectangle 2
Rectangle 1: a1 = 18 × 9 = 162 cm2
18
y1 = = 9 cm
2
Rectangle 2: a2 = 2 × 3 = 6 cm2
3
y2 = 2 + = 3.5 cm
2
4.14  Engineering Mechanics Statics

Now y coordinate of the centroid is


a y −a y
y = 1 1 2 2
a1 − a2
162 × 9 − 6 × 3.5
= = 9.21cm   (Ans.)
162 − 6
Example 7. Determine coordinates of centroid of the shaded area shown in
figure Ex. 7. All dimensions are in mm.
Y 800

200

200 1200

200
X
600
Fig. Ex. 7

Solution: First divide the given area into two parts i.e. rectangle 1, rectangle 2
and rectangle 3
Rectangle 1: a1 = 600 × 200 = 120000 mm2
600
x1 = = 300 mm
2
200
y1 = = 100 mm
2
Rectangle 2: a2 = 1200 × 200 = 240000 mm2
200
x2 = 200 + = 300 mm
2
1200
y2 = 200 + = 800 mm
2
Rectangle 3: a3 = 8000 × 200 = 160000 mm2
800
x3 = = 400 mm
2
200
y3 = 1400 + = 1500 mm
2
Centroid and Moment of Inertia  4.15

Now coordinates of the centroid are


a x + a x + a3 x3
x = 1 1 2 2
a1 + a2 + a3
120000 × 300 + 240000 × 300 + 160000 × 400
= = 330.76 mm   (Ans.)
120000 + 240000 + 160000
a1 y1 + a2 y2 + a3 y3
y =
a1 + a2 + a3
120000 ×100 + 240000 × 800 + 160000 ×1500
= = 853.84 mm   (Ans.)
120000 + 240000 + 160000
Example 8. Determine coordinates of centroid of the shaded area shown in
figure Ex. 8.

10 cm

10 cm

4
cm 5 cm

20 cm
Fig. Ex. 8

Solution: Here it is important to note that given area is symmetric about its own
vertical axis therefore x coordinate of centroid of area will lie on its own vertical
axis and we have to determine only y coordinate of centroid.
Now divide the given area into two parts i.e. triangle and rectangle
1 2
Triangle: a1 = × 20 × 25 = 250 cm
2
25
y1 = = 8.33 cm
3
Rectangle: a2 = 4 × 10 = 40 cm2
10
y2 = 5 + = 10 cm
2
Now y coordinate of the centroid is
a y −a y
y = 1 1 2 2
a1 − a2
4.16  Engineering Mechanics Statics

250 × 8.33 − 40 ×10


= = 8.01cm   (Ans.)
250 − 40
Example 9. Determine coordinates of centroid of the shaded area shown in
figure Ex. 9.

Fig. Ex. 9

Solution: First divide the given area into two parts i.e. triangle 1 and triangle 2
1 2
Triangle 1: a1 = × 40 × 50 = 1000 cm
2
50
x1 = = 16.67 cm
3
40
y1 = = 13.33 cm
3
1
Triangle 2: a2 = × 40 × 30 = 600 cm 2
2
30
x2 = = 10 cm
3
40
y2 = = 13.33 cm
3
Now coordinates of the centroid are
a x −a x
x = 1 1 2 2
a1 − a2
1000 × 16.67 − 600 × 10
= = 26.67 cm   (Ans.)
1000 − 600
a1 y1 − a2 y2
y =
a1 − a2
1000 × 13.33 − 600 × 13.33
= = 13.33 cm   (Ans.)
1000 − 600
Centroid and Moment of Inertia  4.17

Example 10. Determine coordinates of centroid Y


of the shaded area shown in figure Ex. 10. All
dimensions are in mm.
Solution: Here it is important to note that given
area is symmetric about its own vertical axis 100 200
X
therefore x coordinate of centroid of area will lie
on its own vertical axis and we have to determine Fig. Ex. 10
only y coordinate of centroid.
Now divide the given area into two parts i.e. semicircle 1 and semicircle 2
πr 2 π × 2002
Semicircle 1: a1 = = = 62831.85 mm 2
2 2
4r 4 × 200
y1 = = = 84.88 mm
3π 3π
πr 2 π ×1002
Semicircle 2: a2 = = = 15707.96 mm 2
2 2
4r 4 ×100
y2 = = = 42.44 mm
3π 3π
Now y coordinate of the centroid is

y = a1 y1 − a2 y2
a1 − a2
62831.85× 84.88 − 15707.96 × 42.44
= = 99.02 mm (Ans.)
62831.85 − 15707.96
Example 11. Determine location of centroid of homogeneous bend wire shown
in figure Ex. 11. All dimensions are in mm.

Fig. Ex. 11

Solution: Here we consider three segments of wire, AB, BC and CA.


Segment AB: l1 = 120 mm
x1 = 0
4.18  Engineering Mechanics Statics

120
y1 = = 60 mm
2
l2 = 130 mm
Segment BC:
50
x1 = = 25 mm
2
120
y1 = = 60 mm
2
Segment CA:
l3 = 50 mm
50
x3 = = 25 mm
2
y3 = 0
Now coordinates of the centroid are
l x +l x +l x 120 × 0 + 130 × 25 + 50 × 25
x = 1 1 2 2 3 3 = = 15 mm   (Ans.)
l1 + l2 + l3 120 + 130 + 50
l y +l y +l y 120 × 60 + 130 × 60 + 50 × 0
y = 1 1 2 2 3 3 = = 50 mm   (Ans.)
l1 + l2 + l3 120 + 130 + 50

SUMMARY
• Weight of the body is distributed force system.
• Center of gravity is the average position of a distribution of weight.
• Center of mass is the average position of a distribution of mass.
• Centroid is the average position of a distribution of area or shape.
• If an area has symmetry about one axis its centroid lies on the axis.
• If an area has symmetry about two axes its centroid lie at the intersection
of axes.
• Coordinates of centroid of composite areas are determined using following
equations
n

x =
∑ i=1xi Ai
n
∑ i=1 Ai
n

y =
∑ i=1yi Ai
n
∑ i=1 Ai
Centroid and Moment of Inertia  4.19

PROBLEMS

Determine coordinates of centroid of the following shaded area.


y

3 cm 12 cm

6 cm
x
18 cm 6 cm

y 12 cm

4 cm

4 cm
3 cm
2 cm
x
5 cm 2 cm 2 cm 3 cm

y y
60 mm
40 mm
20 mm

75 mm
60 mm

x
75 mm x
20 mm

4.7 Area Moment of Inertia


Moment of inertia is a measure of an object’s resistance to changes to its rotation.
It is also defined as the capacity of a cross section
to resist bending. Moment of inertia depends Moment of Inertia is the
upon how the mass is distributed relative to the second moment of area.
axis of rotation.
The moment of inertia of an area about any axis is the second moment of
area about that axis.
Consider an area A in the X – Y plane as shown in figure 5.
4.20  Engineering Mechanics Statics

x dA

O X

Fig. 5

Moment of inertia of area A about the x axis is defined as,

∫y
2
Ix = dA , ...(Eq. 4.6)
here y is the y coordinate of differential element of area dA.
Moment of inertia of area A about the y axis is defined as,
Iy = ∫ x dA ,
2
...(Eq. 4.7)
here x is the x coordinate of differential element of area dA.
The moment of inertia is denoted by I and its unit is mm4, m4.

4.8 Perpendicular Axis Theorem


Consider a plane area ‘A’ lying in plane XY as shown in figure 6. X and Y are two
mutually perpendicular axes and Z is perpendicular axis.
Y

x dA

r y

O X

Z
Fig. 6
Centroid and Moment of Inertia  4.21

Consider differential element of area dA. Let x be the distance of dA from Y


axis, y be the distance of dA from X axis and r be the distance of dA from Z axis.
Now moment of inertia of area A about Z axis which is also known as Polar
moment of inertia is defines as,

∫r
2
Iz = dA ...(Eq. 4.8)
2 2 2
Now from figure, r = x + y
Therefore Eq. 4.8 can be written as,
Iz = ∫ ( x 2 + y 2 )dA

∫y dA + ∫ x 2 dA ,
2
=
From Eq. 4.6 and Eq. 4.7 we can write,
Iz = Ix + Iy ...(Eq. 4.9)
Theorem of perpendicular axis states that if Ix and Iy be the moment of inertia
of a plane area about two mutually perpendicular axis X and Y in the plane of the
area then the moment of inertia of the area Iz about the axis Z perpendicular to the
plane and passing through the intersection of X and Y is given by Eq. 4.9.

4.9  Parallel Axis Theorem


Sometime the moments of inertia of an area are known for a particular coordinate
system but we need their values in terms of some other coordinate system. When
the coordinate systems are parallel, the desired moments of inertia can be obtained
using parallel axis theorem.
Consider a plane area ‘A’ lying in plane XY as shown in figure 7.
Centroidal axis y

dA

A
y
G Centroidal axis x

A B
Parallel axis
Fig. 7

Let, Centroidal axis x in the plane of area A and passing through the centroid
of the area.
AB is the axis in the plane of area ‘A’ and parallel to the centroidal axis x
h be the distance between axis AB and centroidal axis x.
4.22  Engineering Mechanics Statics

Now consider differential element of area dA at distance y from centroidal


axis. Moment of inertia of the area dA about AB,
IAB = ∫ ( y + h) dA
2

Expanding above integration,

∫y dA + ∫ 2 y h dA + ∫ h 2 dA
2
IAB =

∫y dA + 2h ∫ y dA + h 2 ∫ dA
2
=

∫y
2
From above we see that the first integral dA is moment of inertia of area
A about centroidal axis x = Ix
Second integral ∫ y dA = Ay = 0, as y =0

And third integral is h2A


IAB = Ix + h2 A ...(Eq. 4.10)
This is a parallel-axis theorem which relates the moment of inertia of A about
the centroidal axis through the centroid to the moment of inertia about the parallel
axis AB.

4.10 Radius of Gyration
Consider an area A which has a moment of inertia IAB with respect to the AB axis
as shown in figure 8. Let us imagine that we concentrate this area into a thin strip
parallel to the AB axis as shown in right hand part of figure 8. If the area A is
to have the same moment of inertia with respect to the AB axis, the strip should
be placed at a distance k from the AB axis, where k is defined by the radius of
gyration.
I AB
k =
A
A A
A

A
k

B B
Fig. 8

4.11 Area Moment of Inertia of Common Areas


1. Area M.I. of rectangle about base: Consider a rectangle of base b and
height h. X is the reference axis passing through base.
Centroid and Moment of Inertia  4.23

Now consider an elemental strip of thickness dy located at a distance y


from the reference axis X as shown in figure 9.
Using Eq. 4.6

∫y
2
Ix = dA
dy
Area of elemental strip dA = b dy h

h
h h  y3  y
∫0 y b dy = b ∫0
2 2
Ix = y dy = b  
 3 0 X
3
bh b
Ix =    ...(Eq. 4.11)
3 Fig. 9.

2. Area M.I. of rectangle about centrodial axis: Area moment of inertia


of rectangle about its centrodial x’ axis (horizontal axis passing through
centroid of rectangle) is determine by applying parallel axis theorem.
By parallel axis theorem,
Ix = Ix′ + Ah2
Where h = Distance between the reference axis and parallel centroidal
axis.
Ix′ = Ix – Ah2
bh3
From Eq. 4.11, Ix =
3 2
bh3  h  bh
3
bh3
Ix = – b×h×  = –
3 2 3 4

bh3
Ix′ = ...(Eq. 4.12)
12
3. Area M.I. of triangle about base: Consider a triangle of base b and
height h. X is the reference axis passing through base. Now consider an
elemental strip of thickness dy located at a distance y from the reference
axis as shown in figure 10.

h dy
l
y

X
b
Fig. 10
4.24  Engineering Mechanics Statics

Using Eq. 4.6

∫y
2
Ix = dA
b
Area of elemental strip dA = (h – y )dy
h
2b
Ix = ∫ y (h − y ) dy
h
h
b 2 b  y 3h y 4 
Ix = ∫ y (h − y ) dy =  − 
h h 3 4 0
bh3
Ix = ...(Eq. 4.13)
12
4. Area M.I. of triangle about centrodial axis: Area moment of inertia
of triangle about its centrodial x’ axis (horizontal axis passing through
centroid of rectangle) is determine by applying parallel axis theorem.
By parallel axis theorem,
Ix = Ix′ + Ah2
Where h = Distance between the reference axis and parallel centroidal axis.
Ix′ = Ix – Ah2
bh3
From Eq. 4.13, Ix =
12
2
bh3 1  h  bh
3
bh3
Ix′ = – b×h×  = –
12 2 3 12 18
bh3
Ix′ =
36
Area moment of inertia of some common plane areas are shown in table 4.2.
Table 4.2 Area Moment of Inertia of Plane Area.

Shape Moment of Inertia


Rectangle
y
b y
1 3 1
2
Ix′ = bh Ix = bh3
12 3
1 3 1
h C x Iy′ = b h Iy = b 3 h
12 3
h
2
x
b
Centroid and Moment of Inertia  4.25

Right Triangle
y
b y
3
1 3 1 3
Ix′ = bh Ix = bh
36 12
h
x 1 3 1 3
C Ix′ = hb Iy = hb
h
3 36 12
x
b

Isosceles Triangle

1 3 1 3
Ix′ = bh Ix = bh
36 12

1 3
Iy′ = hb
48

Circle
y

r 1 4
Ix′ = I y ′ = πr
C
x 4

Semicircle
y

1
Ix = I y = πr 4
C 8
x
r 4r
3 x Ix′ = 0.1098r4

Quarter Circle
y
1 4
Ix = I y = πr
16
r
x
C
Ix′ = Iy′ = 0.05488r4
x
r
4.26  Engineering Mechanics Statics

Solved Examples Based on Area Moment of Inertia


Example 1. Determine Ix and Iy of the shaded area shown in figure Ex. 1. All
dimensions are in mm.
y

60

x
20 40

Fig. Ex.1

Solution: Here the x axis is passing through the base of rectangle so we use
following expression to determine Ix 
bh3 40 × 603
Ix = = = 2880000 mm 4   (Ans.)
3 3
Now to determine Iy we use parallel axis theorem.
Iy = Iy’ + Ah2
Here h is the distance between y axis and y’ axis (vertical axis passing through
centroid of rectangle)
60 × 403
= + 60 × 40 × 402 = 4160000 mm 4   (Ans.)
12
Example 2. Determine IxC of the shaded area shown in figure Ex. 2. All
dimensions are in mm.
20

80

Xc

35
20

80

Fig. Ex. 2
Centroid and Moment of Inertia  4.27

Solution: Here first we divide the given area into two parts i.e. rectangle 1 and
rectangle 2 then we apply parallel axis theorem to both rectangle about
Rectangle 1: IX = Ix′ + Ah2
C
20 × 803
= + 20 × 80 × (60 – 35) 2
12
IX = 1853333.33 mm4
C
Rectangle 2: IX = Ix′ + Ah2
C
80 × 203
= + 20 × 80 × (35 – 10) 2
12
IX = 1053333.33 mm4
C
Total IX = (IX of rectangle 1) + (IX of rectangle 2)
C C C
Total IX = 1853333.33 + 1053333.33
C
\ Moment of inertia of given area about x = 2906666.66 mm4  (Ans.)
C
Example 3. A 20 × 20 mm square is removed from a 40 × 40 mm square as
shown in figure Ex. 3. Determine IxC and IyC.
Solution: Here first we divide the given area into two parts i.e. square 1 and
square 2. As xC is centroidal axis which is passing through centroid of given area
we can apply Eq. 4.12 to both rectangle.
bh3 Yc
Square 1: IX = I x , =
C 12
40 × 403
=
12 Xc
4
IX = 213333.33 mm
C
bh3
Rectangle 2: IX = I x , =
C 12 Fig. Ex. 3
3
20 × 20
=
12
IX = 13333.33 mm4
C
Total IX = (IX of square 1) – (IX of square 2)
C C C
Total IX = 213333.33 – 13333.33
C
\ Moment of inertia of given area about xC = 200000 mm4  (Ans.)
4.28  Engineering Mechanics Statics

Example 4. Determine IxC and IyC of the shaded area shown in figure Ex. 4.
yc
0.165 m

0.8 m
C
xc
0.1m
0.265 m

0.1m 0.5 m

Fig. Ex. 4
Solution: Here first we divide the given area into two parts i.e. rectangle 1 and
rectangle 2. Now we apply parallel axis theorem to both rectangle about xC.
Rectangle 1: IX = Ix’ + Ah2
C
0.1× 0.83
= + 0.1 × 0.8 × (0.4 – 0.265) 2
12
IX = 5.72 × 10–3 m4
C
Rectangle 2: IX = Ix’ + Ah2
C
0.5× 0.13
= + 0.5 × 0.1 × (0.265 – 0.05) 2
12
IX = 2.35 × 10–3 m4
C
Total IX = (IX of rectangle 1) + (IX of rectangle 2)
C C C
–3 –3
Total IXC = 5.72 × 10 + 2.35 × 10
\ Moment of inertia of given area about xC = 8.07 × 10–3 m4  (Ans.)
Now we apply parallel axis theorem to both rectangle about yC
0.8× 0.13
Rectangle 1: Iy = Iy’ + Ah2 = + 0.1 × 0.8 × (0.165 – 0.05) 2
C 12
Iy = 1.12 × 10–3 m4
C

0.1× 0.53
Rectangle 2: Iy = Iy’ + Ah2 = + 0.5 × 0.1 × (0.35 – 0.165) 2
C 12
Iy = 2.75 × 10–3 m4
C
Total Iy = (Iy of rectangle 1) + (Iy of rectangle 2)
C C C
Total Iy = 1.12 × 10–3 + 2.75 × 10–3
C
\ Moment of inertia of given area about yC = 3.87 × 10–3 m4  (Ans.)
Centroid and Moment of Inertia  4.29

Example 5. Determine Ix of the shaded area shown in figure Ex. 5.

Fig. Ex. 5

Solution: Here first we divide the given area into two parts i.e. triangle and rectangle.
Triangle: Here the x axis is passing through the base of triangle so we use
following expression to determine Ix
bh3 20 × 253
Ix = = = 26041.7 cm 4
12 12
Rectangle: Here we apply parallel axis theorem about x
Ix = Ix’ + Ah2
4 × 103
= + (4 × 10) × 52 = 1333.3 cm4
12
Total Ix = (Ix of triangle) – (Ix of rectangle)
= 26041.7 – 1333.3
\ Moment of inertia of given area about x = 24708.4 cm4  (Ans.)
Example 6. Determine area moment of inertia about centroidal axes of shaded
area shown in figure Ex. 6.

Fig. Ex. 6
4.30  Engineering Mechanics Statics

Solution: Centroidal axes means the x’ and y’ axis which are passing through
centroid of given area. Therefore, first we determine centroid of given area.
It is important to note that given area is symmetric about its own vertical axis
therefore x coordinate of centroid of area will lie on its own vertical axis and we
have to determine only y coordinate of centroid.
a y + a y + a3 y3
y = 1 1 2 2 (y distance is measured from base of given area)
a1 + a2 + a3
(18 × 2) × 15 + (2 × 12) × 8 + (2 × 5) × 1
=
(18 × 2) + (2 × 12) + (2 × 5)
y = 10.6 cm
Now we divide the given area into three parts i.e. rectangle 1, rectangle 2 and
rectangle 3. Apply parallel axis theorem to all rectangles about xC.
Rectangle 1: IXC = Ix’ + Ah2
1
= (18 × 23 ) + (18 × 2) (15 – 10.6) 2
12
IX = 708.96 cm4
C
Rectangle 2: IX = Ix’ + Ah2
C
1
= (2 × 123 ) + (2 × 2) (10.6 – 8) 2
12
IX = 315.04 cm4
C
Rectangle 3: IX = Ix’ + Ah2
C
1
= (5 × 23 ) + (2 × 5) (10.6 – 1) 2
12
IX = 924.93 cm4
C
Total IX = (IX of rectangle 1) + (IX of rectangle 2) + (IX of rectangle 3)
C C C C

Total IX = 708.96 + 315.04 + 924.93


C
\ Moment of inertia of given area about xC = 1948.93 cm4  (Ans.)
Now we apply parallel axis theorem to all rectangles about yC. In following
calculation, distance h is zero because the vertical axis of individual areas coincide
with vertical axis of composite area.
Rectangle 1: Iy = Iy’ + Ah2
C
1
= (2 × 183 ) + 0
12
Iy = 972 cm4
C
Centroid and Moment of Inertia  4.31

Rectangle 2: Iy = Iy’ + Ah2


C

1
= (12 × 23 ) + 0
12
Iy = 8 cm4
C
Rectangle 3: Iy = Iy’ + Ah2
C
1
= (2 × 53 ) + 0
12
Iy = 20.83 cm4
C
Total Iy = (Iy of rectangle 1) + (Iy of rectangle 2) + (Iy of rectangle 3)
C C C C
Total Iy = 972 + 8 + 20.83
C
\ Moment of inertia of given area about yC = 1000.83 cm4  (Ans.)
Example 7. Determine area moment of inertia about centroidal x’ axes of shaded
area shown in figure Ex. 7.

40 cm

14 cm

20 cm

18 mm
Fig. Ex. 7

Solution: Centroidal x’ means the axis which are passing through centroid of
given area. Therefore first we determine centroid of given area.
It is important to note that given area is symmetric about its own vertical axis
therefore x coordinate of centroid of area will lie on its own vertical axis and we
have to determine only y coordinate of centroid.
a y − 2[a2 y2 ]
y = 1 1 (y distance is measured from base of given area)
a1 − 2a2
(18 × 74) × 37 − 2 × [(3.14 × 7 × 7) × 27]
=
(18 × 74) − 2 × (3.14 × 7 × 7)
y = 40 mm
4.32  Engineering Mechanics Statics

Now we divide the given area into three parts i.e. rectangle and two semicircles.
Apply parallel axis theorem to all areas about xC.
Rectangle: IX = Ix’ + Ah2
C
18 × 743
= + (18 × 74) × (40 – 37) 2
12
IX = 619824 mm4
C
Two semicircle: IX = 2[Ix’ + Ah2]
C
 3.14 × 7 4  3.14 × 7 2 2

= 2   + (40 − 27) 
 8  2 
4
IX = 27887.125 mm
C
Total IX = (IX of rectangle ) + (IX of two semicircle)
C C C
Total IX = 619824 + 27887.125
C
\ Moment of inertia of given area about xC = 647711.125 mm4  (Ans.)

SUMMARY
• Moment of inertia is a measure of an object’s resistance to changes to its
rotation.
• Moment of inertia measures how an area is distributed about particular
axes.
• Area moment of inertia is defined as,
Ix = ∫ y dA, ,
2

∫ x dA
2
Iy =

• Radius of Gyration is defined as


I
k =
A
• The parallel-axis theorem is usually used to calculate the moment of
inertia about a second axis which is parallel to first axis and moment of
inertia about first axis is known.
Centroid and Moment of Inertia  4.33

PROBLEMS

1. Determine Ix , Iy, Kx and Ky of shaded area shown in figure Prob. 1.

Fig. Prob. 1
2. Determine Ix , and Iy, of shaded area shown in figure Prob. 2.
y 60 mm

20 mm

60 mm

x
20 mm
Fig. Prob. 2
3. Determine Ix, and Iy, of shaded area shown in figure Prob. 3. All dimensions are in mm.
y
800

200

200 1200

200
x
600
Fig. Prob. 3
4.34  Engineering Mechanics Statics

4. Determine Ix¢ and Iy¢ of shaded area shown in figure Prob. 4. All dimensions are in mm.
y
20

160

40
x
90

Fig. Prob. 4
5. Determine area moment of inertia about centroidal axes of shaded area shown in
figure Prob. 5.

2 cm 12 cm 2 cm

18 cm 5 cm

2 cm

Fig. Prob. 5
Unit 5

In most of the problems that we have analysed up to last unit, the


contact surfaces assumed to be smooth or frictionless. Actually when
two objects are come in contact, generally friction force develop
between them. In this unit, we focus on friction force and models for
calculating friction forces. We discuss methods of analysing problems
that involve friction and sliding.
Friction
5.1 Introduction
Friction is defined as the resistance force which
Frictional force is tangent
one body offers to the motion of a second body
to the surfaces of contact
when the second body tends to slide or slide over
of the bodies.
the former.
In many situations, friction forces are helpful. On the other hand, friction can
also be unfavourable.
Friction may be classified into two types, Dry friction and Fluid friction.
In this book, we will concentrated on Dry friction

Friction

Dry Friction Fluid Friction

Static Friction Kinetic Friction

Dry friction refers to the friction force that exists between two un-lubricated
solid surfaces. Fluid friction acts between moving surfaces that are separated by
a layer of fluid. Static friction experienced by a body when it is at rest under the
action of forces. Kinetic friction experienced by a body when it is moving.

5.2 Coulomb’s Theory of Dry Friction


Consider the block of weight W rest on rough horizontal surface and subjected to
horizontal force P as shown in left hand part of Figure 1. The free body diagram
of the block is shown in right hand part of Figure 1.
W

P P
W

N
Fig. 1

If applied force P is small, static friction force F balance P and the block
will not move. If the force P is increased, the friction force F also increases to
5.4  Engineering Mechanics Statics

maintain equilibrium until its magnitude reaches


Maximum frictional force
a maximum value Fmax. When P exceeds Fmax
Fmax is called as limiting
the friction force cannot balance it and the block
friction.
start sliding and friction force F drops to a kinetic
value. [There are irregularities of the surfaces
in contact. When one surface comes in contact
At limiting friction body
with other, the actual area of contact is very less
is said to have impending
than the surface area of contact. Because of this,
motion.
pressure due to the reaction force is very high,
hence irregularities deform a little and cold
welds are formed at contact points. So to start relative sliding between surfaces
enough force is required to break these cold welds. Once the welds break when
one surface start sliding over other, a smaller force is enough to keeps it moving
with uniform velocity. This is one of the reason that coefficient of static friction
is more than coefficient of kinetic friction.] If P is further increased F remains
constant. The plot of F verses P is shown in Figure 2.
F
No Motion
Fs Fmax

Motion
Fk

F=P

P
Fig. 2

5.3 Coefficient of Friction
In order to determine the maximum frictional force corresponding to any normal
force, a certain experimental constant known as the coefficient of friction (μ),
is used. Coefficient of friction does not depend upon the area of the surface in
contact but depend on nature of the surfaces in contact.
The coefficient of static friction μs is defined as the ratio of limiting friction
(Fmax) to corresponding normal force,
F
μs = max or Fmax = μs N ...(Eq. 5.1)
N
Typical values of μs for various materials are shown in Table 5.1.
Friction  5.5

Table 5.1 Values of μs for Various Materials

Materials Coefficient of Static Friction


Wood on wood 0.25 to 0.50
Wood on leather 0.25 to 0.50
Metal on metal 0.15 to 0.30
Metal on wood 0.20 to 0.60
Metal on stone 0.30 to 0.70
Metal on leather 0.30 to 0.60
Stone on stone 0.40 to 0.65
Rubber on concrete 0.60 to 0.90
If two surfaces move relative to each other, the ratio of the friction developed
to the corresponding normal force is known as coefficient of kinetic friction μk.
F
μk = k ...(Eq. 5.2)
N
5.4  Angle of Friction
The angle of static friction (fs) is defined as the angle between normal reaction
and resultant force when motion is impending. W
It is evident from Figure 3,
F
tan ∅s = max P
N
But from Eq. 5.1,
Fmax Fmax
μs =
N s
R
\ μs = tan fs ...(Eq. 5.3) N

Angle of static friction ∅s = tan–1 μs ...(Eq. 5.4) Fig. 3


The angle of kinetic friction (∅k) is defined as the
angle between normal reaction and resultant force when two surfaces move
relative to each other.
Angle of kinetic friction ∅k = tan–1 μk ...(Eq. 5.5)

5.5  Angle of Repose


Angle of repose is the maximum angle of an inclined plane from horizontal at
which body is at impending motion. Consider a block of weight W on an inclined
plane as shown in left hand part of Figure 4. The free body diagram at impending
motion (block is about to slide down the inclined plane) is shown in right hand
part of Figure 4.
5.6  Engineering Mechanics Statics

W


N
 F

Fig. 4

Applying conditions of equilibrium i.e. SH = 0, and SV = 0 along and normal


to plane,
SH = 0: F – W sin α = 0
Replacing friction force by μs N from (Eq. 5.1),
μs N – W sin α = 0  \ μs N = W sin α ...Eq. (a)
SV = 0: N – W cos α = 0  \ N = W cos α
Substitute value of N in Eq. (a)
μs W cos α = W sin α  \ μs = tan α
But from (Eq. 5.3), μs = tan ∅s
\ tan ∅s = tan α
\ ∅s = α ...(Eq. 5.6)
From (Eq. 5.6), we can say that, Angle of repose (α) = Angle of static friction
(∅s).

5.6  Applications of Friction


The various equations discussed in previous sections have a number of applications
in engineering practices. Following are some of the important applications of
friction.

5.6.1 Ladder Friction
A ladder is a device used for climbing on the walls. A ladder only stands if there
is friction between ladder and the wall, and ladder and the ground. The left hand
part of Figure 5 shows a ladder AB of weight W with its end A resting on the
ground and end B leaning against a wall. The free body diagram of the ladder
shown in right hand part of Figure 5 is a non-concurrent force system. Now
by applying equilibrium equations assuming ladder impending, we can solved
problem of ladder friction.
Friction  5.7

B B
NB

FB

 A
A
FA

NA

Fig. 5

5.6.2  Wedge Friction


A wedge is a simple device of triangular cross-section used for small adjustment
in the position of a body or to apply large forces. Wedges mainly depend on
friction during functioning. When a wedge is pushed forward, its faces exert large
normal forces which can be used to lift the load. The left hand part of Figure 6
shows a wedge used to lift a large load W. The free body diagram of the wedge
and load is shown in right hand part of Figure 6. The force P necessary to start
raising the load can be determined by applying equations of equilibrium assuming
weight and wedge are impending.
W

F3

N3

W  N2
F2
P P F2
N2 


F1
N1

Fig. 6

5.6.3 Belt Friction
Friction is useful in belt or rope and driving surfaces for the transmission of
power. If a belt or rope passes over a rough cylinder or pulley, the tension on the
two sides of the pulley will not be equal. The tension in the belt or rope on the
two sides of a rough pulley is determined as follow.
5.8  Engineering Mechanics Statics

The left hand part of the Figure 7 shows a thin flexible flat belt passing over a
cylinder. The angle of contact is β and belt tensions are T1 and T2. T1 is the force
on low tension side and T2 is the force on high tension side. Here we assume that
tension T1 is known, and we have to determine largest force T2 that is acting on
another end of belt.

Fig. 7

The free body diagram of the differential element is shown in right hand part
of the Figure 7. As tension in the belt varies with position, the belt tension on the
right side of the element is T and tension on the left side of the element is T + ∆T.
The force ∆N is the normal force exerted on the element by cylinder. As we have
to determine largest value of T2 that will not cause the belt to slip, we assume that
friction force is equal to its maximum value μs ∆N.
Now applying equilibrium equations,
+
 ∆α   ∆α 
→ΣH = 0: –(T + ∆T )cos   + T cos   + µ s ∆N ...(Eq. 5.7)
 2   2 
 ∆α   ∆α 
+↑SV = 0: –(T + ∆T )sin   – T cos   + ∆N ...(Eq. 5.8)
 2   2 

dα  dα   dα  dα  dα 
Since is small, cos   ≈ 1 and sin  ≈ . The term dT sin  
2  2   2  2  2 
is small quantity of the second order and may be neglected.

Hence (Eq. 5.7) and (Eq. 5.8) become,


μs dN – dT = 0
dN – T da = 0
Eliminating dN,
dT
= μsda ...(Eq. 5.9)
T
This expression is a differential equation that describes how the force T
changes with position α. By integrating (Eq. 5.9) the relationship between T1 and
T2 is determined as follow,
Friction  5.9

T2 dT β
∫T1 T
= ∫0 µ s d α
T2
\ In = msb
T1
T2
\ = ems b ...(Eq. 5.10)
T1
Where, e is the base of natural logarithms and b is angle of contact measured
in radians.

Solved Examples Based on Block Friction


Example 1. A 400 N block is resting on rough horizontal surface is subjected
to 80 N force as shown in Figure Ex. 1. Determine the friction force between the
block and the surface. Assume ms = 0.5.

80 N
400 N

Fig. Ex. 1

Solution: Assume equilibrium of the block and apply conditions of equilibrium


i.e.
+
→ΣH = 0, and +↑SV = 0 to the free body diagram of the block as shown in
figure Ex.1(a).
SH = 0: 80 – F = 0 400 N

\ F = 80 N 80 N
SV = 0: N – 400 = 0
\ N = 400 N
F
Now from (Eq. 5.1), Fmax = μs N, we can
check our assumption of equilibrium of the block N
as below, Fig. Ex. 1(a)
Fmax = 0.5 × 400
\ Fmax = 200 N
Since calculated value of friction force from equilibrium analysis is less than
Fmax (F < Fmax), we conclude that the block is in static equilibrium and the value
of the friction force is,
F = 80 N  (Ans.)
5.10  Engineering Mechanics Statics

Example 2. In Example 1, if block is subjected to 120 N force, determine the


friction force between the block and the surface. Assume ms = 0.25 and mk = 0.20.
Solution: Assume equilibrium of the block and apply conditions of equilibrium
+
i.e. →ΣH = 0, and +↑SV = 0 to the free body diagram of the block as shown in
figure Ex. 2.
SH = 0: 120 – F = 0 400 N

\ F = 120 N 120 N
SV = 0: N – 400 = 0
\ N = 400 N
F
Now from (Eq. 5.1), Fmax = μs N, we can
check our assumption of equilibrium of the block N
as below, Fig. Ex. 2(a)
Fmax = 0.25 × 400
\ Fmax = 100 N
Since calculated value of friction force from equilibrium analysis is greater
than Fmax (F > Fmax), we conclude that the block will not be in equilibrium and
slide to the right and the value of the friction force is determined from (Eq. 5.2),
F = Fk = μk N
Fk = 0.20 × 400 = 80 N  (Ans.)
Example 3. A 60 kg block is subjected to force P = 400 N as shown in Figure
Ex. 3. Determine the friction force between the block and the surface. Assume
ms = 0.6.
400 N

30°

Fig. Ex. 3

Solution: Assume equilibrium of the block and apply conditions of equilibrium


+
i.e. →ΣH = 0, and +↑SV = 0 to the free body diagram
of the block as shown in figure Ex. 3(a).
SH = 0: 400 cos 30° – F = 0
\ F = 346.41 N
SV = 0: N – 60 × 9.81 – 400 sin 30°
\ N = 788.6 N
Fig. Ex. 3(a)
Friction  5.11

Now from (Eq. 5.1), Fmax = μs N, we can check our assumption of equilibrium
of the block as below,
Fmax = 0.6 × 788.6  \ Fmax = 473.16 N
Since calculated value of friction force from equilibrium analysis is less than
Fmax (F < Fmax), we conclude that the block is in static equilibrium and the value
of the friction force is,
F = 346.41 N  (Ans.)
Example 4. Determine the force P required to start moving the 250 N block as
shown in Figure Ex. 4 up the inclined surface. Assume ms = 0.3.

P
30º

Fig. Ex. 4

Solution: Here we can consider impending motion of the block up the plane and
then apply conditions of equilibrium i.e. SH = 0 250 N
along the inclined plane, and SV = 0 perpendicular
30°
to the inclined plane. Free body diagram of the
block is shown in figure Ex 4(a).
F
SV = 0: N – 250 cos 30° = 0
\ N = 216.50 N P
SH = 0: P – F – 250 sin 30° = 0 N
Now for impending motion from (Eq. 5.1), we
Fig. Ex. 4(a)
know Fmax = μs N.
\ P – μs N – 250 sin 30° = 0
P = 0.30 × 216.50 + 250 sin 30°  \ P = 190 N  (Ans.)
Example 5. Determine the horizontal force P required to start moving the
500 N block as shown in Figure Ex. 5 up the inclined surface. Assume ms = 0.3.

30º

Fig. Ex. 5

Solution: Here we can consider impending motion of the block up the plane and
then apply conditions of equilibrium i.e. SH = 0 along the inclined plane, and
5.12  Engineering Mechanics Statics

SV = 0 perpendicular to the inclined plane. Free body diagram of the block is


shown in figure Ex 5(a).
SH = 0: P cos 30° – F – 500 sin 30° = 0 500 N

P cos 30° = F + 500 sin 30° 30°

Now for impending motion from (Eq. 5.1), we P


know Fmax = μs N. 30° F
\ P cos 30° = μs N + 500 sin 30° ...(Eq. 1)
Now, SV = 0: N – P sin 30° – 500 cos 30° = 0 N
\ N = P sin 30° + 500 cos 30° Fig. Ex. 5(a)
Substituting value of N in to Eq. 1,
P cos 30° = 0.30 (P sin 30° + 500 cos 30°) + 500 sin 30°
\ P = 530.58 N  (Ans.)
Example 6. The 50 N force as shown in Figure Ex. 6 is required to produce
impending motion of the block down the inclined surface. If the weight of the
block is 250 N, determine the coefficient of static friction (ms) between the block
and the inclined surface.

50 N

30°

Fig. Ex. 6

Solution: Here we can consider impending motion of the block down the plane
and then apply conditions of equilibrium i.e. SH = 0 along the inclined plane,
and SV = 0 perpendicular to the inclined plane. Free body diagram of the block
is shown in figure Ex 6(a).
SH = 0: 50 cos 30° + 250 sin 30° – F = 0
250 N
30°

50 N
30°

F
N
Fig. Ex. 6(a)
Friction  5.13

F = 50 cos 30° + 250 sin 30°


Now for impending motion from (Eq. 5.1), we know Fmax = μs N.
\ μs N = 50 cos 30° + 250 sin 30° ...(Eq. 1)
Now, SV = 0: N – 250 cos 30° + 50 sin 30° = 0
\ N = 250 cos 30° – 50 sin 30°
Substituting value of N in to Eq. 1,
μs (250 cos 30° – 50 sin 30°) = 50 cos 30° + 250 sin 30°
\ μs = 0.8788  (Ans.)
Example 7. A 150 N block is subjected to the horizontal force P as shown in
Figure Ex. 7. Determine the range of horizontal force P that will keep the 150 N
block in equilibrium. Assume ms = 0.25.

Fig. Ex. 7

Solution: Here it is clear that slipping of the block down or up the inclined plane
will impend when the magnitude of force P reaches to 150 N
its minimum value or maximum value respectively.
45°
At minimum value of P, the block will impend down
the plane and at maximum value of P, the block will
impend up the plane. P
45°
Now first consider impending motion of the
block down the plane and then apply conditions of N
F
equilibrium i.e. SH = 0 along the inclined plane, and
SV = 0 perpendicular to the inclined plane. Free body Fig. Ex. 7(a)
diagram of the block is shown in figure Ex 7(a).
SH = 0: P cos 45° + F – 150 sin 45° = 0
Now for impending motion from (Eq. 5.1), we know Fmax = μs N.
\ P cos 45° + μs N – 150 sin 45° = 0 ...(Eq. 1)
Now, SV = 0: N – 150 cos 45° – P sin 45° = 0
\ N = 150 cos 45° + P sin 45°
Substituting value of N in to Eq. 1,
P cos 45° + 0.25 (150 cos 45° + P sin 45°) – 150 sin 45° = 0
\ P = Pmin = 90 N  (Ans.)
5.14  Engineering Mechanics Statics

150 N

45°

P F
45°

Fig. Ex. 7(b)

Now consider impending motion of the block up the plane and then apply
conditions of equilibrium i.e. SH = 0 along the inclined plane, and SV = 0
perpendicular to the inclined plane. Free body diagram of the block is shown in
figure Ex 7(b).
SH = 0: P cos 45° – F – 150 sin 45° = 0
Now for impending motion from (Eq. 5.1), we know Fmax = μs N.
\ P cos 45° – μs N – 150 sin 45° = 0 ...(Eq. 2)
Now, SV = 0: N – 150 cos 45° – P sin 45° = 0
\ N = 150 cos 45° + P sin 45°
Substituting value of N in to Eq. 2,
P cos 45° – 0.25 (150 cos 45° + P sin 45°) – 150 sin 45° = 0
\ P = Pmax = 250 N  (Ans.)
\ Range of horizontal force P is from 90 N to 250 N that can keep the block
in equilibrium.
Example 8. A 50 kg block is attached to mass M using string which is passing
over a smooth pulley as shown in Figure Ex. 8. Determine the range of mass M
that will keep the block in equilibrium. Assume ms = 0.30.

Fig. Ex. 8

Solution: Here it is clear that slipping of the block down or up the inclined plane
will impend when the mass M reaches to its minimum value or maximum value
respectively. At minimum value of mass M, the block will impend down the plane
and at maximum value of mass M, the block will impend up the plane. Also as
pulley is smooth, the tension in string (M × 9.81) will remain same throughout.
Friction  5.15

Now first consider impending motion of the block down the plane and then
apply conditions of equilibrium i.e. SH = 0 along the
inclined plane, and SV = 0 perpendicular to the inclined
plane. Free body diagram of the block is shown in
figure Ex 8(a).
SH = 0: F + M × 9.81 – 50 × 9.81 × sin 30° = 0
Now for impending motion from (Eq. 5.1), we
know Fmax = μs N.
Fig. Ex. 8(a)
\ μs N + M × 9.81 – 50 × 9.81 × sin 30° = 0
M × 9.81 = 50 × 9.81 × sin 30° – 0.30 × N ...(Eq.1)
Now, SV = 0: N – 50 × 9.81 × sin 30° = 0
\ N = 245.25
Substituting value of N in to Eq. 1,
M × 9.81 = 50 × 9.81 × sin 30° – 0.30 × 245.25
\ M = Mmin = 17.5 kg  (Ans.)

Fig. Ex. 8(b)

Now consider impending motion of the block up the plane and then apply
conditions of equilibrium i.e. SH = 0 along the inclined plane, and SV = 0
perpendicular to the inclined plane. Free body diagram of the block is shown in
figure Ex 8(b).
SH = 0: – F + M × 9.81 – 50 × 9.81 × sin 30° = 0
Now for impending motion from (Eq. 5.1), we know Fmax = μs N.
\ – μs N + M × 9.81 – 50 × 9.81 × sin 30° = 0
M × 9.81 = 50 × 9.81 × sin 30° + 0.30 × N ...(Eq.2)
Now, SV = 0: N – 50 × 9.81 × sin 30° = 0
\ N = 245.25
Substituting value of N in to Eq. 2,
M × 9.81 = 50 × 9.81 × sin 30° + 0.30 × 245.25
\ M = Mmax = 32.5 kg  (Ans.)
\ Range of mass M is from 17.5 kg to 32.5 kg that can keep the block in
equilibrium.
5.16  Engineering Mechanics Statics

Example 9. Block A of 100 N and block B of 50 N are resting on a horizontal


surface as shown in Figure Ex. 9. Determine magnitude of horizontal force P
applied to block A to initiate motion. Assume coefficient of static friction between
block A and horizontal surface as 0.20.

B
P
A

Fig. Ex. 9

Solution: Here we can consider impending motion of the block A towards right
+
and then apply conditions of equilibrium i.e. →ΣH = 0, and +↑SV = 0. Free body
diagram of the entire system is shown in figure Ex 9(a).
SH = 0: P – F = 0
50 N
\ P = F
Now for impending motion from (Eq. 5.1), we
know Fmax = μs N. 100 N P
\ P = μs N ...(Eq.1)
F
SV = 0: N – 50 – 100 = 0
\ N = 150 N N
Substituting value of N in to Eq. 1, Fig. Ex. 9(a)
P = 0.2 × 150
\ P = 30 N  (Ans.)
Example 10. In Ex. 9 if the horizontal force P is applied to Block B as shown
in Figure Ex. 10. Determine magnitude of horizontal force P to initiate motion.
Assume coefficient of static friction between block A and horizontal surface as
0.20 and between block A and block B as 0.30.
P
B

Fig. Ex. 10

Solution: Here there are two possible ways in which motion can impend:
impending motion of entire system on horizontal surface, or impending motion
of block B on block A.
First consider impending motion of the entire system towards right and then
+
apply conditions of equilibrium i.e. →ΣH = 0, and +↑SV = 0. Free body diagram
of the entire system is shown in figure Ex 10(a).
Friction  5.17

SH = 0: P – F = 0 50 N
\ P = F P

Now for impending motion from (Eq. 5.1), we


100 N
know Fmax = μs N.
\ P = μs N ...(Eq.1) F
SV = 0: N – 50 – 100 = 0  \ N = 150 N
N
Substituting value of N in to Eq. 1,
Fig. Ex. 10(a)
P = 0.2 × 150
\ P = 30 N
Now consider impending motion of the block B on block A and then apply
+
conditions of equilibrium i.e. →ΣH = 0, and +↑SV = 0 to block A and block B
separately. Free body diagram of block B and block A are shown in figure Ex 10(b).

Fig. Ex. 10(b)

Block B: SH = 0: P – F2 = 0
\ P = F2 ...(Eq.2)
Now for impending motion from (Eq. 5.1), we know Fmax = μs N.
\ P = μs N2 ...(Eq. 3)
SV = 0: N2 – 50 = 0  \ N2 = 50 N
Substituting value of N in to Eq. 3,
P = 0.3 × 50  \ P = 15 N
From Eq. 2, P = F2  \ F2 = 15 N
Block A: SH = 0: F2 – F1 = 0
\ F2 = F1  \ F1 = 15 N
SV = 0: N1 – 50 – 100 = 0  \ N1 = 150 N
Now for impending motion from (Eq. 5.1), we know Fmax = μs N.
\ (F1)max = μs N = 0.2 × 150  \ (F1)max = 30 N
5.18  Engineering Mechanics Statics

We have determined that for impending motion of entire system on horizontal


surface, P = 30 N and for impending motion of block B on block A, P = 15 N.
Therefore, the motion will initiate of block B on block A at P = 15 N  (Ans.)
It is also clear from analysis of block A that calculated value of F1 = 15 N <
(F1)max = 30 N. Therefore at P = 15 N motion will not initiate between block A
and horizontal surface.
Example 11. Two boxes A and B of weight 250 N and 150 N respectively are
placed on an inclined surface as shown in Figure Ex. 11. Determine value of θ so
that motion of boxes impend down the plane. Assume coefficient of static friction
between block A and inclined surface as 0.30 and between block B and inclined
surface as 0.20.

Fig. Ex. 11

Solution: Considering impending motion of the blocks down the plane and
then apply conditions of equilibrium i.e. SH = 0 along the inclined plane, and
SV = 0 perpendicular to the inclined plane to blocks A and B separately. Free body
diagram of the blocks are shown in figure Ex. 11(a).
250 N
150 N
 R 

F
N
N F

Fig. Ex. 11(a)

Block A: SH = 0: R + 250 sin θ – F = 0


R = F – 250 sin θ
Now for impending motion from (Eq. 5.1), we know Fmax = μs N.
\ R = 0.30 N – 250 sin θ ...(Eq. 1)
Now, SV = 0: N – 250 cos θ = 0
\ N = 250 cos θ
Friction  5.19

Substituting value of N in to Eq. 1,


R = 0.30 × 250 cos θ – 250 sin θ ...(Eq. 2)
Block B: SH = 0: – R + 150 sin θ – F = 0
R = 150 sin θ – F
Now for impending motion from (Eq. 5.1), we know Fmax = μs N.
\ R = 150 sin θ – 0.20 N ...(Eq. 3)
Now, SV = 0: N – 150 cos θ = 0
\ N = 150 cos θ
Substituting value of N in to Eq. 3,
R = 150 sin θ – 0.20 × 150 cos θ ...(Eq. 4)
From Eq. 2 and Eq. 4,
0.30 × 250 cos θ – 250 sin θ = 150 sin θ – 0.20 × 150 cos θ
75 cos θ – 250 sin θ = 150 sin θ – 30 cos θ
\ 105 cos θ = 400 sin θ
\ θ = 14.70°  (Ans.)
Example 12. A 300 N crate is subjected to 80 N force as shown in Figure Ex.
12. Does the crate slide, tip or remains in equilibrium? Assume ms = 0.3.
1m

80 N
2m
1m

Fig. Ex. 12

Solution: Here we have to understand that there are two possibility of impending
motion: impending sliding or impending
tipping (In all previous examples possibility
of tipping was neglected).
Now assume equilibrium of the crate
and apply conditions of equilibrium i.e.
+
→ΣH = 0, +↑SV = 0 and SM = 0 (Treating
anti-clockwise moment as positive) to the free
body diagram of the crate as shown in figure
Ex.12(a). Note that the normal reaction must
act at a distance x from the crate’s centre line
in order to counteract tipping effect caused by
Fig. Ex. 12(a)
applied force.
5.20  Engineering Mechanics Statics

SH = 0: 80 – F = 0 \ F = 80 N
SV = 0: N – 300 = 0 \ N = 300 N
SMO = 0: 300x – 80 × 1 = 0 \ x = 0.27 m
Now from (Eq. 5.1), Fmax = μs N,
Fmax = 0.3 × 300 \ Fmax = 90 N
Since calculated value of friction force from equilibrium analysis is less than
Fmax (F < Fmax), we conclude that the crate will not slide.
Now largest possible value for x is 0.5 i.e. half the width of the crate. Since
calculated value of x from equilibrium analysis is less than largest possible value
of x, we conclude that the crate will not tip.
\ Crate remains in equilibrium when subjected to 80 N force  (Ans.)

Solved Examples Based on Ladder Friction


Example 1. A 5 m ladder of weight 300 N is placed against a smooth vertical
wall as shown in Figure Ex. 1. Does the ladder remains in equilibrium? Assume
ms between ladder and horizontal surface as 0.3.

Fig. Ex. 1

Solution: Assume equilibrium of the ladder and apply conditions of equilibrium


+
i.e. →ΣH = 0, +↑SV = 0 and SM = 0 (Treating anticlockwise moment as positive)
to the free body diagram of the ladder as shown in Figure Ex. 1(a).
SH = 0: FA – NB = 0  \ FA = NB ...(Eq.1)
SV = 0: NA – 300 = 0  \ NA = 300 N
SMA = 0: NB × 5 sin 60° – 300 × 2.5 cos 60° = 0  \ NB = 86.60 N
From Eq. 1, FA = 86. 60 N
Friction  5.21

Fig. Ex. 1(a)

Now from (Eq. 5.1), Fmax = μs N, we can check our assumption of equilibrium
of the ladder as below,
Fmax = 0.3 × NA = 0.3 × 300 \ Fmax = 90 N
Since calculated value of friction force from equilibrium analysis is less than
Fmax (86.60 < 90), we conclude that the ladder is in static equilibrium.  (Ans.)
Example 2. A 800 N man starts climbing a ladder that placed against a wall as
shown in Figure Ex. 2. Neglecting the weight of the ladder, determine how far up
the ladder the man can climb before the ladder starts slipping. Assume ms at both
surfaces as 0.25.

30°
8m

Fig. Ex. 2

Solution: Considering impending motion of the ladder, apply conditions of


+
equilibrium i.e. →ΣH = 0, +↑SV = 0 and SM = 0 (Treating anticlockwise moment
as positive) to the free body diagram of the ladder as shown in figure Ex.2(a).
5.22  Engineering Mechanics Statics

B
NB
30°
FB

800 N

A
FA

NA

Fig. Ex. 2(a)

SH = 0: FA – NB = 0
\ FA = NB
Now for impending motion we know from (Eq. 5.1), Fmax = μs N
\ 0.25 NA = NB ...(Eq. 1)
SV = 0: NA – 800 + FB = 0
\ NA = 800 – FB = 800 – 0.25 NB
Substituting the value of NA in Eq. 1,
0.25 (800 – 0.25 NB) = NB
\ NB = 188.23 N
From Eq. (1), 0.25 NA = 188.23
\ NA = 752.94 N
SMA = 0: NB × 8 cos 30° + 0.25 NB × 8 sin 30° – 800 × x sin 30° = 0
Substituting the value NB ,
188.23 × 8 cos 30° + 0.25 × 188.23 × 8 sin 30° = 800 × x sin 30°
Solving,  \ x = 3.73 m  (Ans.)
Example 3. A 600 N man starts climbing a ladder of weight 100 N that placed
against a wall as shown in Figure Ex. 3. The centre of mass of 10 m ladder is
at its midpoint. What is the largest value of α for which a man can climb to the
top of the ladder without slipping? Assume coefficient of static friction between
ladder and floor as 0.4 and coefficient of static friction between ladder and the
wall as 0.3.
Friction  5.23

 A

Fig. Ex. 3

Solution: Considering impending motion of the ladder, apply conditions of


+
equilibrium i.e. →ΣH = 0, +↑SV = 0 and SM = 0 (Treating anticlockwise moment
as positive) to the free body diagram of the ladder as shown in figure Ex.3(a).
600 N

NB
FB

5m

100 N

FA

NA

Fig. Ex. 3(a)

SH = 0: NB – FA = 0
\ NB = FA
Now for impending motion we know from (Eq. 5.1), Fmax = μs N
\ NB = 0.4 NA ...(Eq. 1)
SV = 0: NA – 600 – 100 + FB = 0
\ NA = 700 – FB = 700 – 0.3 NB
Substituting the value of NA in Eq. 1,
NB = 0.4 (700 – 0.3 NB)  \ NB = 250 N
From Eq. 1, 250 = 0.4 NA  \ NA = 625 N
5.24  Engineering Mechanics Statics

SMA = 0: 100 × 5 cos α + 600 × 10 cos α – NB × 10 sin α – FB × 10 cos α = 0


Substituting the value NB,
100 × 5 cos α + 600 × 10 cos α – 250 × 10 sin α – 0.3 × 250 × 10 cos α = 0
Solving, \ α = 66.50°  (Ans.)
Example 4. A 600 N man starts climbing a ladder of weight 100 N that placed
against a smooth wall as shown in Figure Ex. 4. The centre of mass of 10 m ladder
is at its midpoint. Determine the minimum coefficient of static friction between
the ladder and the floor for which a man can climb to the top of the ladder without
slipping.

30°

Fig. Ex. 4

Solution: Considering impending motion of the ladder, apply conditions of


+
equilibrium i.e. →ΣH = 0, +↑SV = 0 and SM = 0 (Treating anticlockwise moment
as positive) to the free body diagram of the ladder as shown in figure Ex.4(a).

Fig. Ex. 4(a)


Friction  5.25

SH = 0: NB – FA = 0
\ NB = FA
Now for impending motion we know from (Eq. 5.1), Fmax = μs N
\ NB = μs NA ...(Eq. 1)
SV = 0: NA – 600 – 100 = 0
\ NA = 700
Substituting the value of NA in Eq. 1,
NB = μs × 700
SMA = 0: 100 × 5 sin 30° + 600 × 10 sin 30° – NB × 10 cos 30° = 0
Substituting the value NB,
100 × 5 sin 30° + 600 × 10 sin 30° – (μs × 700) × 10 cos 30° = 0
Solving, \ μs = 0.536  (Ans.)
Example 5. A 600 N man starts climbing a ladder that placed against a wall as
shown in Figure Ex. 5. The weight of the ladder is 80 N. Determine how far up
the ladder the person can climb before the ladder starts slipping. Assume ms at
both surfaces as 0.20.

1m

3m
x

4m
Fig. Ex. 5

Solution: First determine inclination of ladder with horizontal as below,


3
tan α =   \ α = 36.87°
4
Now considering impending motion of the ladder, apply conditions of
+
equilibrium i.e. →ΣH = 0, +↑SV = 0 and SM = 0 (Treating anticlockwise
moment as positive) to the free body diagram of the ladder as shown in figure
Ex.5(a).
SH = 0: FA – NB sin 36.87° + FB cos 36.87° = 0
\ FA = NB sin 36.87° – FB cos 36.87°
Now for impending motion we know from (Eq. 5.1), Fmax = μs N
5.26  Engineering Mechanics Statics

\ 0.2 NA = NB sin 36.87° – 0.2 × NB cos 36.87°


NA = 2.20 NB ...(Eq.1)
SV = 0: NA – 600 – 80 + NB cos 36.87° + 0.2 × NB sin 36.87° = 0
\ NA + 0.919 NB = 680
Substituting the value of NA from Eq. 1,
2.20 NB + 0.919 NB = 680  \ NB = 218 N
From Eq. (1), NA = 2.20 × 218  \ NA = 479.6 N
SMA = 0: NB × 5 – 600 × x cos 36.87° – 80 × 3 cos 36.87° = 0
Substituting the value NB,
218 × 5 – 600 × x cos 36.87° – 80 × 3 cos 36.87° = 0
Solving
\ x = 1.87 m  (Ans.)

Fig. Ex. 5(a)

Solved Examples Based on Wedge Friction


Example 1. The weight of the block A is 500 N. Neglecting the weight of wedge
B, determine the force P required to produce impending motion of block A. Assume
coefficient of static friction at all contact surfaces as 0.3. Refer Figure Ex. 1.

Fig. Ex. 1
Friction  5.27

Solution: Considering impending motion, apply conditions of equilibrium i.e.


+
→ΣH = 0, and +↑SV = 0 to the block A and wedge B separately. Free body
diagrams of the block and the wedge are shown in figure Ex 1(a). Here we have
to understand that the wedge is subjected to more number of unknowns compare
to block, therefore we apply equations of equilibrium to block first.
Block A: SH = 0: F2 cos 15° + N2 sin 15° – N3 = 0
Now for impending motion from (Eq. 5.1), we know Fmax = μs N.
0.3 × N2 cos 15° + N2 sin 15° = N3 ...(Eq.1)
Now, SV = 0: – F2 sin 15° + N2 cos 15° – 500 – F3 = 0
– 0.3 × N2 sin 15° + N2 cos 15° – 500 – 0.3 N3 = 0
Substituting value of N3 from Eq. 1,
– 0.3 × N2 sin 15° + N2 cos 15° – 0.3 (0.3 × N2 cos 15° + N2 sin 15°) = 500
Solving, N2 = 690.79 N
Substituting value of N2 in Eq. (1),
N3 = 378.95 N
Now, F2 = 0.3 × N2 = 0.3 × 690.79  \ F2 = 207.23 N
F3 = 0.3 × N3 = 0.3 × 378.95  \ F3 = 113.685 N
W

F3

N3
15° N
2
F2
P F2
N2 15°

F1
N1

Fig. Ex. 1(a)

Wedge B: SH = 0: P – F1 – F2 cos 15° – N2 sin 15° = 0


P = F1 + F2 cos 15° + N2 sin 15°
Now for impending motion from (Eq. 5.1), we know Fmax = μs N.
P = 0.3 N1 + F2 cos 15° + N2 sin 15° ...(Eq. 2)
Now, SV = 0: N1 – N2 cos 15° + F2 sin 15° = 0
Substituting values of N2 and F2 obtained in previous part,
\ N1 = 690.79 × cos 15° – 207.23 × sin 15°
\ N1 = 613.62 N
5.28  Engineering Mechanics Statics

Substituting value of N1, F2 and N2 in to Eq. 2,


P = 0.3 × 613.62 + 207.23 × cos 15° + 690.79 sin 15°
\ P = 563 N (considering approximate value)  (Ans.)
Alternate Approach: This example can also be solved by considering angle
of static friction, ∅s = tan–1 μs from (Eq. 5.4). Here consider free body diagrams
of the block and the wedge as shown in figure Ex 1(b). R1, R2 and R3 on free
body diagram are the resultant forces of normal reaction and friction force at that
surfaces. Angle of friction is measured in between normal reaction and resultant
force.
Here ∅s = tan–1 0.3 = 16.69°

Fig. Ex. 1(b)


Now considering equilibrium of block, apply Lami’s theorem to the free body
diagram shown in figure Ex 1(c).
500 R2 R3
= =
sin 138.38° sin 73.31° sin 148.31°
\ R2 = 721.08 N and R3 = 395.46 N

R1
R2
15°+ 

P

R3 15°+
R2

500 N
Fig. Ex. 1(c) Fig. Ex. 1(d)

Now considering equilibrium of wedge, apply Lami’s theorem to the free


body diagram shown in figure Ex 1(d).
P R2
=
sin 131.62° sin 106.69°
\ P = 563 N (considering approximate value)  (Ans.)
Friction  5.29

Example 2. Block A and B each of weight 150 N resting on horizontal surface


are to be pushed apart by 20° wedge as shown in Figure Ex. 2. Neglecting the
weight of the wedge determine the force P required to start movements of blocks.
Assume coefficient of static friction at all contact surfaces as 0.25.

Fig. Ex. 2

Solution: Considering impending motion, apply conditions of equilibrium


+
i.e. →ΣH = 0, and +↑SV = 0 to the blocks and wedge separately. Free body
diagrams of the blocks and the wedge are shown in figure Ex 2(a). Here we have
to understand that the wedge is subjected to more number of unknowns compare
to block, therefore we apply equations of equilibrium to block first. As two blocks
are identical we can consider only one block for analysis.
P

F2 10° 10° F2
N2 N2
A
N2 N2

10° F2 F2 10°

F1 F1

N1 N1
Fig. Ex. 2(a)

Block A: SH = 0: –F1 + N2 cos 10° – F2 sin 10° = 0


Now for impending motion from (Eq. 5.1), we know Fmax = μs N.
–0.25 N1+ N2 cos 10° – 0.25 N2 sin 10° = 0
\ N1 = 3.76 N2 ...(Eq.1)
Now, SV = 0: N1 – N2 sin 10° – 0.25 N2 cos 10° – 150 = 0
Substituting value of N1 from Eq. 1,
3.76 N2 – N2 sin 10° – 0.25 N2 cos 10° = 150
Solving, N2 = 44.91 N
5.30  Engineering Mechanics Statics

Substituting value of N2 in Eq. (1),


N1 = 168.86 N
Now, F2 = 0.25 × N2 = 0.25 × 44.91
\ F2 = 11.22 N
Wedge: SV = 0: 2 (F2 cos 10°) + 2 (N2 sin 10°) – P = 0
Substituting values of N2 and F2 obtained in previous part,
P = 2 (11.22 × cos 10°) + 2 (44.91 × sin 10°)
\ P = 37.68 N  (Ans.)
Example 3. A block of 1000 N is to be raised by means of the wedges A and B as
shown in Figure Ex. 3. Neglecting the weight of the wedges determine the force P
required to produce impending motion of the block upwards. Assume coefficient
of static friction at all contact surfaces as 0.25.

Fig. Ex. 3
Solution: Considering impending motion, apply conditions of equilibrium i.e.
+
→ΣH = 0, and +↑SV = 0 to the block and wedges separately. Free body diagrams
of the block and the wedge are shown in figure Ex 3(a). As two wedges are
identical we can consider only one wedge for analysis.

Fig. Ex. 3(a)


Friction  5.31

Wedge A: SH = 0: F2 cos 15° + N2 sin 15° – F2 cos 15° – N2 sin 15° = 0


Now, SV = 0: 2 (N2 cos 15°) – 2 (F2 sin 15°) – 1000 = 0
Now for impending motion from (Eq. 5.1), we know Fmax = μs N
2(N2 cos 15°) – 2 (0.25 × N2 sin 15°) = 1000
Solving, N2 = 555.55 N
Now, F2 = 0.25 × N2 = 0.25 × 555.55  \ F2 = 138.88 N
Wedge B: SH = 0: – P + 0.25 N1 + F2 cos 15° + N2 sin 15° = 0
Substituting values of N2 and F2 obtained in previous part,
P = 0.25 N1 + 138.88 cos 15° + 555.55 sin 15° ...(Eq.1)
SV = 0: N1 + 138.88 sin 15° – 555.55 cos 15° = 0
Solving, N1 = 500.68 N
Substituting values of N1 in Eq.1
P = 0.25 × 500.68 + 138.88 cos 15° + 555.55 sin 15°
\ P = 403.1 N  (Ans.)
Example 4. A block of 1000 N is rest on horizontal surface as shown in Figure
Ex. 4. Neglecting the weight of the wedge determine the force P required to
produce impending motion of the block. Assume coefficient of static friction at
all contact surfaces as 0.3.
P

1000 N 18°

Fig. Ex. 4

Solution: Considering impending motion, apply conditions of equilibrium


+
i.e. →ΣH = 0, and +↑SV = 0 to the block and wedge separately. Free body
diagrams of the block and the wedge are shown in figure Ex 4(a). Here we have
to understand that the wedge is subjected to more number of unknowns compare
to block, therefore we apply equations of equilibrium to block first.
5.32  Engineering Mechanics Statics

N2 N3
1000 N
F2
F3
F2
N2

F1

N1

Fig. Ex. 4(a)

Block: SH = 0: F1 – N2 = 0
\ F1 = N2
Now for impending motion from (Eq. 5.1), we know Fmax = μs N
\ 0.3 N1 = N2 ...(Eq.1)
Now, SV = 0: N1 – 1000 – F2 = 0
N1 – 1000 – 0.3 N2 = 0
Substituting value of N2 from Eq. 1
N1 – 0.3 (0.3 N1) = 1000
Solving, N1 = 1098.9 N
From Eq. (1), N2 = 329.6 N
Wedge: SH = 0: N2 – N3 cos 18° + F3 sin 18° = 0
N2 – N3 cos 18° + (0.3 N3) sin 18° = 0
Substituting values of N2 obtained in previous part,
329.6 – N3 cos 18° + (0.3 N3) sin 18° = 0
Solving, N3 = 384.46 N
SV = 0: – P + F2 + F3 cos 18° + N3 sin 18° = 0
P = F2 + F3 cos 18° + N3 sin 18°
P = 0.3 × N2 + 0.3 × N3 cos 18° + N3 sin 18°
Substituting values of N2 and N3
P = 0.3 × 329.6 + 0.3 × 384.46 cos 18° + 384.46 sin 18°
\ P = 327.37 N  (Ans.)
Example 5. A beam AB is supported by wedge as shown in Figure Ex. 5.
Neglecting the weight of the wedge and beam determine the force P required to
produce impending motion of the wedge towards left. Assume coefficient of static
friction at all contact surfaces as 0.3.
Friction  5.33

500 N 800 N

B
A P
10°
2m 2m
5m

Fig. Ex. 5

Solution: Considering impending motion, apply conditions of equilibrium i.e.


+
→ΣH = 0, +↑SV = 0 and SM = 0 (Treating anticlockwise moment as positive)
to the beam and wedge separately. Free body diagrams of the beam and the
wedge are shown in figure Ex 5(a). Here we have to understand that the wedge
is subjected to more number of unknowns compare to beam, therefore we apply
equations of equilibrium to beam first.
Beam: SH = 0: Ax – F2 cos 10° – N2 sin 10° = 0

Fig. Ex. 5(a)

Now,
SV = 0: Ay – 500 – 800 + N2 cos 10° – F2 sin 10° = 0
Now for impending motion from (Eq. 5.1), we know Fmax = μs N
Ay – 500 – 800 + N2 cos 10° – 0.3 N2 × sin 10° = 0 ...(Eq.1)
SMB = 0: – 6Ay + 800 × 2 + 500 × 4 = 0
Solving, Ay = 600 N
Substituting value of Ay in Eq. 1
600 – 500 – 800 + N2 cos 10° – 0.3 N2 × sin 10° = 0
Solving, N2 = 750.42 N
As F2 = 0.3 N2 = 0.3 × 750.42  \ F2 = 225.12 N
Wedge:
SH = 0: – P + F1 + F2 cos 10° + N2 sin 10° = 0
Substituting values of N2 and F2 obtained in previous part,
P = 0.3 N1 + 225.126 cos 10° + 750.42 sin 10° ...(Eq.2)
5.34  Engineering Mechanics Statics

SV = 0: N1 + F2 sin 10° – N2 cos 10° = 0


N1 + 225.12 sin 10° – 750.42 cos 10° = 0
Solving, N1 = 699.92 N
Substituting values of N1 in Eq. 2
P = 0.3 × 699.92 + 225.12 cos 10° + 750.42 sin 10°
\ P = 561.98 N  (Ans.)

Solved Examples Based on Belt Friction


Example 1. The 500 N block is attached to a rope that passes over a fixed
cylinder as shown in Figure Ex. 1. Determine the range of the horizontal force P
for which the system will be at rest. Assume coefficient of static friction between
the rope and the cylinder as 0.3.
P

500 N

Fig. Ex. 1

T2
Solution: Considering impending motion, apply (Eq. 5.10) = ems b to figure
T1
Ex 1(a). In this equation T2 is tension on higher side and
π
β= rad.
2
The maximum value of P for equilibrium occurs
when the block is on the verge of moving upward. For
this case, T2 = Pmax and T1 = 500 N
π 
0.3  
Pmax 2 
\ = e Fig. Ex. 1(a)
500
Solving, Pmax = 801 N  (Ans.)
The minimum value of P for equilibrium occurs when the block is on the
verge of moving downward. For this case, T2 = 500 N and T1 = Pmax
π 
0.3  
500 2 
\ = e
Pmin
Friction  5.35

Solving, Pmin = 312.30 N  (Ans.)


\ Range of horizontal force P is from 801 N to 312.30 N that can keep the
block in equilibrium.
Example 2. The 500 N block is attached to a rope that passed over a fixed
cylinder as shown in Figure Ex. 2. If P = 250 N, determine the minimum value of
angle α at which block begins to slip. Assume coefficient of static friction between
the rope and the cylinder as 0.3.

Fig. Ex. 2

T
Solution: Considering impending motion, apply (Eq. 5.10) 2 = ems b to figure
Ex 2(a). T1
Here T2 = 500 N and T1 = 250 N 
500
\ = e0.3 b 250 N
250
500
In = 0.3 β
250
 180  500 N
Solving, β = 2.31 rad = 2.31  
 π  Fig. Ex. 2(a)
β = 132.38°
Now, α = 132.38° – 90°  \ α = 42.4°  (Ans.)
Example 3. The 200 N block is suspended from a rope that passes over two
fixed cylinder as shown in Figure Ex. 3. Determine the smallest vertical force P
required to exert on the rope to support the block. Assume coefficient of static
friction between the rope and the right cylinder as 0.2 and between the rope and
the left cylinder as 0.3.

200 N
P

Fig. Ex. 3
5.36  Engineering Mechanics Statics

T2
Solution: Considering impending motion, apply (Eq. 5.10) = emsb to the right
T1
cylinder of figure Ex. 3(a). In this equation T2 = 200 N, T1 = T (tension in the rope
π
between two cylinders) and β = rad.
2
T T

P 200 N

Fig. Ex. 3(a)

π 
0.2  
200 2 
\ = e
T
Solving, T = 146.08 N
T
Now apply (Eq. 5.10) 2 = emsb to the left cylinder of figure Ex. 3(a). In this
T1
π
equation T2 = T, T1 = P and β = rad.
2
π 
0.3  
146.08 2 
\ = e
P
Solving, P = 91.18 N  (Ans.)
Example 4. Two blocks of 175 N and 350 N respectively
are suspended from a rope that passes over a fixed cylinder
as shown in Figure Ex. 4. Determine minimum coefficient 175 N
of friction between rope and the cylinder that keep system 350 N
in equilibrium.
Fig. Ex. 4
Solution: Considering impending motion, apply (Eq. 5.10)
T2
= emsb to figure Ex. 4(a).
T1
Here T2 = 350 N, T1 = 175 N and β = π rad.
350
\ = em(p) 175 N
175 350 N
Solving, μ = 0.2206  (Ans.) Fig. Ex. 4(a)

Example 5. Block A of weight 200 N connected to block B of weight w using


a rope that passes over a fixed cylinder as shown in Figure Ex. 5. Determine
minimum weight of block B for which no motion occurs. Assume coefficient of
static friction between the rope and the cylinder as 0.2 and between the block A
and horizontal surface as 0.3.
Friction  5.37

140 N
A

Fig. Ex. 5

Solution: Consider impending motion of the block A towards right and apply
+
conditions of equilibrium i.e. →ΣH = 0, and +↑SV = 0 to the free body diagram
of the block A as shown in figure Ex. 5(a)
SH = 0: 140 – F – T = 0
Now for impending motion from (Eq. 5.1), 200 N
we know Fmax = μs N.
T = 140 – 0.3 N ...(Eq.1) T 140 N
SV = 0: N – 200 = 0
\ N = 200 N
F
Substituting value of N in Eq. 1,
N
T = 140 – 0.3 × 200
\ T = 80 N Fig. Ex. 5(a)
Now considering impending motion, apply
T
(Eq. 5.10) 2 = emsb to figure Ex. 5(b).
T1

Fig. Ex. 5(b)

Here T2 = T (tension in the rope between block A and cylinder), T1 = w and


π
β= rad.
2
π 
0.2  
80 2
\ = e  
w
Solving, w = 58.43 N  (Ans.)
5.38  Engineering Mechanics Statics

SUMMARY
• Frictional force is tangent to the surfaces of contact of the two bodies.
• Coulomb’s law of friction before sliding is,
F < μs N
• Coulomb’s law of friction at impending motion is,
Fmax = μs N
• Coulomb’s law of friction after sliding is
F = μk N
• Maximum frictional force Fmax is called as limiting friction.
• At limiting friction body is said to have impending motion.
• The angle of friction is defined as,
∅ = tan–1 μ
• At impending motion, Angle of repose (α) = Angle of static friction (∅s).
• At impending slip between cable/ rope and cylinder, the ratio of tension
on high side and tension on low side are related as,
T2
= eμsβ
T1

PROBLEMS

1. A 300 N block is subjected to force 400 N


of 400 N as shown in figure Prob. 1.
Determine the friction force between the 25°
block and the surface. Assume ms = 0.3 and
mk = 0.25.
Fig. Prob. 1
2. The 10 N force as shown in figure Prob. 2 10 N
is required to produce impending motion 20°
of the block up the inclined surface. If the
weight of the block is 5 kg, determine the
coefficient of static friction (ms) between
the block and the inclined surface. 30°
Fig. Prob. 2
3. The 1 kg block is subjected to the load of 2 Kg
2 kg as shown in figure Prob. 3. Determine
the friction force between the block and the
inclined surface. Assume ms = 0.9. 1 Kg

2
4
Fig. Prob. 3
Friction  5.39

4. Block A of 100 N and block B of 400 N


are connected with cable that passes over a
frictionless pulley as shown in figure Prob. 4. A
At impending motion of the blocks,
determine coefficient of static friction B
between block B and inclined surface.
Assume ms = 0.25 between block A and B. 30°

Fig. Prob. 4
5. Block A of 200 N and block B of 100 N P
are connected with cable that passes over a
frictionless pulley as shown in figure Prob. 5.
Determine the range of force P that keep A
A
blocks in equilibrium. Assume ms = 0.20
between block A and inclined surface. 30°

B
Fig. Prob. 5
6. Block A weight 100 N and block B weight 50 N
60 N. Horizontal force P is acting to block
A as shown in figure Prob. 6. Determine, 45°
(a) if the system is in equilibrium at P = B
40 N, (b) maximum value of P that keep
the system in equilibrium. Assume ms = P
A
0.30 between block A and B and ms = 0.25
between block A and horizontal surface.
Fig. Prob. 6
7. A crate of weight 500 N is subjected 1m
to horizontal force P as shown in figure
Prob. 7. Determine the force P required to
cause tipping of the crate also determine
P
coefficient of static friction for tipping. 500 N 2m
1m

Fig. Prob. 7
8. A 300 N block is resting on an inclined
P
surface and subjected to the force P
m

as shown in figure Prob. 8. Determine 50°


0m

maximum value of force P for equilibrium


50

of block and also check whether the


impending motion will be by tipping or by
slipping. Assume ms = 0.50 between block 80 20°
0m
and inclined surface. m
Fig. Prob. 8
5.40  Engineering Mechanics Statics

9. The rod AC of mass 5 kg rest as shown C


in figure Prob. 9. Determine coefficient of
m B
statics friction for impending motion of the 1.2
rod. 0.5 m
30°
A
Fig. Prob. 9

10. The rod AB of 3 m and weight 5 kg


is placed as shown in figure Prob. 10.
Determine maximum value of angle θ at
which the rod can be placed before it slip.
Assume ms = 0.25 for all contact surfaces.

Fig. Prob. 10
11. The weight of block A is 500 N and P
block B is 50 N. Determine the force P
required to produce impending motion of
block A. Assume = 0.30 for all contact B
surfaces. Refer figure Prob. 11. A

70°

Fig. Prob. 11
12 The weight of block A is 800 N and
wedge B is 400 N. Determine the force A
P required to produce impending motion
of block A. Assume = 0.20 for all contact 8°
surfaces. Refer figure Prob. 12. P
B

Fig. Prob. 12
13. A rope is wrapped around a rod as
shown in figure Prob. 13. If tension on one
end of rope is 180 N and on other end is
20 N
20 N, determine the least number of turns 180 N
of rope around rod in order to prevent Fig. Prob. 13
slipping of the rope. Assume ms = 0.20
between rope and rod.
14. Determine the range of weight W that
may be applied without causing the 250 N
block to slip. Assume ms = 0.25 between
block and inclined surface and ms = 0.30
between the rope and the cylinder. W
40°

Fig. Prob. 14
Unit 6

In this unit, we discuss concept of stress and strain. We study the


stress induced in the member due to axial loading. We also discuss
the deformation caused by the load. Systematically understanding of
these concepts is important to the design engineers.
Simple Stress & Strain
6.1 stress
All bodies offer an equal internal resistance to the Stress is internal resistance
externally applied forces. The magnitude of the set up by a body when it
resisting force per unit area is called stress. The is deformed.
stress is denoted by Greek letter sigma (σ).
P
s = ...(Eq. 6.1)
A
In above equation P is external force expresses in Newton (N) and A is original
cross section area of the specimen and expresses in square meter or millimetre
(m2 or mm2). So, SI unit of stress is N/m2 or N/mm2.
1 N/m2 is called Pascal (Pa).
1 kPa = 103 Pa = 103 N/ m2
1 MPa = 106 Pa = 106 N/ m2 = 1 N/mm2
1 GPa = 109 Pa = 109 N/ m2
Figure 1 shows square bar subjected to pull force P and internal resistance
at cutting section.

Fig. Ex. 1

6.1.1 Types of Stress
Generally there are two types of stress, (a) Normal Stress (b) Shear Stress
(a) Normal Stress: The stress developed on a plane normal to it is called
normal stress. It is equal to the force acting on the body per unit normal
area. A normal stress is a stress that occurs when a member is loaded by
an axial force and member is placed in tension or compression. If the
member is subjected to axial tension, the stress developed at a section is
called tensile stress.
6.4  Engineering Mechanics Statics

If the member is subjected to axial compression, the stress developed at a


section is called compressive stress.

(b) Shear Stress: The stress developed when a member is subjected to a


force that is parallel or tangent to the surface is called shear stress. It is
denoted by Greek letter tau (τ). It is obtained by dividing the magnitude
of the resultant shear force (V) by the cross sectional area (A)
V
t = ...(Eq. 6.2)
A
6.2 Strain
If Lo is the initial gauge length and L is the
observed length under a given load, the gauge Strain is defined as the
elongation, dL = L – Lo as shown in figure 2. The change in length per unit
elongation (or contraction) per unit of the initial length.
gauge length is given as:
δL
e = ...(Eq. 6.3)
L0
Simple Stress & Strain  6.5

Fig. 2

This expression defines the tension (or compression) strain. Since this is
associated with the normal stress, it is usually called the normal strain. It is a
dimensionless quantity (mm/mm). This is also called the nominal or engineering strain.

6.2.1 Shear Strain
Consider a block OABC fixed on horizontal surface A A
P
B B
as shown in figure 3. It is subjected to force P
acting on its upper face. The deformation of the
block because of force P appears as OA’B’C.
Now shear strain is defined as, 
BB′
Shear strain = = tan φ O C
CB
Fig. 3

6.2.2  Longitudinal and Lateral Strains


Consider a circular test specimen of diameter d and
length L as shown in figure 4. If a tensile load P is applied
to the specimen its length increases. The increase in
length per unit length is called the longitudinal strain.
Since the volume of the specimen remains constant,
therefore, the increase in length is accompanied by a
decrease in diameter. This decrease in diameter per unit
diameter is called the lateral strain. The longitudinal
and lateral strains are of opposite nature.
δL
eLongitudinal =
L
δd
eLateral = ...(Eq. 6.4)
d
Fig. 4
6.6  Engineering Mechanics Statics

6.3  Poisson’s Ratio


The ratio of the lateral strain to the longitudinal strain is called the Poisson’s ratio.
It is denoted by v (Greek letter Nue).
Poisson’s ratio for most
Lateral strain
Poisson’s ratio = of the materials varies
Longitudinal strain from 0.25 to 0.40.
6.4  Volumetric Strain
It is the ratio of the change in volume of the body to its original volume V0, when
subjected to hydrostatic stress.
∆V
Volumetric strain eV = ...(Eq. 6.5)
V

6.5 Hooke’s Law And Elastic Moduli


According to this Law, stress is directly proportional to strain, within the elastic
limits.
σ ∝ ε
or σ = Eε
(a) Modulus of Elasticity, E: It is defined as the ratio of normal stress to
normal strain within the elastic limits. In above equation the constant of
proportionality, E is called modulus of Elasticity.
This equation can be used to find out change in length of the bar as below
P
σ A
E = =
ε δL
L
PL
dL = ...(Eq. 6.6)
AE
(b) Modules of Rigidity, G: It is defined as the ratio of shearing stress to
shearing strain.
τ
G = ...(Eq. 6.7)
γ
(c) Bulk Modulus, K: It is the ratio of hydrostatic stress to volumetric strain.

6.6  Bar of Varying Cross-Section


Consider a bar of varying cross-section shown in figure 5. The free body diagrams
for the various parts are shown in figure 6. The stresses in the various parts are:
P P −P P
sAB = 1 , σ BC = 1 2 , σCD = 3
A1 A2 A3
Simple Stress & Strain  6.7

Fig. 5

Fig. 6

The total elongation in the bar is find out as


n
PL
δL = ∑ Ai Ei ...(Eq. 6.8)
i =1 i i

6.7  Stress-Strain Diagram For Mild Steel


Stress-Starin diagram is a graphical representation of the relationship between
stress (σ) and strain (ε). It shows behaviour of material under loading. The curve
varies from material to material.
The stress-strain diagram for a ductile material like mild steel is shown in
figure 7. The curve starts from the origin, showing thereby that there is no initial
stress of strain in the specimen.

Fig. 7
6.8  Engineering Mechanics Statics

Salient Point of the Diagram


A: Upto point A, Hooke s law is obeyed and  stress is proportional to strain or
elongation is proportional to the load. Therefore, OA is a straight line. Point A is
called the limit of proportionality or proportional limit.
B: Up to point B, the material remains elastic, i.e.  on removal of the load,
deformations are completely recovered. AB is not a straight line. Point B is called
the elastic limit point. Beyond point B, the material goes to the plastic stage until
the upper yield point C is reached.
C & D: Beyond point B plastic deformation occurs and material is not totally
recoverable. Here permanent deformation takes place when load is removed. At
this point the cross-sectional area of the material starts decreasing. Points C & D
are termed as upper and lower yield points respectively. The stress at the yield
point is called the yield strength.
E: Between DE, the specimen elongates by a considerable amount without any
increase in stress. Point E corresponds to ultimate strength of a material. The
maximum load which a specimen can withstand without failure is known as load
at ultimate strength.
F: Beyond point E, the bar begins to form neck and its cross-sectional area
decreases at a rapid rate. The apparent stress deceases but the actual or true
stress goes on increasing until the specimen breaks at point F, called the point of
fracture. The fracture of ductile material is of the cup and cone type.

6.8  Temperature Stress and Strain


When the temperature of a material is changed, its dimensions also change. A
stress is setup in the material for change in dimension due to temperature change
is prevented. This is called the temperature stress.
Let,
L be the length of the member at temperature t,
dt is the change in temperature,
α is the coefficient of linear expansion for the material change in length
Now, δL = αLδt
Expanded length = L (1 + α dt)
α L δt
\ Temperature strain et =
L(1 + α δt )
\ Temperature strain et = adt ...(Eq. 6.9)
Temperature stress st = et E
at = a dt E ...(Eq. 6.10)
When the temperature increases, length increases. Since this increase in
length is prevented, compressive stress is developed in the material. The reverse
phenomena occur when the temperature is decreased and tensile stress is developed.
Simple Stress & Strain  6.9

6.9  Composite System


Composite system consists of two or more bar of different materials in parallel.
In such system the sharing of load by each material can be found using equilibrium.
Consider a composite system shown in figure 8
consisting of different materials and area of cross- P

section, subjected to load P.


P = P1 + P2 + P3
1 2 3
Now using following compatibility equation we
A1 A2 A3 L
can determine load or stresses in each materials.
P1L PL PL
= 2 = 3
A1E1 A2 E2 A3 E3
Fig. 8
6.10  Factor Of Safety
It is the ratio of the maximum permissible stress to which a member can be
subjected to the allowable or working stress.
ultimate load
Factor of safety =
allowable load
ultimate stress
Factor of safety =
allowable stress

Solved Examples Based on Stress and Strain


Example 1. A metal wire is 5 mm diameter and 2 m long is subjected to 24 N
tensile force that stretches 0.3 mm. Determine stress and strain in the wire.
Solution: Area of cross-section A = π r2 , where r is radius of wire
\ A = π × (2.5)2 = 19.634 mm2
P 24
Now we know, s = =
A 19.634
\ s = 1.22 N/mm2  (Ans.)
δL 0.3
Now we know, e = = = 0.00015   (Ans.)
L0 2000
Example 2. A circular rod of diameter 8 mm and 300 mm long is subjected to
a tensile force 20 kN. Take E = 200 kN/mm2. Find stress, strain and elongation of
the bar due to applied load.
Solution: Area of cross-section A = πr2, where r is radius of wire
\ A = π × (4)2 = 50.26 mm2
P 20 × 1000
Now we know, s = =
A 50.26
6.10  Engineering Mechanics Statics

\ s = 397.93 N/mm2  (Ans.)


σ 397.93
Now we know, e = =
E 200 × 1000
\ e = 0.001989  (Ans.)
dL = e × L = 0.001989 × 300
\ dL = 0.5968 mm  (Ans.)
Example 3. A hollow circular tube of 200 mm long is subjected to compressive
load of 26 kN. The inner and outer diameters of the tube are 40 mm and 45 mm
respectively. The shortening of the tube due to load is 0.12 mm. Determine stress
and strain in the tube.
Solution: Area of cross-section A = π (outer radius2 – inner radius2)
\ A = π × (22.52 – 202) = 333.79 mm2
P 26 × 1000
Now we know, s = =
A 333.79
\ s = 77.89 N/ mm2  (Ans.)
δL 0.12
Now we know, e = = = 0.0006   (Ans.)
L0 200
Example 4. A rectangular bar of 100 mm × 40 mm and 2 m long is subjected to
an axial tensile load of 60 kN. If the extension in the length of the bar is 1 mm,
determine stress, strain and modulus of elasticity of the bar.
Solution: Area of cross-section A = 100 × 40 = 4000 mm2
P 60 × 1000
Now we know, s = =
A 4000

\ s = 15 N/mm2  (Ans.)
δL 1
Now we know, e = = = 0.0005   (Ans.)
L0 2000
σ 15
Now, E = = = 30 × 103 N/mm 2   (Ans.)
ε 0.0005
Example 5. A steel specimen of 10 mm diameter with a gauge length of 200
mm is tested to destruction. It has an extension of 0.50 mm under a load of 40
kN and the load at elastic limit is 55 kN. The maximum load is 70 kN. The total
extension at fracture is 28 mm and diameter at neck is 7 mm. Find (i) The stress
at elastic limit. (ii) Young’s modulus. (iii) Percentage elongation. (iv) Percentage
reduction in area. (v) Ultimate tensile stress.
Solution: Area of cross-section A = π × (5)2 = 78.53 mm2
P 55 × 1000
(i) Stress at elastic limit, s = =
A 78.53
\ s = 700.36 N/ mm2  (Ans.)
Simple Stress & Strain  6.11

δL 0.50
Now we know, e = = = 0.0025
L0 200
P 40 × 1000
E = A = 78.53 = 203744 N/mm   (Ans.)
2
(ii)
ε 0.0025
Total extension 28
(iii) Percentage elongation = = = 14%   (Ans.)
L0 200
Initial area – Final area
(iv) Percentage reduction in area =
Initial area

π(5) 2 − π(3.5) 2
= = 51%   (Ans.)
π(5) 2
Maximum load
(v) Ultimate tensile stress =
Area
70 × 1000
= = 891.37 N/mm 2   (Ans.)
78.53
Example 6. A steel wire 1.5 m long and 5 mm in diameter is extended by
0.50 mm due to weight suspended from the wire. If the same weight is suspended
from the brass wire, 3 m long and 3 mm in diameter, it is elongated by 5.35 mm.
Determine the modulus of elasticity of brass if that of steel is 2 × 105 N/mm2
Solution: For the type of given loading, a change in the length of wire is given by,
PL
dl =
AE
Now for the steel wire,
P ×1500
0.50 = = 0.0003819 P
π(2.5) 2 × (2 × 105 )
\ Weight suspended P = 1309.24 N
Now for the brass wire,
1309.24 × 3000
5.35 =
π(1.5) 2 × EB
\ EB = 103861.7 N/mm2  (Ans.)

Example 7. A solid circular steel rod of 25 mm in diameter and 300 mm long


is rigidly fastened to the end of a square brass bar of 10 × 10 mm and 250 mm
long as shown in figure Ex. 7. An axial tensile force of 20 kN is applied at each
of extreme ends. Determine the elongation of assembly. Take Es = 200 GPa and
EB = 90 GPa.
6.12  Engineering Mechanics Statics

Fig. Ex. 7

Solution: Let dlb and dls be the elongations in brass and steel. Then total increase
in length of the assembly is,
PL PL
dl = δlb + δls = b b + s s
Ab Eb As Es
20 ×1000 × 250 20 ×1000 × 300
= +
(10 ×10) × (90 ×1000) π(12.5)2 × (200 ×1000)
dl = 0.555 + 0.0611 = 0.616 mm  (Ans.)
Example 8. A member PQRS of uniform diameter 250 mm has been subjected
to point loads as shown in figure Ex. 8. Determine the net change in the length of
the bar. Take E = 200 × 109 N/m2.

Fig. Ex. 8

Solution: Cross-sectional area A = p × (125)2 = 49087.38 mm2


The free body diagram of each segment of the member are shown in figure
Ex. 8(a).

Fig. Ex. 8(a)

(150 × 1000) × (150)


Segment PQ; dl1 =
49087.38× (200 × 109 )
Simple Stress & Strain  6.13

(60 × 1000) × (400)


Segment QR; dl2 =
49087.38× (200 × 109 )
(180 × 1000) × (150)
Segment RS; dl3 =
49087.38× (200 × 109 )
Total contraction in length,
dl = dl1 + dl2 + dl3
= 7.48 × 10–9 mm  (Ans.)
Example 9. A straight bar of 900 mm long is 40 mm in diameter for the first
500 mm length and 20 mm diameter for the remaining length. If the bar is subjected
to an axial pull of 20 kN find the extension of the bar. Take E = 2 × 105 N/mm2.
Solution: Here extension of the bar dl = dlb + dls
PL1 PL2
= +
A1E A2 E
20 ×1000 × 500 20 ×1000 × 400
= +
π(20) × (2 ×10 ) π(10) 2 × (2 ×105 )
2 5

dl = 0.0397 + 0.1273 = 0.524 mm  (Ans.)


Example 10. A bar made up of two square sections. A steel of 15 × 15 mm
square cross-section and 300 mm length and aluminium bar of 30 × 30 mm and
400 mm long. The bar is subjected to a compressive force P. If total decrease in
length of the bar is 0.15 mm, than determine the value of P. Take E for steel as
205 × 103 N/mm2 and for aluminium 75 × 103 N/mm2.
PLS PLA
Solution: Here extension of the bar dl = +
AS ES AA E A

 300 400 
0.15 = P  +
15×15× 205×1000 30 × 30 × 75×1000 
Solving, p = 12 kN  (Ans.)
Example 11. A rectangular block of 20 mm wide and 10 mm deep and 100 mm
long is subjected to 40 kN tensile load. Measurement show that the elongation of
the block is 0.0700 mm and decrease in width is 0.00500 mm. Determine modulus
of elasticity and Poisson’s ratio.
Solution: Cross-sectional area A = 10 × 20 = 200 mm2
P 40 × 1000
Now we know, s = =
A 200
\ s = 200 N/ mm2
δL 0.0700
Longitudinal strain e = = = 0.0007
L0 100
6.14  Engineering Mechanics Statics

σ 200
Now, E = = = 285714 N/mm 2   (Ans.)
ε 0.0007
δw 0.00500
eLateral = = = 0.00025
w 20
Lateral strain
Poisson’s ratio =
Longitudinal strain
0.00025
= = 0.357   (Ans.)
0.0007
Example 12. A steel bar of 100 mm wide, 25 mm thick and 500 long is subjected
to an axial pull of 90 kN. Find the change in length, width and thickness of the
bar. Take E = 2 × 105 N/mm2 and Poisson’s ratio = 0.30.
Solution: Cross-sectional area A = 100 × 25 = 2500 mm2
P 90 × 1000
Now we know, s = =
A 2500
\ s = 36 N/ mm2
σ 36
Longitudinal strain e = = = 0.00018
E 2 × 105
Lateral strain = Poisson’s ratio × Longitudinal strain
= 0.30 × 0.00018 = 0.000054
δL
Now, e =
L0
\ dL = e × Lo
Change in length = 0.00018 × 500 = 0.09 mm  (Ans.)
δw
eLateral =
w
\ dw = eLateral × w
Above equation shows that change in width = lateral strain × original width
\ Change in width 0.00054 × 100 = 0.054 mm  (Ans.)
Now change in thickness = lateral strain × original thickness
= 0.00054 × 25 = 0.0135  (Ans.)
Example 13. A rod of length 150 mm and diameter 10 mm is subjected to an
axial pull of 10 kN. Determine the change in the dimension of the rod. Take E =
2 × 105 N/mm2 and Poisson’s ratio = 0.30.
Solution: Cross-sectional area A = π × r2 = π × 52 = 78.53mm2
PL 10 ×1000 ×150
Change in length of the rod dl = =
AE 78.53× 2 ×105
\ dl = 0.095 mm  (Ans.)
Simple Stress & Strain  6.15

δL 0.095
Longitudinal strain e = = = 0.000633
L0 150
Lateral strain
Now, Poisson’s ratio =
Longitudinal strain
Lateral strain = Poisson’s ratio × Longitudinal strain
= 0.30 × 0.000633 = 0.000189
δd
Also, lateral strain eLateral =
d
dd = eLateral × d
dd = 0.000189 × 10 = 0.0018 mm  (Ans.)
Example 14. An aluminium rod of 25 mm diameter and 150 mm long is
subjected to tensile load of 50 kN. The elongation of the rod is 0.1095 mm and
its diameter is reduced by 0.00607 mm. Determine Poisson’s ratio of the material.
δL 0.1095
Solution: Longitudinal strain e = = = 0.00073
L0 150
δd 0.00607
Lateral strain eLateral = = = 0.000242
d 25
Lateral strain
Now, Poisson’s ratio =
Longitudinal strain
0.000242
Poisson’s ratio = = 0.33   (Ans.)
0.00073
Example 15. A hard rubber bar is deformed by 0.5 mm as shown in figure
Ex. 15 by dash line due to force P. Determine the A A P B B
shear strain at point O.
Solution: The shear strain is represented by the
angle between AOA1. 150
mm
Shear strain

–1 AA′ 0.5 O C
∅ = tan = tan –1 = 0.19°   (Ans.) Fig. Ex. 15
OA 150

Example 16. A composite bar of a circular rod of copper of 50 mm diameter is


fixed into a steel tube of internal diameter 50 mm and external diameter 80 mm.
If the composite bar of length L is subjected to 150 kN load, determine the stress
developed in the two materials. Take E for steel = 2 × 105 N/mm2 and E for copper
= 1.2 × 105 N/mm2
Solution: Area of cross-section of copper rod AC = π × (25)2 = 1963.49 mm2
Area of cross-section of steel tube AS = π × (402 – 252) = 3063 mm2
6.16  Engineering Mechanics Statics

Now from equation of equilibrium,


150 kN load = Load on copper rod + Load on steel tube
150 × 1000 = PC + PS ...(Eq. 1)
From compatibility condition,
PC L PL
= S
AC EC AS ES
PC PS
5 =
1963.49 × 1.2 × 10 3063 × 2 × 105
PC = 0.3846 PS ...(Eq. 2)
Substituting in Eq. 1
150 × 1000 = 0.3846 PS + PS
PS = 108334.5 N
And, PC = 0.3846 × 108334.5 = 41665.46 N
Now, Stress in copper
41665.46
sC = = 21.22 N/mm 2   (Ans.)
1963.49
108334.5
Stress in steel sS = = 35.36 N/mm 2   (Ans.)
3063
Example 17. A composite tube consist of a steel tube 300 mm internal diameter
and 20 mm thickness and an outer brass tube 340 mm internal diameter and 20 mm
thickness. The composite tube of length L is subjected to an axial load of 800 kN.
Find the stress developed in two materials. Take E for steel = 2 × 105 N/mm2 and
E for brass = 1 × 105 N/mm2.
Solution: Area of cross-section of steel tube AS = π × (1702 – 1502) = 20106.19 mm2
Area of cross-section of brass tube AB = π × (1902 – 1702) = 22619.46 mm2
Now from equation of equilibrium,
800 kN load = Load on steel tube + Load on brass tube
800 × 1000 = PS + PB ...(Eq. 1)
From compatibility condition,
PB L PL
= S
AB EB AS ES
PB PS
=
22619.46 × 1 × 105
20106.19 × 2 × 105
PB = 0.5625 PS ...(Eq. 2)
Substituting in Eq. 1
800 × 1000 = 0.5625 PS + PS
Simple Stress & Strain  6.17

PS = 512000 N
And, PB = 0.5625 × 512000 = 288000 N
Now, Stress in brass
288000
sB = = 12.73 N/mm 2   (Ans.)
22619.46
512000
Stress in steel sS = = 25.46 N/mm 2   (Ans.)
20106.19
Example 18. Two aluminium rod and one copper rod together support a load of
125 kN as shown in figure Ex. 18. If area of each aluminium rod is 1000 mm2 and
that of copper is 1200 mm2. Determine stresses in each rod. Take E for aluminium
= 1 × 105 N/mm2 and E for copper = 1.2 × 105 N/mm2.
125 KN

A C A
150 mm

200 mm

Fig. Ex. 18

Solution: Area of cross-section of aluminium rod AA = 1000 mm2


Area of cross-section of copper AC = 1200 mm2
Now from equation of equilibrium,
125 kN load = 2 (Load on aluminium rod) + Load on copper rod
125 × 1000 = 2PA + PC ...(Eq. 1)
From compatibility condition,
PA LA P L
= C C
AA E A AC EC
PA × 150 PC × 200
5 =
1000 × 1 × 10 1200 × 1.2 × 105
PA = 0.9259 PC ...(Eq. 2)
Substituting in Eq. 1
125 × 1000 = 2 × 0.9259 PC + PC
PC = 43831.16 N
6.18  Engineering Mechanics Statics

and, PA = 0.9259 × 43831.16 = 40583.27 N


43831.16
Now, Stress in copper sC = = 36.52 N/mm 2   (Ans.)
1200
40583.27
Stress in aluminium sA = = 40.58 N/mm 2   (Ans.)
1000
Example 19. A rod is 1 m long at 15°C. Determine the expansion of the rod
when the temperature is raised to 90°. Take E = 1 × 105 N/mm2 and coefficient
of thermal expansion α = 0.000014 per °C. If the expansion is prevented find the
stress induced in the material.
Solution: Here rise in temperature from initial temperature is, T = 75°C.
Expansion of the rod = α T L
= 0.000014 × 75 × 1 × 1000 = 1.05 mm  (Ans.)
Stress induced in the rod st = α T E
= 0.000014 × 75 × 1 × 105 = 105 N/ mm2  (Ans.)
Example 20. A steel rod of length 2 m and diameter 15 mm is at temperature
of 10°C. Determine the force exerted by the rod when prevented for expansion at
temperature 50°C. Take E = 2 × 105 N/mm2 and α = 0.000012 per °C.
Solution: Here rise in temperature from initial temperature is T = 40°C.
Expansion of the rod = α T L = 0.000012 × 40 × 2 × 1000 = 0.96 mm
Stress induced in the rod st A = α T E = 0.000012 × 40 × 2 × 105 = 96 N/ mm2
Force exerted by the rod = st A = 96 × p (7.5)2 = 16964.6 N  (Ans.)

SUMMARY
• Stress is internal resistance setup by a body when it is deformed.
P
Mathematically σ = and its unit is N/mm2
A
Normal stress and Shear stress are types of stress.

The stress developed on a plane normal to it is called normal stress.
Tensile stress and compressive stress are types of normal stress.
The stress developed when a member is subjected to a force that is parallel
or tangent to the surface is called shear stress.
V
Mathematically τ =
A
• Strain is defined as the change in length per unit length.
δL
εLongitudinal =
L
δd δw δb
εLateral = = =
d w b
Simple Stress & Strain  6.19

• The ratio of the lateral strain to the longitudinal strain is called the
Poisson’s ratio.
• The ratio of the change in volume of the body to its original volume is
known as volumetric strain.
• The ratio of normal stress to normal strain is constant and this constant is
known as modulus of elasticity.
• The ratio of shearing stress to shearing strain is known as modulus of
rigidity.
• Elongation of the bar due to axial loading is given by,
PL
δL =
AE
• A stress is setup in the material for change in dimension due to temperature
change is prevented. This is called the temperature stress.
Temperature stress σt = α δt E
Temperature strain εt = α δt

PROBLEMS
1. A rectangular bar having a cross-sectional area of 100 mm2 has a tensile force of
20 kN applied to it. Determine the stress in the bar.
2. A circular wire has a tensile force of 30.0 N applied to it and this force produces
a stress of 2.06 MPa in the wire. Determine the diameter of the wire.
3. A square-sectioned support of side 24 mm is loaded with a compressive force of
20 kN. Determine the compressive stress in the support.
4. A metal bar which is part of a frame is 30 mm diameter and 250 mm long. It
has tensile force acting on it of 35 kN which tends to stretch it. The modulus of
elasticity E is 205 × 109 N/m2. Calculate the stress and strain in the bar and the
amount it stretches.
5. A bolt having a diameter of 10 mm is loaded so that the shear stress in it is
200 MPa. Determine the value of the shear force on the bolt.
6. A 2.5 m long and 15 mm diameter steel bar is stretched by 20 mm on application
of an axial load of 800 N. Calculate the stress, strain and Young’s modulus.
7. A rectangular metal bar has a width of 20 mm and can support a maximum
compressive stress of 25 MPa; (a) determine the minimum breadth of the bar
when loaded with a force of 5 kN. (b) If the bar is 1.5 m long and decreases in
length by 0.25 mm when the force is applied, determine the strain.
8. A mild steel specimen of cross-sectional area 200 mm2 and gauge length 100 mm
is subjected to a tensile test and the following data is obtained: within the limit
of proportionality, a load of 70 kN produced an extension of 0.150 mm, load at
yield point is 90 kN, maximum load on specimen is 125 kN, final cross-sectional
6.20  Engineering Mechanics Statics

area of waist at fracture is 79 mm2, and the gauge length had increased to
105 mm at fracture. Determine for the specimen: (a) Young’s modulus of elasticity,
(b) the yield stress, (c) the tensile strength, (d) the percentage elongation, and
(e) the percentage reduction in area.
9. A load of 40 kN is applied to a steel wire. If the unit stress in the wire must not
exceed 90 N/mm2 what is the minimum diameter of the wire is required? What
will be the extension of 3.80 metre length of wire? Take E = 2 × 105 N/mm2.
10. A rod of length 150 mm and diameter 10 mm is subjected to an axial force of
15 kN. The modulus of elasticity of the material is 2 × 105 N/mm2 and Poisson’s
ratio is 0.30. Calculate longitudinal and lateral strain.
11. A brass bar having cross-sectional area of 750 mm2 is subjected to axial force as
shown in figure prob. 11. Find the total elongation of the bar. Take E = 1 × 105 N/
mm2.

Fig. Prob. 11

12. A composite rod is 800 mm long, its two ends are 30 mm2 and 20 mm2 in area
and Length are 300 mm and 200 mm respectively. The middle portion of the rod
is 15 mm2 in area. If the rod is subjected to an axial tensile load of 20 kN, find its
total elongation. Take E = 200 GPa
13. Determine the total strain in a bar made up of 50 mm diameter solid for a length
of 200 mm and a hollow circular section of outer diameter 45 mm and inner
diameter of 25 mm for a length of 120 mm. Take E = 200 kN/mm2. The axial load
is 80 kN.
14. A tensile stress is to be applied along the axis of a cylindrical rod that has a
diameter of 20 mm. Determine the magnitude of the load required to produce
0.0045 mm change in diameter. Take E = 97 × 103 N/mm2 and poisons ratio is
0.34.
15. A hollow steel rod of outer diameter 20 mm, inner diameter 12 mm and length
2 m is subjected to compressive load of 40 kN. Determine, (i) change in
length of the rod (ii) lateral strain, and change in diameters of the rod. Take
E = 200 × 103 N/mm2 and poisons ratio = 0.30.
16. A reinforced concrete column is 200 × 200 mm in cross-section and carries a load
of 300 kN. The column is provided with 6 bars of 12 mm diameter. Find the stress
in the concrete and the bar. Take E for bar = 2 × 105 N/mm2 and E for concrete
= 1.4 × 104 N/mm2.
Objective Type Questions
1. Study of forces and the conditions of equilibrium of bodies subjected to the
action of forces is known as
(a) Statics (b) Dynamics (c) Kinematics (d) Kinetics
2. The action of one body on another body that changes or tends to change the
state of the body is known as
(a) Mass (b) Rigid body (c) Force (d) None of these
3. The intensity of the force is its
(a) Magnitude (b) Direction (c) Sense (d) Arrow head
4. Force can be characterized by
(a) Magnitude (b) Direction
(c) Point of application (d) All of these
5. The forces whose lines of action are passing through one point, are known as
(a) Concurrent force system (b) Non-concurrent force system
(c) Parallel force system (d) None of these
6. The forces whose lines of actions are passing through one common line of
action are known as
(a) Concurrent force system (b) Non-concurrent force system
(c) Collinear force system (d) Parallel force system
7. The forces whose lines of actions are parallel to each other and lie in the same
plane are known as
(a) Concurrent force system (b) Non-concurrent force system
(c) Collinear force system (d) Parallel force system
8. The single force that can replace the original system of forces without
changing its external effect is known as
(a) Resultant (b) Couple (c) Moment (d) None of these
9. The resultant of the two forces by parallelogram law be represented in
magnitude and direction by
(a) Diagonal of the parallelogram which does not pass through point of
intersection of two forces
(b) Diagonal of the parallelogram which passes through point of intersection
of two forces
(c) Longer side of the other two sides
(d) Shorter side of the other two sides
10. The resultant of two forces P1 and P2 acting at an angle θ is
2 2
(a) P1 + P2 + 2 P1 P2 cos q
2 2
(b) P1 - P2 - 2 P1 P2 cos q
OTQ.2  Engineering Mechanics Statics

P 2 + P22 + 2 P1 P2 sin q P12 + P22 - 2 P1 P2 sin q


(c) 1 (d)
11. The resultant is maximum and minimum respectively when the angles
between two forces are
(a) 90° and 0° (b) 180° and 90° (c) 0° and 180° (d) 90° and 180°
12. If two forces acting simultaneously at a point be represented in magnitude
and direction by two sides of triangle taken in order, their resultant may be
represented in magnitude and direction by the third side of triangle taken in
opposite order, this is
(a) Parallelogram law (b) Triangle law
(c) Polygon law (d) None of these
13. The resultant of two equal concurrent forces of magnitude P is
θ θ θ θ
(a) 2 P cos (b) 2 P sin (c) P cos (d) 3P cos
2 2 2 2
14. The resultant of two equal concurrent forces of magnitude P and angle
between them 90° is
p p 2
(a) (b) (c) 2P (d)
2 2 p
15. The resultant of more than two concurrent forces is
(a) (ΣV ) 2 (b) (ΣH )2 − (ΣV ) 2
2 2
(c) (ΣH )2 (d) (ΣH ) + (ΣV )
16. Moment of a force produce
(a) Rotation of a body about fixed point
(b) Translation of a body about fixed point
(c) Rotation and translation of a body about a fixed point
(d) None of these
17. Varignon’s theorem can be applied to determine
(a) Position of resultant (b) Location of centroid
(c) Magnitude of resultant (d) None of these
18. A couple is a
(a) Pair of parallel forces of different magnitude separated by distance and
acting in opposite direction.
(b) Pair of parallel forces of same magnitude separated by distance and
acting in opposite direction.
(c) Pair of parallel forces of different magnitude separated by distance and
acting in same direction.
(d) Pair of parallel forces of same magnitude separated by distance and
acting in same direction.
Objective Type Questions  OTQ.3

19. Consider following statements.


The couple is unchanged if, I) it is shifted to any other position, II) it is
rotated through any angle , III) it is replaced by another pair of forces with
same rotational effect, of these statements
(a) I alone is correct (b) II alone is correct
(c) I and II are correct (d) I, II and III are correct
20. Moment of a couple is
(a) Independent of a point (b) Depend upon point
(c) Depend upon axis (d) None of these
21. If moment of force applied on a door is 16 N.m and force applied is 4 N then
distance of handle from pivot is
(a) 2 m (b) 6 m (c) 4 m (d) 8 m
22. If moment arm is zero, then moment produce will be
(a) Doubled (b) 1 (c) Zero (d) None of these
23. Moment depends upon
(a) Magnitude of force (b) Moment arm
(c) both (a) and (b) (d) none of these
24. If the body is at rest or in uniform velocity, it is said to be in
(a) Rest (b) Equilibrium
(c) Uniform motion (d) None of these
25. Number of forces acting at a point will be in equilibrium if
(a) Their sum in horizontal direction is zero S = 0
(b) Their sum in vertical direction is zero S = 0
(c) Both (a) and (b)
(d) None of these
26. Consider following statement
Two forces can be in equilibrium only if they are I) equal in magnitude
II) Opposite in direction III) Collinear in action. Of these statements
(a) I and II are correct (b) I and III are correct
(c) II and III are correct (d) All are correct
27. For equilibrium of co-planer non-concurrent force system, the following
conditions are to be satisfied
(a) SH = 0, SV = 0 (b) SH = 0, SM = 0
(c) SV = 0, SM = 0 (d) SH = 0, SV = 0, SM = 0
28. A sketch of the body isolated from its surrounding is known as
(a) Free body diagram (b) Equilibrium
(c) Force system diagram (d) None of these
OTQ.4  Engineering Mechanics Statics

29. The resultant of a system of forces acting on a body is zero, this state is
known as
(a) Co-planar forces (b) Equilibrium
(c) Free body diagram (d) None of these
30. “If a body is in equilibrium under the action of three co-planar and concurrent
forces, each of the forces is proportional to the sine of the angle between the
other two”, this is the statement of
(a) Triangle law of forces (b) Lami’s theorem
(c) Parallelogram law of forces (d) Polygon law of forces
31. If a body is acted upon by a number of co-planar non-concurrent forces, it
may
(a) Rotate about itself without moving
(b) Move in any one direction rotating about itself
(c) Be in equilibrium
(d) All of these
32. If three co-planar forces acting upon a rigid body keep it in equilibrium, then
they must
(a) Meet at a point (b) Be all parallel
(c) Both (a) and (b) (d) None of these
33. The concurrent force system is in equilibrium if their resultant is
(a) Zero (b) Positive (c) Negative (d) None of these
34. Lami’s theorem is applicable for
(a) Two concurrent forces in equilibrium
(b) Three concurrent forces in equilibrium
(c) More than three concurrent forces in equilibrium
(d) None of these
35. The normal reaction on a smooth roller from horizontal surface will act
(a) Horizontal to the plane of contact
(b) Perpendicular to the plane of contact
(c) Inclined to the plane of contact
(d) None of these
36. Simplest form of a perfect frame is
(a) Rectangle (b) Square (c) Triangle (d) Pentagon
37. Minimum number of members that form a simple truss are
(a) 2 (b) 3 (c) 4 (d) 5
38. Redundant truss is a type of
(a) Perfect truss (b) Imperfect truss
(c) Stable truss (d) None of the above
Objective Type Questions  OTQ.5

39. Which axial force is determined while analyzing a truss? 


(a) Compressive force (b) Tensile force
(c) Both (a) and (b) (d) None of the above
40. In analysis of truss by method of joints, the number of unknowns at a joint
should not be
(a) > 2 (b) > 3 (c) > 4 (d) None of these
41. In analysis of truss by method of sections, the number of unknowns at a
section should not be
(a) > 3 (b) > 4 (c) > 5 (d) None of these
42. Method of joints is more suitable when
(a) Forces in few of the members only is desired
(b) Forces in all the members are desired
(c) Reactions at supports are desired
(d) None of these
43. Method of sections is more suitable when
(a) Forces in few of the members only is desired
(b) Forces in all the members are desired
(c) Reactions at supports are desired
(d) None of these
44. The force system in method of joints involves
(a) Collinear forces (b) Concurrent forces
(c) Parallel forces (d) Non concurrent forces
45. The force system in method of joints involves
(a) Collinear forces (b) Concurrent forces
(c) Parallel forces (d) Non concurrent forces
46. Beam is a structural member subjected to
(a) Axial load (b) Transverse load
(c) Twisting moment (d) No load
47. In cantilever beam
(a) One end is fixed and other end is free
(b) Both ends are fixed
(c) Both ends are supported on roller
(d) None of these
48. In simply supported beam
(a) One end is fixed and other end is free
(b) Both ends are fixed
(c) One end is supported on roller and other end is hinged
(d) None of these
OTQ.6  Engineering Mechanics Statics

49. The bending moment of cantilever carrying a point load at free end is
(a) Parabola
(b) Triangle with maximum height at fixed end
(c) Triangle with maximum height at free end
(d) None of these
50. A sudden increase or decrease in shear force diagram between two points
indicate
(a) No loading
(b) Distributed load between two points
(c) Concentrated load between two points
(d) None of these
51. If the bending moment diagram is parabolic curve between two points then
it indicate
(a) No loading
(b) Uniformly distributed load between two points
(c) Concentrated load between two points
(d) None of these
52. Bending moment diagram for any part of a simply supported beam between
two concentrated load is
(a) Horizontal straight line (b) Vertical straight line
(c) Inclined line (d) Parabola
53. The point through which the whole weight of the body acts is known as
(a) Center of gravity (b) Center of mass
(c) Centroid (d) Moment of inertia
54. The centroid and centre of gravity coincide if
(a) The body has uniform density
(b) Centre of gravity and center of mass coincide
(c) g is uniform throughout the Earth
(d) None of these
55. Centre of gravity of a body is a
(a) Point in the body at which g is constant.
(b) Point in the body which changes with orientation of the body.
(c) Point in the body at which the entire weight is assumed to be concentrated.
(d) None of these.
56. The first moment of an area about the x-axis is
(a) ∫ x dA (b) ∫ y dA (c) ∫x
2
dA (d) ∫y
2
dA
Objective Type Questions  OTQ.7

57. If an area has an axis of symmetry then


(a) Its first moment about that axis is zero
(b) Its centroid lies on that axis
(c) Both (a) and ((b)
(d) None of these
58. For an area having two axes of symmetry, the centroid lies on
(a) Horizontal axis (b) Vertical axis
(c) Intersection point of two axes (d) Not on any axes
59. If an area is symmetric about x axis, its
(a) x = 0 (b) y = 0
(c) Both (a) and (b) (d) None of these
60. If an area is symmetric about y axis, its
(a) x = 0 (b) y = 0
(c) Both (a) and (b) (d) None of these
61. The centroid of a semi-circle lies at a distance of __________ from its base
measured along the vertical radius.
(a) 4r/ 3π (b) 3r/ 8 (c) 3r/4π (d) 4r/ 2π
62. The unit of coordinates of centroid from reference axis is in
(a) kg (b) N.mm (d) gram (d) mm
63. Moment of inertia is denoted by
(a) I (b) C (c) M (d) K
64. The unit of moment of inertia is
(a) L (b) L2 (c) L3 (d) L4
65. Second moment are also termed as
(a) Center of gravity (b) Moment of force
(c) Moment of inertia (d) Couple
66. According to perpendicular axis theorem,
(a) Iz = Ix – Iy (b) Iz = Ix + Iy (c) Ix = Iz + Iy (d) Iy = Ix + Iz
67. The moment of inertia of a rectangle base ‘b’ and depth ‘d’ about the base is
bd 2 bd 8 bd 8 bd 8
(a) (b) (c) (d)
6 12 12 3
68. The moments of inertia of an area about x and y axes are Ix and Iy respectively.
Its polar moment of Inertia is
(a) Ix Iy (b) Ix/Iy (c) Ix + Iy (d) Ix – Iy
69. The moment of inertia of a circular section of diameter d is
π π π π 4
(a) (d )
4
(b) (d ) 4 (c) (d ) 4 (d) (d )
16 32 64 48
OTQ.8  Engineering Mechanics Statics

70. Area moment of inertia relative to a non-centroidal axis is


(a) Larger than centroidal axis values
(b) Smaller than centroidal axis values
(c) Can be larger or smaller than centroidal axis value
(d) Equal to centroidal axis values
71. Moment of inertia can also be calculated by
(a) Integration (b) Differentiation
(c) Moments (d) None of these
72. The moment of inertia of a triangle base ‘b’ and depth ‘d’ about the centroidal
axis is
bd 2 bd 8 bd 3 bd 8
(a) (b) (c) (d)
6 12 24 36
73. The moment of inertia of a triangle base ‘b’ and depth ‘d’ about the base is
bd 2 bd 8 db8 bd 8
(a) (b) (c) d (d)
6 12 24 36
74. The relation used by radius of gyration is _____
(a) I = Ak (b) I = A2k (c) I = Ak2 (d) I = A2k2
75. If X-X is the centrodial axis of an area ‘A’ and A-B is another axis at a
distance ‘d’ and parallel to X-X, then by parallel axis theorem
(a) IXX = IAB + Ad2 (b) IAB = IXX + Ad2
(c) IAB + IXX = Ad2 (d) None of these
76. Moment of inertia of any composite area about an axis passing through its
centroid is
(a) Minimum
(b) Maximum
(c) Depend upon dimensions of area
(d) None of these
77. The polar moment of inertia is measured about
(a) x axis (b) y axis (c) z axis (d) None of these
78. Moment of inertia of hollow circular section of outer diameter D and inner
diameter d is
π π π π
(a) ( D 4 − d 4 ) (b) ( D 4 − d 4 ) (c) ( D 4 – d 4 ) (d) (D4 – d 4 )
16 32 64 48
79. Friction force act in a direction ______ to the direction of motion of an
object.
(a) Same (b) Perpendicular ( c) Opposite (d) None of these
80. The maximum frictional force which comes into play when a body just
begins to slide on a surface is known as
Objective Type Questions  OTQ.9

(a) Static friction (b) Kinetic friction


(c) Limiting friction (d) None of these
81. The body will move only when
(a) Force of friction = applied force
(b) Force of friction < applied force
(c) Force of friction > applied force
(d) All of the above
82. Which friction of the following is the highest?
(a) Static friction (b) Kinetic friction
(c) Rolling friction (d) None of these
83. Friction force is a
(a) Scalar quantity (b) Vector quantity
(c) Can be scalar or vector (d) None of these
84. To start a crate moving requires ___________ to keep it moving at a constant
speed on a horizontal rough surface.
(a) More force than (b) The same force
(c) Less force than (d) None of these
85. A body of weight W is placed on an inclined plane. The angle made by the
inclined plane with the horizontal, when the body is on the point of moving
down is called
(a) Angle of inclination (b) Angle of friction
(c) Angle of repose (d) None of these
86. The ratio of static friction to kinetic friction is
(a) Equal to one (b) Greater than one
(c) Less than one (d) None of these
87. Static friction is always
(a) Equal to kinetic friction (b) Less than kinetic friction
(c) Greater than kinetic friction (d) None of these
88. Coefficient of friction depends upon
(a) Nature of surface (b) Area of contact
(c) Both (a) and (b) (d) None of these
89. If the angle of friction is 25°, the coefficient of friction is
(a) 0.404 (b) 0.466 (c) 0.927 (d) None of these
90. The ratio of the limiting force of friction (F) to the normal reaction (N) is
known as
(a) Force of friction (b) Angle of friction
(c) Angle of repose (d) Coefficient of friction
91. The coefficient of friction (µ) is equal to
(a) tan φ (b) cos φ (c) sin φ (d) None of these
OTQ.10  Engineering Mechanics Statics

92. Stress is
(a) External force (b) Friction force
(c) Internal resistive force (d) None of these
93. The ratio of linear stress to linear strain is known as
(a) Poisson’s ratio (b) Bulk modulus
(c) Modulus of rigidity (d) Modulus of elasticity
94. The ratio of lateral strain to longitudinal strain is called
(a) Modulus of elasticity (b) Modulus of rigidity
(b) Poisson’s ratio (c) None of these
95. Which of the following is a dimensionless quantity?
(a) Poison’s ratio (b) Strain
(c) Both (a) and (b) (d) None of these
96. The total extension in a bar, consists of 3 bars of same material, of varying
sections is
(a) P/E(L1/A1+L2/A2+L3/A3) (b) P/E(L1A1+L2A2+L3A3)
(c) PE(L1/A1+L2/A2+L3/A3) (d) PE(L1/A1+L2/A2+L3/A3)
Where P = Load applied, E = young’s modulus for the bar, L1,2,3 = Length
of corresponding bars, A1,2,3 = Area of corresponding bars.
97. The change in length due to tensile or compressive force acting on a bar is
given by
PLA E AE PL
(a) (b) (c) (d)
E PLA PL AE
Where, P = force acting on a bar; L = length of the bar; E = modulus of
elasticity; A = cross section area
98. When compressive stress is applied axially on a circular rod its
I. diameter increases II. length decreases
III. volume decreases
Of these statements,
(a) Only I is correct (b) Only II is correct
(c) Both I and II are correct (d) None of these
99. Tensile Strain is
(a) Increase in length / original length
(b) Decrease in length / original length
(c) Change in volume / original volume
(d) None of these
100. Hooke’s Law is truly valid up to
(a) Elastic limit (b) Proportional limit
(c) Plastic limit (d) None of these
Review Problems
1. The resultant of two forces P and Q is 1400 N vertical. Determine the force
Q and the corresponding angle θ for the system of forces as shown in figure.

2. Determine the magnitude of force F and direction θ so that the resultant of


the three forces as shown in figure is vertically downward with a magnitude
of 6 kN.

3. Determine the support reactions for the beam loaded as shown in figure.

4. Two identical rollers, each of weights 1000 N are supported by an inclined


plane as shown in figure. Assuming smooth surfaces find the reactions
induced at the points of supports.
R.2  Engineering Mechanics Statics

5. Two spheres A and B of diameter 80 mm and 120 mm respectively are held


in equilibrium by separate strings as shown in figure. Sphere B rests against
vertical wall. If masses of sphere A and B are 10 kg and 20 kg respectively,
determine the tension in the string and reactions at point of contact.

6. Locate the coordinates of centroid of shaded region shown in figure.

7. Determine the y coordinate of the centroid of shaded area shown in figure.


Review Problems  R.3

8. A block of weight W1 = 1290 N rests on a horizontal surface and supports


another block of weight W2 = 570 N on top of it as shown in figure. Find the
force applied to lower block that will be necessary to cause the slipping to
impend. Assume coefficient of friction between blocks = 0.25 and between
block 1 and horizontal surface = 0.40.

9. Determine the forces in each member of truss shown in figure.

10. A tensile load of 50 kN is acting on the rod of 50 mm diameter and length of


5 m. Determine length of a bore of 25 mm diameter that can be made central
in the rod, if the total extension is not to exceed by 25% under the same
tensile load.
Take E = 2.05 × 105 N/mm2
Index
A Clockwise moment 1.13, 1.14, 1.15, 1.43,
Analysis of truss 3.1, 3.4 1.48, 1.55, 2.24, 3.13, 3.17, 3.22,
3.34, 3.44
Angle of friction 5.5, 5.28, 5.38
Coefficient of friction 5.4, 5.36
Angle of repose 5.5, 5.6, 5.38
Collinear forces 1.5, 1.7, 1.70
Anticlockwise moment 1.13, 1.14, 1.15,

1.44, 1.45, 1.46, 1.47, 1.48, 1.49, Composite system 6.9
1.50, 1.52, 1.55, 1.56, 1.57, 1.58, 3.6, Coplanar concurrent forces 1.5, 1.6
3.9, 3.13, 3.13, 3.14, 3.15, 3.16, 3.17, Coplanar non-concurrent forces 1.5,
3.17, 3.18, 3.22, 3.24, 3.26, 3.28, Coplanar parallel forces 1.6
3.30, 3.32, 3.34, 3.36, 3.38, 3.40,
Co-planer concurrent forces 1.35,
3.42, 3.44, 3.46, 5.20, 5.21, 5.23,
5.24, 5.33 Coulomb’s theory of dry friction
Applied forces 2.4, 6.3 Couple 1.1, 1.16, 1.17, 1.59, 1.65, 2.8, 3.21
Area M.I. of rectangle 4.22 4.23 D
Area M.I. of triangle 4.23, 4.24 Determination of centroid 4.1
Area moment of inertia 4.1, 4.19, 4.23, E
4.24, 4.26, 4.29, 4.31, 4.32 Equilibrium 0.1, 2.1, 2.3, 2.8, 2.9, 2.10,
Area moment of inertia 4.1, 4.19, 4.23, 2.14, 2.12, 2.19, 2.34, 2.5, 3.5, 3.13,
4.24, 4.26, 4.29, 4.31, 4.32 3.21, 3.22, 3.34, 3.44, 5.6, 5.7, 5.9,
5.20, 5.29
B
Equilibrium of forces 2.10
Bar of varying cross-section 6.6
Equilibrium under three force system 2.9
Beam 3.1, 3.19, 3.20, 3.21, 3.22, 3.34,
3.44, 3.46, 3.48 Equilibrium under two force system 2.8
Belt friction 5.34 Equivalent systems of forces 1.18
Bending moment 3.21, 3.22, 3.30 3.32, F
3.34, 3.42 3.44, 3.48 4.19 Factor of safety 6.9
Bending moment diagram 3.22, 3.25, 3.34, Force 0.3, 0.4, 0.5, 1.1, 1.3, 1.4, 1.7, 1.8,
3.40, 3.42, 3.44, 3.48 1.9, 1.10, 1.13, 1.14, 1.15, 1.16, 1.17,
Block friction 5.9 1.18, 1.26, 1.35, 1.43, 1.55, 1.59, 1.70,
1.71, 2.3, 2.6, 2.8, 2.10, 2.34, 3.3, 3.4,
Bulk modulus 6.6
3.5, 3.13, 3.19, 3.21, 3.22, 3.34, 3.44,
C 3.48, 4.3, 4.18, 5.3, 5.4, 5.5, 5.6, 5.7,
Cantilever beam 3.20, 3.34, 3.38, 3.42, 5.9, 5.26, 5.34, 5.38, 6.3, 6.5, 6.9
3.48 Force–couple system 1.17
Center of gravity 4.1, 4.3, 4.5, 4.18, Force-couple system and equivalent
Center of mass 4.3, 4.18 system 1.59
Centroid 4.3, 4.5, 4.6, 4.7, 4.8, 4.10, 4.14, Free body diagram 2.3, 2.6, 3.19, 3.22,
4.18, 4.19, 4.21, 4.22, 4.26 3.34, 3.44, 4.3, 5.3, 5.5, 5.6, 5.7, 5.9
Centroid of a triangular area 4.5 Friction 1.4, 2.5, 3.4, 5.1, 5.3, 5.4, 5.5, 5.6,
Centroid of semicircular area 4.6 5.7, 5.20, 5.26, 5.34, 5.38,
Characteristics of the force 1.3, 1.70
I.2  Index

H R
Hooke’s law 6.6 Radius of gyration 4.22, 4.32
L Reactive forces 1.4, 2.4
Ladder friction 5.6, 5.20, 5.50 Resolution 1.9, 1.10, 1.26, 1.70
Lami’s theorem 2.9, 5.28 Resolution of forces 1.26
Lateral strain 6.5, 6.6 Resultant 0.4, 1.7, 1.14, 1.15, 1.16, 1.18,
Law of transmissibility 0.4 1.26, 1.35, 1.55, 1.70, 2.34, 4.3, 5.5
Limiting friction 5.4, 5.38 Resultant moment 1.14, 1.18
Longitudinal strain 6.5, 6.6, Resultant of collinear forces
M Resultant of concurrent forces 1.7
Mechanics 0.3 0.4, 0.5, 1.3, 2.3, Resultant of parallel force system 1.55,
Method of joints 3.4, 3.5 1.70
Method of sections 3.4, 3.5, 3.13 Rigid body 0.3, 1.18, 1.70, 2.3, 2.6, 4.3,
Modules of rigidity 6.6, S
Modulus of elasticity 6.6, 6.19, Shear force 3.21, 3.22, 3.30, 3.34, 3.44,
3.48, 6.4, 6.19
Moment 1.13, 1.14, 1.15, 1.17, 1.18, 1.43,
1.59, 2.3, 3.21, 3.22, 3.34, 3.44, 3.48, Shear force diagram 3.25, 3.30,
4.3, 4.19, 4.21, 4.22, 4.26, 4.32, Shear stress 6.3, 6.4, 6.18, 6.19
Moment & non-concurrent forces 1.43 Simply supported beam 3.20, 3.22, 3.48
Moment of a couple 1.17 Solved examples based on 1.18, 1.26,
N 1.35, 1.43, 1.55, 1.59, 2.10, 3.5,
3.13, 3.22, 3.34, 3.44, 4.8, 4.26, 5.9,
Newton’s law of gravitation 0.4
5.20, 5.26, 5.34, 6.9
Newton’s laws of motion 0.4
Space 0.3, 3.3,
Non-coplanar concurrent forces 1.6
Strain 6.3, 6.4, 6.5, 6.6, 6.7, 6.8, 6.9
Non-coplanar non-concurrent forces 1.6
Stress 6.3, 6.6, 6.7, 6.8, 6.9, 6.18
Non-coplanar parallel forces 1.6
Stress and strain 6.1, 6.8, 6.9, 6.19
Non-perpendicular resolution 1.10, 1.12
Stress-strain diagram 6.7
Normal stress 6.3, 6.6, 6.18
System of forces 1.3, 1.4, 1.7, 1.70, 2.3,
O 2.10
Overhang beam 3.44 T
P Temperature strain 6.8, 6.19
Parallel axis theorem 4.21, 4.26, 4.32 Temperature stress 6.8
Parallel forces 1.6, 1.15, 1.55, 2.8 Triangle law of forces 1.8
Parallelogram law of forces 1.7, 1.18 Truss 3.3, 3.4, 3.5, 3.19
Parallelogram law of forces 1.7, 1.18 Types of beams 3.19
Particle 0.4, 2.3, 2.6, 3.4, 4.3 Types of loading 3.20
Perpendicular axis theorem 4.20 Types of truss 3.3
Perpendicular resolution 1.10, 1.12, 1.26 V
Poisson’s ratio 6.6 Varignon’s theorem 1.14, 1.15, 1.50
Polygon law of forces 1.9 Volumetric strain 6.6, 6.19
W
Wedge friction 5.7, 5.26

Вам также может понравиться